123
TỔNG HỢP CÁC BÀI TOÁN TỪ ĐỀ THI TUYỂN SINH CHUYÊN TOÁN CỦA CÁC TỈNH THÀNH PHỐ −1 Blog TOÁN HỌC CHO MỌI NGƯỜI https://thcmn.wordpress.com/ https://www.facebook.com/thcmn/ [email protected] Đơn vị tài trợ

1 ¾ F - · PDF fileđang chuẩn bị cho kì thi tuyển sinh vào các lớp 10 ... phẩm “Tuyển tập các bài toán trong đề thi ... đam mê với các môn học

  • Upload
    trananh

  • View
    244

  • Download
    8

Embed Size (px)

Citation preview

Page 1: 1 ¾ F - · PDF fileđang chuẩn bị cho kì thi tuyển sinh vào các lớp 10 ... phẩm “Tuyển tập các bài toán trong đề thi ... đam mê với các môn học

TỔNG HỢP CÁC BÀI TOÁNTỪ ĐỀ THI TUYỂN SINH CHUYÊN TOÁN

CỦA CÁC TỈNH – THÀNH PHỐ

−1 Blog TOÁN HỌC CHO MỌI NGƯỜI

https://thcmn.wordpress.com/

https://www.facebook.com/thcmn/

[email protected]

Đơn vị tài trợ

Page 2: 1 ¾ F - · PDF fileđang chuẩn bị cho kì thi tuyển sinh vào các lớp 10 ... phẩm “Tuyển tập các bài toán trong đề thi ... đam mê với các môn học
Page 3: 1 ¾ F - · PDF fileđang chuẩn bị cho kì thi tuyển sinh vào các lớp 10 ... phẩm “Tuyển tập các bài toán trong đề thi ... đam mê với các môn học

VÕ TRẦN DUY – VÕ THÀNH ĐẠT – LƯƠNG VĂN KHẢI – ĐẶNG NHÌ – NGUYỄN DUY TÙNG

TRẦN DƯƠNG VIỆT HOÀNG – PHẠM THỊ HỒNG NHUNG – PHẠM QUỐC THẮNG

NGÔ HOÀNG ANH – NGUYỄN TRƯỜNG HẢI

TUYỂN TẬP CÁC BÀI TOÁN TRONG ĐỀ THI TUYỂN SINH CHUYÊN TOÁN

CỦA CÁC TỈNH – THÀNH PHỐ

Tháng 4 – 2017

Page 4: 1 ¾ F - · PDF fileđang chuẩn bị cho kì thi tuyển sinh vào các lớp 10 ... phẩm “Tuyển tập các bài toán trong đề thi ... đam mê với các môn học

Đây là tài liệu miễn phí. Bất cứ ai cũng có thể tải về và chia sẻ đến

những người khác, nhưng khi chia sẻ, vui lòng ghi rõ nguồn tài liệu.

Mọi hành động sử dụng tài liệu này vào mục đích thương mại đều

phải được sự cho phép bằng văn bản của THCMN, nếu không sẽ bị

coi là vi phạm bản quyền.

Page 5: 1 ¾ F - · PDF fileđang chuẩn bị cho kì thi tuyển sinh vào các lớp 10 ... phẩm “Tuyển tập các bài toán trong đề thi ... đam mê với các môn học

LỜI NÓI ĐẦU

“Đi nhiều người, bạn sẽ đi rất xa.”

Kì thi tuyển sinh vào bậc THPT luôn là một kì thi cam go, quyết liệt đối với các bạn

học sinh, nhất là các bạn học sinh muốn thi vào các trường chuyên. Thông thường, một lớp

chuyên chỉ có khoảng dưới 40 học sinh, nhưng số lượng các bạn học sinh đăng kí thi vào lớp

chuyên đó luôn ở mức hàng trăm, thậm chí hàng nghìn. Nói như vậy để thấy rằng, vượt qua kì

thi tuyển sinh vào các lớp chuyên luôn là một thử thách lớn đối với các thí sinh, và điều đó đòi

hỏi sự chuẩn bị, ôn tập kĩ lưỡng và những kĩ năng vững vàng đến từ các bạn.

Tiếp nối thành công đến từ các ấn phẩm trước, với mong muốn giúp các bạn học sinh

đang chuẩn bị cho kì thi tuyển sinh vào các lớp 10 chuyên Toán có một nguồn tài liệu đầy đủ

và chất lượng để ôn tập trong giai đoạn nước rút, Blog Toán học cho mọi người cho ra mắt ấn

phẩm “Tuyển tập các bài toán trong đề thi tuyển sinh chuyên Toán của các tỉnh – thành phố”.

Trong cuốn sách này, để thuận tiện cho các bạn theo dõi, chúng tôi chia các bài toán ra làm 5

lĩnh vực: Bất đẳng thức, Đại số, Hình học, Số học, Tổ hợp. Mỗi bài toán đều có hướng dẫn giải

hoặc lời giải đầy đủ ở phần sau.

Các biên tập viên từng phần của ấn phẩm này gồm có:

• Bất đẳng thức: Võ Thành Đạt (Sinh viên hệ Cử nhân tài năng khoa Toán – Tin học

trường Đại học Khoa học tự nhiên, ĐHQG Tp. HCM) và Phạm Quốc Thắng (Học sinh

chuyên Toán trường THPT chuyên Long An, tỉnh Long An).

• Đại số: Võ Trần Duy (Sinh viên hệ Cử nhân tài năng khoa Toán – Tin học trường Đại

học Khoa học tự nhiên, ĐHQG Tp. HCM) và Ngô Hoàng Anh (Học sinh chuyên Toán

trường Phổ thông Năng khiếu, ĐHQG Tp. HCM).

• Hình học: Lương Văn Khải (Sinh viên hệ Cử nhân tài năng khoa Toán – Tin học trường

Đại học Khoa học tự nhiên, ĐHQG Tp. HCM) và Nguyễn Duy Tùng (Sinh viên đại học

Wabbash, Hoa Kỳ).

• Số học: Phạm Thị Hồng Nhung (Học sinh chuyên Toán trường THPT chuyên Lê Quý

Đôn, tỉnh Bà Rịa – Vũng Tàu) và Nguyễn Trường Hải (Học sinh chuyên Toán trường

THPT chuyên Trần Hưng Đạo – Bình Thuận).

• Tổ hợp: Đặng Nhì (Sinh viên hệ Cử nhân tài năng khoa Toán – Tin học trường Đại học

Khoa học tự nhiên, ĐHQG Tp. HCM) và Trần Dương Việt Hoàng (Học sinh chuyên

Toán trường Phổ thông Năng khiếu, ĐHQG Tp. HCM).

Chúng tôi xin bày tỏ lòng biết ơn sâu sắc đến TS Trần Nam Dũng (Trường Đại học

Khoa học Tự nhiên ĐHQG Tp. HCM) đã luôn động viên và giúp đỡ chúng tôi trong quá trình

hoàn thành cuốn tài liệu này. Trân trọng cảm ơn anh Huỳnh Phước Trường (Sinh viên trường

Đại học Sư phạm Tp. HCM) đã chỉnh sửa và trang trí lại các phần của cuốn sách, chị Đỗ Thị

Lan Anh (Chủ nhiệm CLB Học thuật khoa Toán – Tin học trường Đại học Khoa học tự nhiên

– ĐHQG Tp. HCM) đã đưa ra những nhận xét thẳng thắn cho bản thảo ấn phẩm. Cảm ơn các

cộng tác viên Võ Ngọc Trăm (Sinh viên khoa Toán – Tin học trường Đại học Khoa học Tự

nhiên, ĐHQG Tp. HCM) đã sưu tập đề thi, Lư Thương Thương (Học sinh chuyên Toán trường

THPT chuyên Lê Hồng Phong, Tp. HCM) đã đọc và kiểm tra bản thảo. Cảm ơn các thầy cô và

Page 6: 1 ¾ F - · PDF fileđang chuẩn bị cho kì thi tuyển sinh vào các lớp 10 ... phẩm “Tuyển tập các bài toán trong đề thi ... đam mê với các môn học

các thành viên của Diễn đàn Toán học Việt Nam – VMF (

https://diendantoanhoc.net/home/), Thư viện trực tuyến Violet

(https://violet.vn/) , Mathscope (http://mathscope.org/index.php) đã

cung cấp một số đề thi và đáp án. Đặc biệt, chúng tôi xin cảm ơn Công ty cổ phần Giáo dục

Titan – Titan Education đã tài trợ kinh phí cho chúng tôi hoàn thành ấn phẩm này.

Chúng tôi rất mong nhận được những ý kiến đóng góp của các bạn để những lần biên

tập các ấn phẩm khác được hoàn thiện hơn. Mọi nhận xét và góp ý xin gửi về email

[email protected] hoặc gửi tin nhắn đến fanpage Toán học cho

mọi người – Math for Everyone (https://www.facebook.com/thcmn/).

Cảm ơn tất cả các bạn!

Page 7: 1 ¾ F - · PDF fileđang chuẩn bị cho kì thi tuyển sinh vào các lớp 10 ... phẩm “Tuyển tập các bài toán trong đề thi ... đam mê với các môn học

GIỚI THIỆU VỀ ĐƠN VỊ TÀI TRỢ

CÔNG TY CỔ PHẦN GIÁO DỤC TITAN

Khi giáo dục ngày càng phát triển, việc chọn cho con em mình môi trường học tập tốt

luôn là điều trăn trở của các bậc phụ huynh. Ở trường, các môn tự nhiên khiến học sinh gặp

không ít khó khăn, bởi đa số các em chưa nắm vững kiến thức căn bản từ lớp trước, đặc biệt

chưa có phương pháp học hợp lý.

Được thành lập vào năm 2010, TITAN Education là trung tâm đào tạo và bồi dưỡng

văn hóa trực tiếp nhằm mang lại môi trường giáo dục tốt nhất, nơi các em có thể thỏa chí

đam mê với các môn học đầy lý thú cũng như các khóa học bổ ích giúp phát triển tối đa kỹ

năng, sở trường của mỗi em. Các chương trình được giảng dạy tại TITAN:

• Toán, Lý, Hóa, Văn, Anh văn cơ bản – nâng cao

• Toán chuyên

• Toán IQ cho thiếu nhi

• Luyện thi THPT Quốc gia

• Toán tiếng Anh

• Lớp hè đặc biệt – 9 tuần thử thách

• Bồi dưỡng học sinh giỏi - Gặp gỡ toán học

Page 8: 1 ¾ F - · PDF fileđang chuẩn bị cho kì thi tuyển sinh vào các lớp 10 ... phẩm “Tuyển tập các bài toán trong đề thi ... đam mê với các môn học

Với tập thể giảng viên giàu kinh nghiệm cùng chương trình đào tạo phù hợp, TITAN sẽ

giúp học sinh củng cố các kiến thức cơ bản và dần dần tiếp cận với các kiến thức nâng cao,

rèn khả năng tư duy, hình thành kỹ năng giải các bài tập, giúp các em đạt kết quả tốt ở trường

cũng như các kỳ thi chuyển cấp và kỳ thi tuyển sinh Đại học. Đối với những em có năng khiếu

đặc biệt về toán, các giảng viên tại TITAN sẽ bồi dưỡng dạy chuyên sâu và nâng cao hơn, để

các em có thể tự tin tham dự các kỳ thi Olympic, học sinh giỏi Quốc gia, Quốc tế.

Phối hợp với gia đình, TITAN luôn chủ động liên hệ, thông tin đến phụ huynh về tình

hình học tập của các em. Đặc biệt, TITAN còn có những bài kiểm tra định kỳ để theo dõi việc

học tập cũng như rèn kỹ năng làm bài thi cho từng học sinh, và báo cáo kết quả cho phụ huynh

bằng Phiếu báo học tập.

Nhằm đảm bảo chất lượng dạy và học, TITAN bố trí lớp học không quá 20 học sinh

cùng một giảng viên và một trợ giảng. Với môi trường thân thiện, gần gũi, cơ sở vật chất khang

trang, các phòng học được thiết kế đạt chuẩn quốc tế, Học viện còn có môi trường hoạt động

thể dục thể thao... để giúp học viên có được sức khỏe tốt phục vụ cho việc học và những kĩ

năng cần thiết hỗ trợ cho công việc sau này.

Thư viện hiện đại với nhiều sách chuyên nghành phục vụ cho nhu cầu nâng cao kiến

thức của học viên. Bên canh đó trường còn trang bị các phòng tự học cho học viên và những

phòng để học viên làm việc nhóm nhóm theo đề tài giảng viên đưa ra.

Page 9: 1 ¾ F - · PDF fileđang chuẩn bị cho kì thi tuyển sinh vào các lớp 10 ... phẩm “Tuyển tập các bài toán trong đề thi ... đam mê với các môn học

THCM

NMục lục

1 CÁC BÀI TOÁN . . . . . . . . . . . . . . . . . . . . . . . . . . . . . . . . . . . . . . . . . . . . . . . . 11

1.1 Bất đẳng thức 11

1.2 Đại số 14

1.3 Hình học phẳng 16

1.4 Số học 22

1.5 Tổ hợp 25

2 LỜI GIẢI . . . . . . . . . . . . . . . . . . . . . . . . . . . . . . . . . . . . . . . . . . . . . . . . . . . . . . . . 27

2.1 Bất đẳng thức 27

2.2 Đại số 44

2.3 Hình học phẳng 60

2.4 Số học 104

2.5 Tổ hợp 116

Page 10: 1 ¾ F - · PDF fileđang chuẩn bị cho kì thi tuyển sinh vào các lớp 10 ... phẩm “Tuyển tập các bài toán trong đề thi ... đam mê với các môn học

THCM

N

Page 11: 1 ¾ F - · PDF fileđang chuẩn bị cho kì thi tuyển sinh vào các lớp 10 ... phẩm “Tuyển tập các bài toán trong đề thi ... đam mê với các môn học

THCM

N1. CÁC BÀI TOÁN

1.1 Bất đẳng thứcBài 1. (Trường Phổ thông Năng khiếu ĐHQG Tp. HCM) Biết x ≥ y ≥ z, x+ y+ z = 0 vàx2 + y2 + z2 = 6.

1. Tính S = (x− y)2 +(x− y)(y− z)+(y− z)2

2. Tìm giá trị lớn nhất của P = |(x− y)(y− z)(z− x)|

Bài 2. (Bà Rịa - Vũng Tàu) Cho x,y,z là 3 số dương thỏa mãn x2+y2+ z2 = 3xyz. Chứng minh:

x2

y+2+

y2

z+2+

z2

x+2≥ 1

Bài 3. (Bắc Ninh) Cho a,b,c > 0. Tìm giá trị nhỏ nhất của

M =3a4 +3b4 + c3 +2

(a+b+ c)3

Bài 4. (Bình Định) Cho x,y,z là 3 số thay đổi thỏa mãn x2 + y2 + z2 = 1. Tìm giá trị lớn nhất

của biểu thức:

P = xy+ yz+ zx+12[x2(y− z)2 + y2(z− x)2 + z2(x− y)2]

Bài 5. (Bình Thuận) Cho các số dương x,y,z. Chứng minh rằng

xyx2 + yz+ zx

+yz

y2 + zx+ xy+

zxz2 + xy+ yz

≤ x2 + y2 + z2

xy+ yz+ zx

Bài 6. (Cần Thơ) Cho a,b,c lần lượt là độ dài ba cạnh của một tam giác và thỏa mãn

2ab+3bc+4ca = 5abc. Tìm giá trị nhỏ nhất của biểu thức

P =7

a+b− c+

6b+ c−a

+5

c+a−b

Page 12: 1 ¾ F - · PDF fileđang chuẩn bị cho kì thi tuyển sinh vào các lớp 10 ... phẩm “Tuyển tập các bài toán trong đề thi ... đam mê với các môn học

THCM

N

12 Chương 1. CÁC BÀI TOÁN

Bài 7. (Đồng Nai) Cho a,b,c là các số thực không âm thỏa a+b+ c = 3.

1. Chứng minh rằng: ab+bc+ ca≤ 32. Chứng minh rằng: a2b+b2c+ c2a≤ 4

Bài 8. (Hà Nội) Cho các số thực dương a,b,c thỏa mãn a2 +b2 + c2 = 3. Chứng minh

2a2

a+b2 +2b2

b+ c2 +2c2

c+a2 ≥ a+b+ c

Bài 9. (Hà Tĩnh) Cho a,b,c > 0 thỏa mãn a+b+ c = 2016. Tìm giá trị lớn nhất của

aa+√

2016a+bc+

bb+√

2016b+ ca+

cc+√

2016c+ab

Bài 10. (Hải Dương) Cho a,b,c là các số thực dương thay đổi thỏa mãn a+b+ c = 1. Tìm giátrị nhỏ nhất của biểu thức

Q = 14(a2 +b2 + c2)+ab+bc+ ca

a2b+b2c+ c2a

Bài 11. (Hải Phòng) Cho a,b,c > 0 và a+b+ c≥ 9. Tìm giá trị nhỏ nhất của

A = 2

√a2 +

b2

3+

c2

5+3

√1a+

9b+

25c

Bài 12. (Tp. HCM) Cho x,y là hai số thực dương. Chứng minh rằng

x√

y+ y√

xx+ y

− x+ y2≤ 1

4

Bài 13. (Lào Cai) Cho a,b,c là các số dương thỏa mãn1

a+1+

1b+1

+1

c+1= 2. Chứng minh

18a2 +1

+1

8b2 +1+

18c2 +1

≥ 1

Bài 14. (Long An) Cho a,b,c là 3 cạnh của một tam giác. Tìm giá trị nhỏ nhất của

Q =abc

(b+ c−a)(c+a−b)(a+b− c)

Bài 15. (Nam Định) Cho x,y,z là các số thực thỏa mãn (x− y)(x− z) = 1 và y 6= z. Chứng

minh1

(x− y)2 +1

(y− z)2 +1

(z− x)2 ≥ 4

Page 13: 1 ¾ F - · PDF fileđang chuẩn bị cho kì thi tuyển sinh vào các lớp 10 ... phẩm “Tuyển tập các bài toán trong đề thi ... đam mê với các môn học

THCM

N

1.1 Bất đẳng thức 13

Bài 16. (Ninh Bình) Cho a,b,c là các số thực dương thỏa mãn abc = 1. Chứng minh rằng

1ab+a+2

+1

bc+b+2+

1ca+ c+2

≤ 34

Bài 17. (Ninh Thuận) Cho ba số thực a,b,c thỏa mãn điều kiện ab+bc+ ca = 3. Tìm giá trị

nhỏ nhất của biểu thứcP = a2 +b2 + c2−6(a+b+ c)+2017

Bài 18. (Phú Thọ): Cho các số dương x,y. Tìm giá trị nhỏ nhất của biểu thức

P =2√

(2x+ y)3 +1−1+

2√(x+2y)3 +1−1

+(2x+ y)(x+2y)

4− 8

3(x+ y)

Bài 19. (Quảng Bình) Cho a,b,c là các số thực dương thỏa mãn ab+bc+ ca = 3abc. Chứng

minh rằng1√

a3 +b+

1√b3 + c

+1√

c3 +a≤ 3√

2

Bài 20. (Quảng Nam) Cho ba số thực a,b,c sao cho 0≤ a,b,c≤ 1. Chứng minh

a+b+ c+3abc≥ 2(ab+bc+ ca)

Bài 21. (Thái Bình) Cho các số thực x,y,z≥ 1 và thỏa mãn 3x2 +4y2 +5z2 = 52. Tìm giá trị

nhỏ nhất của biểu thứcF = x+ y+ z

Bài 22. (Thái Nguyên) Tìm giá trị nhỏ nhất của biểu thức

P =

√x+6

√x−9+

√x−6

√x−9

Bài 23. (Thừa Thiên - Huế) Cho x,y > 0 và x+ y≥ 3. Tìm giá trị nhỏ nhất của

M = 6x2 +4y2 +10xy+4xy+

3yx+2016

Bài 24. (Vĩnh Phúc) Cho a,b,c là các số thực dương thỏa mãn a+b+c = 3. Chứng minh rằng

4(a2 +b2 + c2)− (a3 +b3 + c3)≥ 9

Page 14: 1 ¾ F - · PDF fileđang chuẩn bị cho kì thi tuyển sinh vào các lớp 10 ... phẩm “Tuyển tập các bài toán trong đề thi ... đam mê với các môn học

THCM

N

14 Chương 1. CÁC BÀI TOÁN

1.2 Đại sốBài 1. (Trường THPT chuyên Khoa học Tự nhiên, ĐH KHTN, ĐHQG HN)

1. Giải hệ phương trình:{

x2 +4y2 = 54x2y+8xy2 +5x+10y = 1

2. Giải phương trình:√

5x2 +6x+5 = 64x3+4x5x2+6x+6 .

Bài 2. (Trường Phổ thông Năng khiếu - ĐHQG Tp. HCM)

1. Giải hệ{

(x−2y)(x+my) = m2−2m−3(y−2x)(y+mx) = m2−2m−3

khi m = −3 và tìm m để hệ có ít nhất một

nghiệm (x0,y0) thỏa x0 > 0;y0 > 0.2. Tìm a ≥ 1 để phương trình ax2 +(1− 2a)x+ 1− a = 0 có hai nghiệm phân biệt x1,x2

thỏa x22−ax1 = a2−a−1.

Bài 3. (THPT chuyên Đại học Sư phạm Hà Nội Chứng minh biểu thức sau nhận giá trị nguyêndương với mọi giá trị nguyên dương của n:

P =

(√n2 +(n+1)2 +

√(n−1)2 +n2

)√4n2 +2−2

√4n4 +1

Bài 4. (Bắc Ninh)

1. (a) Phân tích đa thức x4 +5x3 +5x2−5x−6 thành nhân tử.

(b) Rút gọn Q =

√x−√

4(x−1)+√

x+√

4(x−1)√x2−4(x−1)

(1− 1

x−1

)với x > 1 và x 6= 2.

2. (a) Giải phương trình: 2(2x−1)−3√

5x−6 =√

3x−8.(b) Cho bốn số thực a, b, c, d khác 0 thỏa mãn các điều kiện sau: a, b là hai nghiệm của

phương trình x2−10cx−11d = 0; c, d là hai nghiệm của phương trình x2−10ax−11b = 0. Tính giá trị của S = a+b+ c+d.

Bài 5. (Đà Nẵng)

1. Cho biểu thức P =a

a+1+

√1+a2 +

a2

(a+1)2 với a 6=−1. Rút gọn biểu thức P và tính

giá trị của P khi a = 2016.2. Giải các phương trình:

(a) (17−6x)√

3x−5+(6x−7)√

7−3x = 2+8√

36x−9x2−35.(b)√

x2−3x+2 =√

10x−20−√

x−3.

Bài 6. (Hà Nội)1. (a) Giải phương trình x4−2x3 + x−

√2(x2− x) = 0.

(b) Giải hệ phương trình{

x2 +2y−4x = 04x2−4xy2 + y4−2y+4 = 0

.

2. Cho các số thực a,b,c đôi một khác nhau thỏa mãn a3 +b3 + c3 = 3abc và abc 6= 0. Tính

P =ab2

a2 +b2− c2 +bc2

b2 + c2−a2 +ca2

c2 +a2−b2 .

Bài 7. (Hải Phòng)

Page 15: 1 ¾ F - · PDF fileđang chuẩn bị cho kì thi tuyển sinh vào các lớp 10 ... phẩm “Tuyển tập các bài toán trong đề thi ... đam mê với các môn học

THCM

N

1.2 Đại số 15

1. (a) Cho biểu thức

P =

√a3−√

b3

a−b− a√

a+√

b− b√

b−√

a

với a,b > 0 và a 6= b. Thu gọn rồi tính giá trị của P biết (a−1)(b−1)+2√

ab = 1.(b) Cho phương trình x2−x+b = 0 có các nghiệm x1,x2 và phương trình x2−97x+a =

0 có các nghiệm là x41;x4

2. Tìm giá trị của a.2. (a) Giải phương trình:

(9x2−18x+5

)(3x2−4x

)−7 = 0.

(b) Giải hệ phương trình{ √

2x+3y+√

2x−3y = 3√

2y2√

2x+3y−√

2x−3y = 6

Bài 8. (Tp. HCM)1. Cho hai số thức a,b sao cho |a| 6= |b| và ab 6= 0 thỏa mãn điều kiện a−b

a2+ab +a+b

a2−ab =3a−ba2−b2 .

Tìm giá trị của biểu thức P = a3+2a2b+3b3

2a3+ab2+b3 .2. (a) Giải phương trình x2−6x+4+2

√2x−1 = 0.

(b) Giải hệ phương trình{

x3− y3 = 9(x+ y)x2− y2 = 3

.

Bài 9. (Hưng Yên)

1. (a) Đặt a =√

2;b = 3√

2. Chứng minh rằng:1

a−b− 1

b= a+b+

ab+

ba+1.

(b) Cho x = 3√√

28+1− 3√√

28−1+2. Tính giá trị của P = x3−6x2 +21x+2016.2. (a) Giải hệ phương trình{

x2y2−2x+ y2 = 02x2−4x+3 =−y3

(b) Giải phương trình(√

2x+5−√

2x+2)(

1+√

4x2 +14x+10)= 3.

Bài 10. (Khánh Hoà)

1. (a) Rút gọn biểu thức P =

√1− 1

22

√1− 1

32 ...

√1− 1

20162 .

(b) Cho a là nghiệm của phương trình x2−3x+1 = 0. Không tính giá trị của a, hãy tính

giá trị của biểu thức Q =a2

a4 +a2 +1.

2. (a) Giải phương trình(

x−1x+2

)2

− 15x2−4

+4(

x+1x−2

)2

= 5.

(b) Giải hệ phương trình(x2− xy

)(xy− y2)= 25√

x2− xy+√

xy− y2 = 3(x− y)

Bài 11. (Long An)1. Cho biểu thức P = 1√

x+1 +10

2√

x+1 −5

2x+3√

x+1 với điều kiện x≥ 0.(a) Rút gọn biểu thức P.(b) Tìm tất cả các số tự nhiên x để P là số nguyên tố.

Page 16: 1 ¾ F - · PDF fileđang chuẩn bị cho kì thi tuyển sinh vào các lớp 10 ... phẩm “Tuyển tập các bài toán trong đề thi ... đam mê với các môn học

THCM

N

16 Chương 1. CÁC BÀI TOÁN

2. Cho phương trình x2−2(m−1)x−2m+5 = 0 (m là tham số). TÌm m để phương trìnhcó hai nghiệm x1,x2 sao cho x1 + x2 +2x1x2 = 26.

3. Giải phương trình 2(x2 +2

)= 5√

x3 +1.

Bài 12. (Nam Định)

1. (a) Đơn giản biểu thức√

x+2+2√

x+1−√

x+2−2√

x+1 với x > 0.(b) Cho a,b,c là các số thực thỏa mãn điều kiện a+b+ c = 6; 1

a+b +1

b+c +1

c+a = 4760 .

Tính giá trị của biểu thức ab+c +

ba+c +

ca+b .

2. (a) Giải phương trình√

2x2 +3x+1+√

1−3x = 2√

x2 +1.

(b) Giải hệ phương trình{

x2 +3y2−3x−1 = 0x2− y2− x−4y+5 = 0

.

Bài 13. (Phú Thọ)1. Cho các số a,b thoả mãn 2a2 + 11ab− 3b2 = 0,b 6= 2a,b 6= −2a. Tính giá trị của biểu

thức T = a−2b2a−b +

2a−3b2a+b .

2. (a) Giải phương trình√

2x+1−√

x−3 = 2.

(b) Giải hệ phương trình{

2x3 + x2y+2x2 + xy+6 = 0x2 +3x+ y = 0

Bài 14. (Vĩnh Phúc)1. Cho phương trình x4 +3x3−mx2 +9x+9 = 0 (m là tham số).

(a) Giải phương trình khi m =−2.(b) Tìm tất cả các giá trị của m để phương trình đã cho có ít nhất một nghiệm dương.

2. Giải phương trình 3x2−4x√

4x−3+4x−3 = 0.

1.3 Hình học phẳngBài 1. (THPT chuyên KHTN, ĐH KHTN, ĐHQG HN) Cho hình vuông ABCD nội tiếp đườngtròn tâm (O),P là điểm thuộc cung nhỏ AD của đường tròn (O) và P khác A,D. Các đường thẳngPB,PC lần lượt cắt đường thẳng AD tại M,N. Đường trung trực của AM cắt các đường thẳngAC,PB lần lượt tại E,K. Đường trung trực của DN cắt các đường thẳng BD,PC lần lượt tại F,L.

1. Chứng minh ba điểm K,O,L thẳng hàng.2. Chứng minh đường thẳng PO đi qua trung điểm EF .3. Giả sử đường thẳng EK cắt đường thẳng BD tại S, các đường thẳng FL và AC cắt nhau tại

T , đường thẳng ST cắt các đường thẳng PC,PB lần lượt tại U,V . Chứng minh rằng bốnđiểm K,L,U,V cùng thuộc một đường tròn.

Bài 2. (Trường Phổ thông Năng khiếu, ĐHQG Tp. HCM)4ABC nhọn có ABC > 45o. Dựngcác hình vuông ABMN,ACPQ (M và C khác phía đối với AB,B và Q khác phía đối với AC). AQcắt BM tại E,NA cắt CP tại F .

1. Chứng minh4ABE ∼4ACF và tứ giác EFQN nội tiếp.2. Chứng minh trung điểm I của EF là tâm đường tròn ngoại tiếp4ABC.3. MN cắt PQ tại D. Đường tròn ngoại tiếp các tam giác DMQ và DNP cắt nhau tại K khác

D. Tiếp tuyến tại B và C của đường tròn ngoại tiếp 4ABC cắt nhau tại J. Chứng minhD,A,K,J thẳng hàng.

Page 17: 1 ¾ F - · PDF fileđang chuẩn bị cho kì thi tuyển sinh vào các lớp 10 ... phẩm “Tuyển tập các bài toán trong đề thi ... đam mê với các môn học

THCM

N

1.3 Hình học phẳng 17

Bài 3. (THPT chuyên Đại học Sư phạm Hà Nội) Cho tam giác ABC nhọn, AB < AC. Kẻ đườngcao AH. Đường tròn (O) đường kính AH cắt cạnh AB,AC tương ứng tại D,E. Đường thẳng DEcắt đường thẳng BC tại S.

1. Chứng minh rằng BDEC là tứ giác nội tiếp.2. Chứng minh rằng SB.SC = SH2.3. Đường thẳng SO cắt AB,AC tương ứng tại M,N, đường thẳng DE cắt HM,HN tương ứng

tại P,Q. Chứng minh rằng BP,CQ và AH đồng quy.

Bài 4. (An Giang) Từ một điểm M nằm ngoài (O) vẽ hai tiếp tuyến MA,MB đến đường tròn(A,B là hai tiếp điểm). Qua A vẽ đường thẳng song song với MB cắt đường tròn tại C; đoạnthẳng MC cắt đường tròn tại D. Hai đường thẳng AD và MB. Hai đường thẳng AD,MB cắt nhautại E. Chứng minh rằng

1. Tứ giác MAOB nội tiếp.2. ME2 = ED.EA.3. E là trung điểm đoạn MB.

Bài 5. (Bà Rịa Vũng Tàu) Cho hai đường tron (O;R),(O′;R′) cắt nhau tại A và B (OO′>R>R′).Trên nửa mặt phẳng bờ OO′ có chứa điểm A, kẻ tiếp tuyến chung của MN của hai đường tròntrên (với M thuộc (O) và N thuộc (O′). Biết MB cắt (O′) tại điểm E nằm trong đường tròn (O)và đường thẳng AB cắt MN tại I.

1. Chứng minh MAN + MBN = 1800 và I là trung điểm MN.2. Qua B, kẻ đường thẳng (d) song song với MN, (d) cắt (O) tại C và cắt (O′) tại D (với C,D

khác B). Gọi P,Q lần lượt là trung điềm của CD và EM. Chứng minh ∆AME ∼ ∆ACD vàcác điểm A,B,P,Q cùng thuộc 1 đường tròn.

3. Chứng minh ∆BIP cân.

Bài 6. (Bà Rịa Vũng Tàu) Cho ∆ABC nhọn và H là trực tâm. Chứng minh

HABC

+HBCA

+HCAB≥√

3

Bài 7. (Bắc Ninh) Trên đường tròn (C) tâm O bán kính R vẽ dây cung AB < 2R. Từ A và B vẽhai tiếp tuyến Ax,By với (C). Lấy điểm M bất kì thuộc cung nhỏ AB (M khác A,B). Gọi H,K, Ilần lượt là chân các đường vuông góc hạ từ M xuống AB,Ax,By.

1. Chứng minh rằng MH2 = MK.MI.2. Gọi E là giao điểm của AM và KH,F là giao điểm của BM và HI. Chứng minh EF là tiếp

tuyến chung của hai đường tròn ngoại tiếp các tam giác MEK,MFI.3. Gọi D là giao điểm thứ hai của hai đường tròn ngoại tiếp các tam giác MEK,MFI. Chứng

minh rằng khi M di chuyển trên cung nhỏ AB thì đường thẳng DM luôn đi qua một điểmcố định.

Bài 8. (Bình Định)1. Cho tứ giác ABCD nội tiếp đường tròn tâm O sao cho hai đường thẳng AD và BC cắt

nhau tại T . Đường thẳng (d) ⊥ OT tại T cắt AB và CD lần lượt tại M và N. Chứng minhT M = T N.

2. Cho xOy nhọn và M là một điểm cố định thuộc miền trong xOy. Đường thẳng d qua M cắtcác cạnh Ox,Oy lần lượt tại A,B không trùng với O. Xác định vị trí của A để ∆OAB cóđiện tích nhỏ nhất.

Page 18: 1 ¾ F - · PDF fileđang chuẩn bị cho kì thi tuyển sinh vào các lớp 10 ... phẩm “Tuyển tập các bài toán trong đề thi ... đam mê với các môn học

THCM

N

18 Chương 1. CÁC BÀI TOÁN

Bài 9. (Bình Định)1. Từ một điểm S ở ngoài đường tròn tâm O kẻ các tiếp tuyến SA,SC và cát tuyến SBD (B

nằm giữa S và D). Gọi I là giao điểm của AC và BD. Chứng minh rằng:(a) AB.DC = AD.BC.(b) SB

SD = IBID = AB.CB

AD.CD .2. Cho nửa đường tròn tâm O, đường kính AB = 2R. Điểm M nằm trên nửa đường tròn sao

cho MAB = 60o. Kẻ MH⊥AB tại H, HE⊥AM tại E, HF⊥BM tại F . Các đường thẳngEF và AB cắt nhau tại K. Tính diện tích tam giác MEF và độ dài các đoạn thẳng KA,KBtheo R.

Bài 10. (Cần Thơ)4ABC nội tiếp đường tròn (O),AB < AC. Phân giác trong góc BAC cắt (O)

tại D khác A. Trên tia AB lấy M tuỳ ý sao cho đường tròn ngoại tiếp4ADM cắt AC tại N khácA,C.

1. Chứng minh4BDM =4CDN.2. Khi BN không song song với MC, đường trung trực của đoạn BN cắt đường trung trực

của đoạn NC tại P. Chứng minh A,C,P,M cùng thuộc một đường tròn.3. Xác định vị trí tâm I của đường tròn ngoại tiếp 4ADM để độ dài đoạn thẳng MN nhỏ

nhất.

Bài 11. (Đà Nẵng) Cho tam giác ABC có BAC > 90o,AB < AC và nội tiếp đường tròn tâm O.Trung tuyến AM của tam giác ABC cắt (O) tại điểm thứ hai D. Tiếp tuyến của (O) tại D cắtđường thẳng BC tại S. Trên cung nhỏ DC của (O) lấy điểm E, đường thẳng SE cắt (O) tại điểmthứ hai là F . Gọi P,Q lần lượt là giao điểm của các đường thẳng AE,AF với BC.

1. Chứng minh rằng MODS là tứ giác nội tiếp.2. Chứng minh rằng QB = PC.

Bài 12. (Đồng Nai) Từ điểm M nằm ngoài đường tròn (ω) tâm O, vẽ đến (ω) hai tiếp tuyếnMA,MB và cát tuyến MCD (C nằm giữa M và D).Gọi H là giao điểm MO và AB.

1. Chứng minh MA2 = MC.MD.2. Chứng minh tứ giác CDOH nội tiếp.3. Chứng minh đường thẳng AB và hai tiếp tuyến của (ω) tại C,D đồng quy.4. Đường thẳng CH cắt (ω) tại điểm thứ hai E khác C. Chứng minh AB//DE.

Bài 13. (Đồng Nai)4ABC có bán kính đường tròn nội tiếp r và độ dài các đường cao là x,y,z.

1. Chứng minh1x+

1y+

1z=

1r

.

2. Biết r = 1 và x,y,z là các số nguyên dương. Chứng minh4ABC đều.

Bài 14. (Hà Nội) Cho tam giác nhọn ABC có AB < AC và nội tiếp đường tròn (O). Các đườngcao BB′,CC′ cắt nhau tại điểm H. Gọi M là trung điểm BC. Tia MH cắt đường tròn (O) tại điểmP.

1. Chứng minh ∆BPC′ ∼ ∆CPB′.2. Các đường phân giác của các góc BPC′,CPB′ lần lượt cắt AB,AC tại E,F . Gọi O′ là tâm

đường tròn ngoại tiếp ∆AEF ; K là giao điểm của HM và AO′.(a) Chứng minh tứ giác PEKF nội tiếp.(b) Chứng minh các tiếp tuyến tại E và F của đường tròn (O′) cắt nhau tại 1 điểm nằm

trên đường tròn (O).

Page 19: 1 ¾ F - · PDF fileđang chuẩn bị cho kì thi tuyển sinh vào các lớp 10 ... phẩm “Tuyển tập các bài toán trong đề thi ... đam mê với các môn học

THCM

N

1.3 Hình học phẳng 19

Bài 15. (Hà Tĩnh) Cho tam giác đều ABC nội tiếp đường tròn tâm O. Điểm E thay đổi trêncung nhỏ AB (E khác A và B). Từ B và C lần lượt kẻ các tiếp tuyến với đường tròn (O), các tiếptuyến này cắt đường thẳng AE theo thứ tự tại M và N. Gọi F là giao điểm của BN và CM.

1. Chứng minh rằng MB.CN = BC2.2. Khi điểm E thay đổi trên cung nhỏ AB. Chứng minh rằng đường thẳng EF luôn đi qua

một điểm cố định.

Bài 16. (Hải Dương) Cho đường tròn tâm O đường kính BC, A là điểm di động trên đường tròn(O) (A khác B và C). Kẻ AH⊥BC tại H. M là điểm đối xứng của điểm A qua điểm B.

1. Chứng minh điểm M luôn nằm trên một đường tròn cố định.2. Đường thẳng MH cắt (O) tại E và F (E nằm giữa M và F). Gọi I là trung điểm HC, đường

thẳng AI cắt (O) tại G (G 6= A). Chứng minh AF2 +FG2 +GE2 +EA2 = 2.BC2.3. Kẻ HP⊥AB tại P. Tìm vị trí điểm A sao cho bán kính đường tròn ngoại tiếp tam giác BPC

đạt giá trị lớn nhất.

Bài 17. (Hải Phòng) Cho ∆ABC nhọn nội tiếp đường tròn tâm O có AB < AC. Các đường caoBD,CE cắt nhau tại H (D thuộc AC, E thuộc AB). Gọi M là trung điểm của BC, tia MH cắtđường tròn (O) tại N.

1. Chứng minh rằng năm điểm A,D,H,E,N cùng thuộc 1 đường tròn.2. Lấy điểm P trên đoạn BC sao cho BHP = CHM, Q là hình chiếu vuông góc của A trên

đường thẳng HP. Chứng minh rằng tứ giác DENQ là hình thang cân.3. Chứng minh rằng đường tròn ngoại tiếp tam giác MPQ tiếp xúc với đường tròn (O).

Bài 18. (TP. HCM)1. Cho ∆ABC nhọn có các đường cao AA1,BB1,CC1. Gọi K là hình chiếu của A trên A1B1; L

là hình chiếu của B lên B1C1. Chứng minh rằng: A1K = B1L.2. Cho tứ giác nội tiếp ABCD có AC cắt BD tại E. Tia AD cắt tia BC tại F . Dựng hình bình

hành AEBG.(a) Chứng minh FD.FG = FB.FE(b) Gọi H là điểm đối xứng của E qua AD. Chứng minh 4 điểm F,H,A,G cùng thuộc

một đường tròn.

Bài 19. (Hưng Yên) Cho hai đường tròn (O) và (O′) cắt nhau tại A,B. Tiếp tuyến chung gần Bcủa hai đường tròn lần lượt tiếp xúc (O),(O′) tại C và D. Qua A kẻ đường thẳng song song CDlần lượt cắt (O),(O′) tại M,N. Các đường thẳng CM,DN cắt nhau tại E. Gọi P là giao điểm củaBC với MN, Q là giao điểm của BD và MN. Chứng minh:

1. AE ⊥CD.2.

BDBO

+BCBP

=MNPO

.3. 4EPQ cân.

Bài 20. (Khánh Hoà) Cho hai đường tròn (O) và (O′) cắt nhau tại A,B. Từ điểm E nằm trêntia đối của tia AB kẻ đến (O′) các tiếp tuyến EC,ED (C và D là các tiếp điểm phân biệt). Cácđường thẳng AC,AD theo thứ tự cắt (O) lần lượt tại P,Q khác A. Chứng minh:

1. 4BCP∼4BDQ.2. CA.DQ =CP.DA.3. Ba điểm C,D và trung điểm I của PQ thẳng hàng.

Bài 21. (Lào Cai) Cho ∆ABC (AB < AC) nội tiếp đường tròn (O). Gọi H là chân đường cao kẻtừ A đến BC. Gọi P,Q lần lượt là chân đường cao kẻ từ H đến AB,AC. Hai đường thẳng PQ và

Page 20: 1 ¾ F - · PDF fileđang chuẩn bị cho kì thi tuyển sinh vào các lớp 10 ... phẩm “Tuyển tập các bài toán trong đề thi ... đam mê với các môn học

THCM

N

20 Chương 1. CÁC BÀI TOÁN

BC cắt nhau tại M, đường thẳng MA cắt đường tròn (O) tại K với K 6= A. Gọi I là tâm đườngtròn ngoại tiếp ∆BCP.

1. Chứng minh các tứ giác APHQ,BPQC nội tiếp;2. Chứng minh MP.MQ = MB.MC và MB.MC = MK.MA;3. Chứng minh AKPQ là tứ giác nội tiếp;4. Chứng minh I,H,K thẳng hàng.

Bài 22. (Long An) Cho ∆ABC nhọn có đường cao BE và nội tiếp (O). Tiếp tuyến của (O) tạiB,C của (O) cắt nhau tại S,BC và OS cắt nhau tại M. Chứng minh:

1. AB.BM = AE.BS2. AME = ASB

Bài 23. (Long An) Cho tứ giác ABCD có BAD = 60o; BCD = 90o. Đường phân giác trong củaBAD cắt BD tại E. Đường phân giác trong của BCD cắt BD tại F . Chứng minh:

√3

AE+

√2

CF=

1AB

+1

BC+

1CD

+1

DA

Bài 24. (Nam Định) Cho ∆ABC nhọn, nội tiếp đường tròn (O). Các đường cao AK,BM,CNcủa ∆ABC cắt nhau tại H.

1. Chứng minh: NKH = MKH2. Đường thẳng MN cắt đường tròn (O) tại hai điểm I,J. Chứng minh AO đi qua trung điểm

của IJ.3. Gọi P là trung điểm BC, diện tích tứ giác AMHN là S. Chứng minh 2.OP2 > S.

Bài 25. (Ninh Bình) Cho đường tròn (O), bán kính R, dây BC cố định khác đường kính. A làmột điểm di động trên cung lớn BC sao cho4ABC nhọn. Các đường cao BE,CF của4ABC cắtnhau tại H.

1. Chứng minh tứ giác BECF nội tiếp và AO⊥ EF .2. Tia EF cắt (O) tại I, tia AO cắt (O) tại G. Gọi M là trung điểm BC,D là giao điểm hai

đường thẳng AH và BC. Chứng minh AI2 = 2AD.OM.3. Trong trường hợp4ABC cân tại A, goi x là khoảng cách từ (O) đến BC. Tìm x để chu vi4ABC lớn nhất.

Bài 26. (Phú Thọ) Cho đường tròn (O;R) và dây cung BC cố định. Gọi A là điểm di động trêncung lớn BC sao cho ∆ABC nhọn. Bên ngoài ∆ABC dựng các hình vuông ABDE,ACFG và hìnhbình hành AEKG.

1. Chứng minh rằng AK = BC và AK⊥BC.2. DC cắt BF tại M. Chứng minh rằng A,K,M thẳng hàng.3. Chứng minh rằng khi A thay đổi trên cung lớn BC của (O;R) thì K luôn thuộc một đường

tròn cố định.

Bài 27. (Quảng Bình)4ABC nhọn (AB < AC) nội tiếp đường tròn (O). Phân giác của góc BACcắt BC tại D, cắt (O) tại E. Gọi M là giao điểm của AB,CE. Tiếp tuyến tại C của (O) cắt AD tạiN, tiếp tuyến tại E của (O) cắt CN tại F .

1. Chứng minh tứ giác MACN nội tiếp.2. Gọi K là điểm trên cạnh AC sao cho AB = AK. Chứng minh AO⊥ DK.

Page 21: 1 ¾ F - · PDF fileđang chuẩn bị cho kì thi tuyển sinh vào các lớp 10 ... phẩm “Tuyển tập các bài toán trong đề thi ... đam mê với các môn học

THCM

N

1.3 Hình học phẳng 21

3. Chứng minh1

CF=

1CN

+1

CD.

Bài 28. (Quảng Nam) Cho hình bình hành ABCD có góc A tù và AB = AC, gọi H là hình chiếucủa C lên AB. Trên cạnh AB lấy E sao cho H là trung điểm BE. Gọi F là điểm đối xứng với Dqua E,G là điểm đối xứng với A qua B. Chứng minh:

1. EC là tia phân giác của DEB.2. 4CFG cân.

Bài 29. (Quảng Nam) Cho đường tròn (O) đường kính AB, dây CD vuông góc với AB tại Hnằm giữa O và A. Lấy điểm E bất kì trên cung nhỏ BD, gọi M là hình chiếu của B lên CE.

1. Chứng minh rằng HM//AE.2. Đường tròn ngoại tiếp4DEM đi qua trung điểm N của dây AF .

Bài 30. (Tây Ninh) Cho ∆ABC vuông tại A có đường cao AH (H thuộc cạnh BC). Cho BH = 2và CH = m. Xác định m để đường thẳng BC tiếp xúc với đường tròn tâm A bán kính R = 4. Khiđó tính độ dài các đoạn AB và AC.

Bài 31. (Tây Ninh) Cho ∆ABC cân tại A và nội tiếp (O) tâm O. Gọi M là một điểm bất kì trêncung nhỏ AC của (O) (M khác A và C). Trên tia BM lấy điểm E sao cho ME = MC (E ở ngoàiđoạn BM). Chứng minh rằng đường tròn tâm A bán kính AE luôn đi qua B và C.

Bài 32. (Tây Ninh) Từ một điểm M nằm ngoài (O) kẻ hai tiếp tuyến MA,MB với (O) (A,B làcác tiếp điểm). Gọi C là giao điểm của MO và AB, lấy D thuộc đoạn AC (D khác A,C). Đườngthẳng MD cắt (O) tại E,F (ME < MF). Chứng minh rằng:

1. MA2 = ME.MF .2. E,C,O,F cùng thuộc 1 đường tròn.

Bài 33. (Thái Bình) Từ một điểm I nằm ngoài đường tròn (O), vẽ các tiếp tuyến IA, IB (A,B làcác tiếp điểm) và cát tuyến ICD không qua tâm O của (O) (C nằm giữa I và D).

1. Chứng minh AC.BD = AD.BC.2. Gọi K là giao điểm của CD và AB,E là trung điểm OI. Chứng minh KA.KB=OE2−EK2.3. Gọi H là trung điểm AB. Chứng minh ADH = IDB.

Bài 34. (Thái Nguyên) Cho đường tròn tâm O và dây cung AB. Từ một điểm M bất kì trênđường tròn (M 6= A,B), kẻ MH⊥AB tại H. Gọi E,F lần lượt là hình chiếu vuông góc với Htrên MA,MB. Qua M kẻ đường thẳng vuông góc với EF , cắt dây cung AB tại D. Chứng minhMA2

MB2 =AHBH .AD

BH .

Bài 35. (Thái Nguyên) Cho ∆ABC vuông tại A, đường cao AH. Gọi (O) là đường tròn ngoạitiếp ∆AHC. Trên cung nhỏ AH của (O) lấy điểm M bất kì khác A và H. Tren tiếp tuyến tại Mcủa (O) lấy hai điểm D,E sao cho BD = BE = BA. Đường thẳng BM cắt (O) tại điểm thứ haiN. Chứng minh rằng:

1. Tứ giác BDNE nội tiếp.2. Đường tròn ngoại tiếp tứ giác BDNE và đường tròn (O) tiếp xúc nhau.

Bài 36. (Thanh Hóa) Cho hình bình hành ABCD với BAD < 90o. Tia phân giác góc BCD cắtđường tròn ngoại tiếp ∆BCD tại O (O khác C). Kẻ đường thẳng (d) đi qua A và vuông góc vớiCO. Đường thẳng (d) cắt các đường thẳng CB,CD lần lượt tại M,N.

Page 22: 1 ¾ F - · PDF fileđang chuẩn bị cho kì thi tuyển sinh vào các lớp 10 ... phẩm “Tuyển tập các bài toán trong đề thi ... đam mê với các môn học

THCM

N

22 Chương 1. CÁC BÀI TOÁN

1. Chứng minh rằng: OBM = ODC.2. Chứng minh ∆OBM = ∆ODC và O là tâm đường tròn ngoại tiếp ∆CMN.3. Gọi K là giao điểm của OC và BD; I là tâm đường tròn ngoại tiếp ∆BCD. Chứng minh

rằng: NDMB = IB2−IK2

KD2 .

Bài 37. (Thừa Thiên - Huế) Cho hai đường tròn (O1) và (O2) có bán kính khác nhau, cắt nhautại A,B sao cho O1,O2 thuộc hai nửa mặt phẳng có bờ là đường thẳng AB. Đường tròn (O) ngoạitiếp4BO1O2 cắt (O1),(O2) lần lượt tại K,L khác A và B. Đường thẳng AO cắt (O1),(O2) lầnlượt tại M,N khác A. Hai đường thẳng MK,NL cắt nhau tại P sao cho P,B thuộc hai nửa mặtphẳng có bờ là đường thẳng KL. Chứng minh:

1. Tứ giác BKPL nội tiếp.2. Điểm A cách đều hai đường thẳng BK,BL.3. Điểm P thuộc đường thẳng AB khi và chỉ khi4PKL cân.

Bài 38. (Vĩnh Phúc)4ABC nhọn (AB < AC) nội tiếp đường tròn (O),M là trung điểm BC. AMcắt (O) tại D khác A. Đường tròn ngoại tiếp4MDC cắt đường thẳng AC tại E khác C. Đườngtròn ngoại tiếp4MDB cắt đường thẳng AB tại F khác B.

1. Chứng minh4BDF ∼4CDE và E,M,F thẳng hàng.2. Chứng minh OA⊥ EF .3. Phân giác góc BAC cắt EF tại N. Phân giác góc CEN và BFN lần lượt cắt CN,BN tại

P,Q. Chứng minh rằng PQ//BC.

1.4 Số họcBài 1. (THPT chuyên KHTN, ĐH KHTN, ĐHQG HN)

1. Với x,y là các số nguyên dương thỏa mãn x2−12 = y2−1

3 , chứng minh rằng x2− y2 ... 402. Tìm tất cả các cặp số nguyên (x,y) thỏa mãn x4 +2x2 = y3.

Bài 2. (Trường Phổ thông Năng khiếu, ĐHQG Tp. HCM) Cho x,y là hai số nguyên dương thỏa

mãn x2 + y2 +10... xy.

1. Chứng minh rằng x,y là hai số lẻ và nguyên tố cùng nhau.2. Chứng minh k = x2+y2+10

xy chia hết cho 4 và k ≥ 12.

Bài 3. (THPT chuyên Đại học Sư phạm Hà Nội) Tìm các số nguyên dương x,y thỏa mãnx3− y3 = 95(x2 + y2) .

Bài 4. (THPT chuyên Đại học Sư phạm Hà Nội) Cho S là tập hợp các số nguyên dương n códạng n = x2 +3y2 trong đó x,y nguyên dương. Chứng minh rằng:

1. Nếu a,b ∈ S thì ab ∈ S.

2. Nếu N ∈ S và N chẵn thì N... 4 và N

4 ∈ S.

Bài 5. (Bà Rịa - Vũng Tàu) Tìm tất cả các cặp số nguyên tố (p;q) thỏa

p2−5q2 = 4

Bài 6. (Bắc Ninh) Tìm bộ ba số nguyên tố (a;b;c) thỏa a < b < c và

Page 23: 1 ¾ F - · PDF fileđang chuẩn bị cho kì thi tuyển sinh vào các lớp 10 ... phẩm “Tuyển tập các bài toán trong đề thi ... đam mê với các môn học

THCM

N

1.4 Số học 23bc−1

...a

ac−1...b

bc−1...a

Bài 7. (Bình Định) Tìm nghiệm nguyên của phương trình

x2− (y+5)x+5y−2 = 0.

Bài 8. (Bình Thuận): Cho 2 số nguyên dương lẻ m,n nguyên tố cùng nhau và thỏa

{m2 +2

... n

n2 +2... m

.

Chứng minh rằng: m2 +n2 +2... 4mn.

Bài 9. (Cần Thơ) Tìm tất cả các cặp số nguyên (x;y) thỏa mãn

2x2−2x−6y2 +3y+ xy+7 = 0

Bài 10. (Đà Nẵng) Tìm tất cả các số nguyên dương x và số nguyen tố p sao cho

x5 + x4 +1 = p2.

Bài 11. (Hà Nội) Tìm tất cả các cặp số tự nhiên (x;y) thỏa

2x.x2 = 9y2 +6y+16.

Bài 12. (Hà Tĩnh) Tìm các bộ số nguyên dương (x;y;z) thỏa{x+ y− z = 0

x3 + y3− z2 = 0

Bài 13. (Hải Dương)

1. Tìm dạng tổng quát của số nguyên dương n biết M = n.4n +3n chia hết cho 72. Tìm số các cặp số (x;y) nguyên dương thỏa mãn

(x2 +4y2 +28)2−17(x4 + y4) = 238y2 +833.

Bài 14. (Hải Phòng) Tìm tất cả các số nguyên m,n với m ≥ n ≥ 0 sao cho (m+2n)3 là ướccủa 9(m2 +mn+n2 +16.

Page 24: 1 ¾ F - · PDF fileđang chuẩn bị cho kì thi tuyển sinh vào các lớp 10 ... phẩm “Tuyển tập các bài toán trong đề thi ... đam mê với các môn học

THCM

N

24 Chương 1. CÁC BÀI TOÁN

Bài 15. (Tp. HCM) Cho m,n là các số nguyên dương sao cho 5m+ n chia hết cho 5n+m.

Chứng minh rằng m...n.

Bài 16. (Hưng Yên) Tìm tất cả các nghiệm nguyên dương (x;y;z) của phương trình

xyz+ xy+ yz+ zx+ x+ y+ z = 2015 thỏa x≥ y≥ z≥ 8.

Bài 17. (Khánh Hoà) Tìm tất cả số nguyên tố p sao cho 8p2 +1 và 8p2−1 là số nguyên tố.

Bài 18. (Nam Định)1. Chứng minh rằng tồn tại vô hạn bộ ba số nguyên (x;y;z) thỏa xyz 6= 0 và x5+8y3+7z2 = 0.2. Tìm tất cả các số nguyên không âm (a;b;c) thỏa và{

(a−b)2 +(b− c)2 +(c−a)2 = 6abc

a3 +b3 + c3 +1...a+b+ c+1

Bài 19. (Ninh Bình) Tìm tất cả các căp số nguyên (x;y) thỏa

1+ x+ x2 + x3 + x4 = y2.

Bài 20. (Phú Thọ) Tìm các số nguyên (x;y) thỏa mãn

2x3 +2x2y+ x2 +2xy = x+10.

Bài 21. (Quảng Nam) Hãy tìm bộ ba số nguyên dương a≤ b≤ c thỏa mãn đẳng thức sau:

abc = 2(a+b+ c).

Bài 22. (Thái Nguyên)

1. Tìm tất cả các nghiệm nguyên của phương trình:

2xy+ y+ x = 83

2. Tìm tất cả các số có 5 chữ số abcde sao cho3√

abcde = ab.3. Cho 3 số nguyên dương a,b,c nguyên dương, nguyên tố cùng nhau thỏa điều kiện

1a+

1b=

1c

. Chứng minh a+b là số chính phương.

Bài 23. (Thái Bình) Tìm các số nguyên x,y thỏa

9x2 +3y2 +6xy−6x+2y−35 = 0

Bài 24. (Thừa Thiên - Huế) Tìm các bộ số nguyên dương (x;y;z) thỏa mãn{ 1x +

!y +

1z = 1√

x− y+ z =√

x−√y+√

z

Bài 25. (Vĩnh Phúc) Tìm tất cả nghiệm nguyên x,y của phương trình

x2 = y2(x+ y4 +2y2)

Page 25: 1 ¾ F - · PDF fileđang chuẩn bị cho kì thi tuyển sinh vào các lớp 10 ... phẩm “Tuyển tập các bài toán trong đề thi ... đam mê với các môn học

THCM

N

1.5 Tổ hợp 25

1.5 Tổ hợpBài 1. (Trường THPT chuyên Khoa học Tự nhiên, ĐH KHTN, ĐHQG HN) Chứng minh rằngvới mọi số nguyên n≥ 3 luôn tồn tại một cách sắp xếp bộ n số 1,2, ...,n thành x1,x2, ...xn saocho x j 6= xi+xk

2 với mọi bộ số chỉ số (i, j,k) mà 1≤ i < j < k ≤ n.

Bài 2. (Trường THPT Chuyên ĐH Sư phạm HN) Giả sử mỗi điểm của mặt phẳng được tô bởimột trong ba màu : xanh,đỏ,vàng . Chứng minh rằng tồn tại ba điểm cùng màu là ba đỉnh củamột tam giác cân.

Bài 3. (Trường Phổ thông Năng khiếu, ĐHQG Tp. HCM) Với mỗi số nguyên dương m > 1, kíhiệu S(m) là ước nguyên dương lớn nhất của m và khác m. Cho số tự nhiên n > 1, đặt n0 = nvà lần lượt tính các số n1 = n0−S(n0);n2 = n1−S(n1); . . . ;ni+1 = ni−S(ni); . . . . Chứng minhrằng tồn tại số nguyên dương k để nk = 1 và tính k khi n = 216.1417.

Bài 4. (Đà Nẵng) Người ta dùng một số quân cờ tetromino gồm 4 ô vuông kích thước 1 x 1,hình chữ L, có thể xoay hoặc lật ngược như hình 1 để ghép phủ kín một bàn cờ hình vuông kíchthước n x n (n là số nguyên dương) gồm n2 ô vuông kích thước 1 x 1 theo quy tắc:

i. Với mỗi quân cờ sau khi ghép vào bàn cờ, các ô vuông của nó phải trùng với các ô vuôngcủa bàn cờ.

ii. Không có hai quân cờ nào mà sau khi ghép vào bàn cờ chúng kê lên nhau.

1. Khi n = 4, hãy chỉ ra một cách ghép phủ kín bàn cờ.2. Tìm tất cả giá trị của n để có thể ghép phủ kín bàn cờ.

Bài 5. (Hà Nội) Cho 2017 số hữu tỷ dương được viếttrên một đường tròn. Chứng minh tồn tạihai số được viết cạnh nhau trên đường tròn sao cho khi bỏ 2 số đó thì 2015 số còn lại không thểchia thành hai nhòm mà tổng các số ở mỗi nhóm bằng nhau.

Bài 6. (Hà Tĩnh) Trên một đường tròn, lấy 1000 điểm phân biệt, các điểm được tô màu xanh vàđỏ xen kẽ nhau. Mỗi điểm được gán với một giá trị là một số thực khác 0. Giá trí của mỗi điểmxanh bằng tổng giá trị của hai điểm đỏ kề với nó, giá trị của mỗi điểm đỏ bằng tích giá trị củahai điểm xanh kề với nó. Tính tổng giá trị của 1000 điểm trên.

Bài 7. (Hải Phòng) Trong dãy số thực a1;a2;a3; . . . ;a2016 ta đánh dấu tất cả các số dương vàsố mà có ít nhất một tổng của nó với một số các số liên tiếp liền ngay sau nó là một số dương(ví dụ trong dãy −6;5;−3;3;1;−1;−2;−3; . . . ;2011 ta đánh dấu các số a2 = 5;a3 =−3;a4 =

Page 26: 1 ¾ F - · PDF fileđang chuẩn bị cho kì thi tuyển sinh vào các lớp 10 ... phẩm “Tuyển tập các bài toán trong đề thi ... đam mê với các môn học

THCM

N

26 Chương 1. CÁC BÀI TOÁN

3;a5 = 1). Chứng minh rằng nếu trong dãy đã cho có ít nhất một số dương thỉ tổng tất cả các sốđược đánh dấu là một số dương.

Bài 8. (Tp. HCM) Nam cắt 1 tờ giấy ra làm 4 miếng hoặc 8 miếng rồi lấy một số miếng nhỏ đócắt ra làm 4 miếng hoặc 8 miếng nhỏ hơn, và Nam cứ tiếp tục việc cắt như thế nhiều lần. Hỏi vớiviệc cắt như vậy, Nam có thể cắt được 2016 miếng lớn, nhỏ được hay không? Vì sao?

Bài 9. (Khánh Hòa) Trong mặt phẳng cho 10 điểm đôi một phân biệt sao cho bất kỳ 4 điểmnào trong 10 điểm đã cho cũng có 3 điểm thẳng hàng. Chứng minh rằng ta có thể bỏ đi mộtđiểm trong 10 điểm đã cho để có 9 điểm còn lại thuộc một đường thẳng.

Bài 10. (Long An) Số A được tạo thành bởi các chữ số viết liền nhau bao gồm các số nguyên

dương từ 1 đến 60 theo thứ tự từ nhỏ đến lớn: A = 12345678910. . .585960. Ta xóa 100 chữa sốcủa A sao cho số tạo thành bởi các chữ số còn lại là số nhỏ nhất (không thay đổi trật tự của cácchữ số ban đầu). Hãy tìm số nhỏ nhất được tạo thành đó.Bài 11. (Nam Định) Trên bảng ban đầu ghi số 2 và số 4. Ta thực hiện cách viết thêm các số lên

bảng như sau: trên bảng đã có 2 số, giả sử là a,b (a khác b), ta viết thêm lên bảng số có giá trị làa+b+ab. Hỏi với cách thực hiện như vậy, trên bảng có thể xuất hiện số 2016 được hay không ?Giải thích.Bài 12. (Phú Thọ) Cho 19 diểm phân biệt nằm trong 1 tam giác đều có cạnh bằng 3, trong đó

không có 3 điểm nào thẳng hàng. Chứng minh rằng luôn tìm được 1 tam giác có 3 đỉnh là 3

trong 19 điểm đã cho mà có diện tích không lớn hơn

√3

4.

Bài 13. (Quảng Bình) Trong 100 số tự nhiên từ 1 đến 100 hãy chọn n số (n≥ 2) sao cho 2 sốphân biệt bất kì được chọn có tổng chia hết cho 6. Hỏi có thể chọn n số thỏa mãn điều kiện trênvới n lớn nhất là bao nhiêu ?

Bài 14. (Vĩnh Phúc) Tập hợp A ={

1;2;3; ...;3n−1;3n}

với n là số nguyên dương được gọi làtập hợp cân đối nếu có thể chia A thành n tập hợp con A1,A2, ...,An và thỏa mãn hai điều kiệnsau:

i. Mỗi tập hợp Ai(i = 1;2; ...;n) gồm 3 số phân biệt và có một số bằng tổng hai số còn lại.ii. Mỗi tập hợp A1,A2, ...,An đôi một không có phần tử chung.

. Chứng minh rằng:1. Tập A =

{1;2;3; ..;92;93

}không là tập cân đối.

2. Tập A ={

1;2;3; ...;830;831}

là tập cân đối.

Page 27: 1 ¾ F - · PDF fileđang chuẩn bị cho kì thi tuyển sinh vào các lớp 10 ... phẩm “Tuyển tập các bài toán trong đề thi ... đam mê với các môn học

THCM

N2. LỜI GIẢI

2.1 Bất đẳng thức

Bài 1. Biết x≥ y≥ z, x+ y+ z = 0 và x2 + y2 + z2 = 6.1. Tính S = (x− y)2 +(x− y)(y− z)+(y− z)2

2. Tìm giá trị lớn nhất của P = |(x− y)(y− z)(z− x)|

(Trường Phổ thông Năng khiếu, ĐHQG Tp. HCM)

Lời giải

1. Ta có:

S = x2 + y2 + z2− xy− yz− zx = 6− (x+ y+ z)2− (x2 + y2 + z2)

2= 6− 02−6

2= 9

2. Đặt a = x− y,b = y− z. Khi đó ta có a≥ 0,b≥ 0 và a2 +ab+b2 = 9

⇒ (a+b)2−9 = ab≤ (a+b)2

4⇒ a+b≤ 2

√3

Đặt t = a+b. Khi đóP = t(t2−9)

Ta sẽ chứng minh:t(t2−9)≤ 6

√3

⇔ (t−2√

3)(t +√

3)2 ≤ 0

Bất đẳng thức trên luôn đúng⇒MaxP = 6√

3.Dấu −” xảy ra khi x =

√3,y = 0,z =−

√3

Nhận xét. Các bạn có thể luyện tập thêm bằng các bài toán sau:1. Cho a,b,c là các số không âm. Chứng minh rằng

(a+b+ c)3 ≥ 6√

3(a−b)(b− c)(c−a)

Page 28: 1 ¾ F - · PDF fileđang chuẩn bị cho kì thi tuyển sinh vào các lớp 10 ... phẩm “Tuyển tập các bài toán trong đề thi ... đam mê với các môn học

THCM

N

28 Chương 2. LỜI GIẢI

2. (Trần Quốc Anh) Cho các số không âm a,b,c thỏa mãn a+b+c =√

5. Chứng minh rằng

(a2−b2)(b2− c2)(c2−a2)≤√

5

Bài 2. Cho x,y,z là 3 số dương thỏa mãn x2 + y2 + z2 = 3xyz. Chứng minh:

x2

y+2+

y2

z+2+

z2

x+2≥ 1

(Bà Rịa - Vũng Tàu)

Lời giải

Áp dụng AM-GM ta được:x2

y+2+

y+29≥ 2

3x

y2

z+2+

z+29≥ 2

3y

z2

x+2+

x+29≥ 2

3z

Khi đó:x2

y+2+

y2

z+2+

z2

x+2≥ 5

9(x+ y+ z)− 2

3

Áp dụng các bất đẳng thức quen thuộc ta có:

3xyz = x2 + y2 + z2 ≥ xy+ yz+ zx

⇒ 3≥ 1x+

1y+

1z≥ 9

x+ y+ z⇒ x+ y+ z≥ 3

Suy ra:x2

y+2+

y2

z+2+

z2

x+2≥ 5

9(x+ y+ z)− 2

3≥ 5

9.3− 2

3= 1

Do đó ta thu được điều phải chứng minh.

Bài 3. Cho a,b,c > 0. Tìm giá trị nhỏ nhất của

M =3a4 +3b4 + c3 +2

(a+b+ c)3

(Bắc Ninh)

Lời giải

Trong bài có dùng tới bất đẳng thức Holder:

Page 29: 1 ¾ F - · PDF fileđang chuẩn bị cho kì thi tuyển sinh vào các lớp 10 ... phẩm “Tuyển tập các bài toán trong đề thi ... đam mê với các môn học

THCM

N

2.1 Bất đẳng thức 29

Bổ đề 1 (BĐT Holder). Cho a,b,c,x,y,z,m,n, p là các số dương. Khi đó:

(a3 +b3 + c3)(x3 + y3 + z3)(m3 +n3 + p3)≥ (axm+byn+ czp)3

Nhưng trước hết, áp dụng bất đẳng thức AM - GM ta có

3a4 +1 = a4 +a4 +a4 +1≥ 4 4√

(a4)3 = 4a3

Tương tự, ta được

M ≥ 4a3 +4b3 + c3

(a+b+ c)3

Đến đây áp dụng BĐT Holder ta có:

(4a3 +4b3 + c3)(12+

12+1)2 ≥ (a+b+ c)3⇔ 4a3 +4b3 + c3

(a+b+ c)3 ≥14

Suy ra: M ≥ 14

. Dấu bằng xảy ra khi a = b = 1;c = 2.

Nhận xét. Sau đây là một số bài tập tự luyện:1. (VMO 2004) Cho x,y,z là các số thực dương thỏa mãn (x+ y+ z)3 = 32xyz. Tìm giá trị

nhỏ nhất và lớn nhất của biểu thức

P =x4 + y4 + z4

(x+ y+ z)4

2. (United Kingdom 2008) Cho x,y,z thỏa mãn x3 + y3 + z3−3xyz = 1. Tìm giá trị nhỏ nhấtcủa biểu thức:

M = x2 + y2 + z2

Bài 4. Cho x,y,z là 3 số thay đổi thỏa mãn x2 + y2 + z2 = 1. Tìm giá trị lớn nhất của biểuthức:

P = xy+ yz+ zx+12[x2(y− z)2 + y2(z− x)2 + z2(x− y)2]

(Bình Định)

Lời giải

Ta sẽ chứng minh:

P = xy+ yz+ zx+12[x2(y− z)2 + y2(z− x)2 + z2(x− y)2]≤ 1

⇔ (xy+ yz+ zx)(x2 + y2 + z2)+ x2y2 + y2z2 + z2x2− xyz(x+ y+ z)≤ (x2 + y2 + z2)2

⇔ 12[(x2 + y2)(x− y)2 +(y2 + z2)(y− z)2 +(z2 + x2)(z− x)2]≥ 0

Bất đẳng thức trên luôn đúng.

Page 30: 1 ¾ F - · PDF fileđang chuẩn bị cho kì thi tuyển sinh vào các lớp 10 ... phẩm “Tuyển tập các bài toán trong đề thi ... đam mê với các môn học

THCM

N

30 Chương 2. LỜI GIẢI

Bài 5. Cho các số dương x,y,z. Chứng minh rằng

xyx2 + yz+ zx

+yz

y2 + zx+ xy+

zxz2 + xy+ yz

≤ x2 + y2 + z2

xy+ yz+ zx

(Bình Thuận)

Lời giải

Để ý rằng theo bằng đẳng thức Cauchy - Schwarz ta có

xyx2 + yz+ zx

=xy(y2 + yz+ zx)

(x2 + yz+ zx)(y2 + yz+ zx)≤ xy(y2 + yz+ zx)

(xy+ yz+ zx)2

Tương tự:yz

y2 + zx+ xy≤ yz(z2 + zx+ xy)

(xy+ yz+ zx)2

zxz2 + xy+ yz

≤ zx(x2 + xy+ yz)(xy+ yz+ zx)2

Suy ra:

xyx2 + yz+ zx

+yz

y2 + zx+ xy+

zxz2 + xy+ yz

≤ (xy+ yz+ zx)(x2 + y2 + z2)

(xy+ yz+ zx)2 =x2 + y2 + z2

xy+ yz+ zx

Dấu bằng xảy ra khi x = y = z.

Nhận xét. Chúng ta có một số bài toán tương tự:1. Chứng minh rằng với mọi số thực dương a,b,c ta luôn có

aba2 +ab+bc

+bc

b2 +bc+ ca+

cac2 + ca+ab

≤ a2 +b2 + c2

ab+bc+ ca

2. Chứng minh rằng với mọi số thực dương a,b,c ta luôn có

aba2 +bc+b2 +

bcb2 + ca+ c2 +

cac2 +ab+a2 ≤

a2 +b2 + c2

ab+bc+ ca

Bài 6. Cho a,b,c lần lượt là độ dài ba cạnh của một tam giác và thỏa mãn 2ab+3bc+4ca =5abc. Tìm giá trị nhỏ nhất của biểu thức

P =7

a+b− c+

6b+ c−a

+5

c+a−b

(Cần Thơ)

Lời giải

Page 31: 1 ¾ F - · PDF fileđang chuẩn bị cho kì thi tuyển sinh vào các lớp 10 ... phẩm “Tuyển tập các bài toán trong đề thi ... đam mê với các môn học

THCM

N

2.1 Bất đẳng thức 31

Theo giả thiết, áp dụng bất đẳng thức Cauchy Schwarz ta có:

5 =3a+

4b+

2c≥ (3+4+2)2

3a+4b+2c⇒ 3a+4b+2c≥ 81

5

Suy ra

P≥ (7+6+5)2

7(a+b− c)+6(b+ c−a)+5(c+a−b)=

182

2(3a+4b+2c)= 10

Dấu −” xảy ra khi a = b = c =95

.

Bài 7. Cho a,b,c là các số thực không âm thỏa a+b+ c = 3.1. Chứng minh rằng: ab+bc+ ca≤ 32. Chứng minh rằng: a2b+b2c+ c2a≤ 4

(Đồng Nai)

Lời giải

1. Ta có:

ab+bc+ ca≤ (a+b+ c)2

3= 3

2. Không giảm tính tổng quát, giả sử

(b−a)(b− c)≤ 0⇔ b2 +ac≤ ab+bc⇔ b2c+ac2 ≤ abc+bc2

Do a,b,c không âm nên kết hợp AM-GM ta có:

a2b+b2c+ c2a≤a2b+bc2 +2abc = b(a+ c)2 = b(a+ c)(a+ c)≤ 4(a+b+ c)3

27= 4

Nhận xét. Bài toán trên thuộc đề thi chọn học sinh giỏi quốc gia của Canada năm 1999 (CanadaMO 1999). Xin giới thiệu một bài toán được xây dựng từ bài toán trên, xuất hiên trên tạp ChíToán học và Tuổi trẻ: Cho các số thực không âm a,b,c thỏa mãn a+b+ c = 3. Tìm giá trị nhỏnhất của biểu thức

P =a

b4 +16+

bc4 +16

+c

a4 +16

Bài 8. Cho các số thực dương a,b,c thỏa mãn a2 +b2 + c2 = 3. Chứng minh

2a2

a+b2 +2b2

b+ c2 +2c2

c+a2 ≥ a+b+ c

(Hà Nội)

Lời giải

Page 32: 1 ¾ F - · PDF fileđang chuẩn bị cho kì thi tuyển sinh vào các lớp 10 ... phẩm “Tuyển tập các bài toán trong đề thi ... đam mê với các môn học

THCM

N

32 Chương 2. LỜI GIẢI

Áp dụng AM-GM với chú ý rằng

2a2

a+b2 =2a(a+b2)−2ab2

a+b2 = 2a− 2ab2

a+b2 ≥ 2a− 2ab2

2b√

a= 2a−

√b.ab≥ 2a− b+ab

2

Tương tự ta được

2a2

a+b2 +2b2

b+ c2 +2c2

c+a2 ≥32(a+b+ c)− ab+bc+ ca

2

Mặt khác ta có:

(a+b+ c)2(a2 +b2 + c2)≥ 3(ab+bc+ ca)2⇔ a+b+ c≥ ab+bc+ ca

Từ đó suy ra:

2a2

a+b2 +2b2

b+ c2 +2c2

c+a2 ≥32(a+b+ c)− ab+bc+ ca

2≥ a+b+ c

Nhận xét. Bài toán trên được giải quyết bằng kĩ thuật Cauchy ngược dấu. Sau đây là một số bàitoán tương tự.

1. (Trần Quốc Anh) Cho các số dương x,y,z có tích bằng 1. Chứng minh rằng:

x4yx2 +1

+y4z

y2 +1+

z4xz2 +1

≥ 32

2. (Trần Quốc Anh) Cho các số thực không âm a,b,c thỏa mãn a+b+ c = 3. Chứng minhrằng

ab3 +16

+b

c3 +16+

ca3 +16

≥ 16

3. Chứng minh rằng với mọi số thực dương a,b,c ta luôn có

a4

a3 +2b3 +b4

b3 +2c3 +c4

c3 +2d3 +d4

d3 +2a3 ≥a+b+ c+d

3

Bài 9. Cho a,b,c > 0 thỏa mãn a+b+ c = 2016. Tìm giá trị lớn nhất của

aa+√

2016a+bc+

bb+√

2016b+ ca+

cc+√

2016c+ab

(Hà Tĩnh)

Lời giải

Ta có: aa+√

2016a+bc=

a

a+√

a(a+b+ c)+bc=

a

a+√(a+b)(a+ c)

≤ a

a+

√(√ab+

√ac)2

=

√a

√a+√

b+√

c

Page 33: 1 ¾ F - · PDF fileđang chuẩn bị cho kì thi tuyển sinh vào các lớp 10 ... phẩm “Tuyển tập các bài toán trong đề thi ... đam mê với các môn học

THCM

N

2.1 Bất đẳng thức 33

Chứng minh tương tự:

bb+√

2016b+ ca≤

√b

√a+√

b+√

c

cc+√

2016c+ab≤

√c

√a+√

b+√

c

Từ đó suy ra

aa+√

2016a+bc+

bb+√

2016b+ ca+

cc+√

2016c+ab≤ 1

Dấu −” xảy ra khi a = b = c =2016

3.

Nhận xét. Chúng ta có vài bài toán tương tự:1. Cho a,b,c dương thỏa mãn ab+bc+ ca = 1. Chứng minh rằng

a√a2 +1

+b√

b2 +1+

c√c2 +1

≤ 32

2. Cho ba số thực dương a,b,c thỏa mãn a+b+ c = 1. Chứng minh rằng

ab√c+ab

+bc√

a+bc+

ca√b+ ca

≤ 12

Bài 10. Cho a,b,c là các số thực dương thay đổi thỏa mãn a+b+c = 1. Tìm giá trị nhỏ nhấtcủa biểu thức

Q = 14(a2 +b2 + c2)+ab+bc+ ca

a2b+b2c+ c2a

(Hải Dương)

Lời giải

Chúng ta có bổ đề quan trọng sau:

Bổ đề 2. Cho a,b,c > 0. Khi đó

(a2 +b2 + c2)(a+b+ c)≥ 3(a2b+b2c+ c2a)

Chứng minh bổ đề trên bằng AM - GM là rất dễ dàng, xin nhường lại cho bạn đọc.

Quay lại bài toán. Áp dụng bổ đề 2 ta được:

Q≥ 14(a2 +b2 + c2)+3(ab+bc+ ca)

a2 +b2 + c2 = 14(a2 +b2 + c2)+3(1−a2−b2− c2)

2(a2 +b2 + c2)

= 14(a2 +b2 + c2)+3

2(a2 +b2 + c2)− 3

2

Ta có:3(a2 +b2 + c2)≥ (a+b+ c)2 = 1⇒ 1

2(a2 +b2 + c2)≥ 1

6(2.1.10.1)

Page 34: 1 ¾ F - · PDF fileđang chuẩn bị cho kì thi tuyển sinh vào các lớp 10 ... phẩm “Tuyển tập các bài toán trong đề thi ... đam mê với các môn học

THCM

N

34 Chương 2. LỜI GIẢI

Áp dụng AM-GM ta có:

272(a2 +b2 + c2)+

32(a2 +b2 + c2)

≥ 9 (2.1.10.2)

Từ (2.1.10.1) và (2.1.10.2) suy ra:

14(a2 +b2 + c2)+3

2(a2 +b2 + c2)≥ 55

6

Từ đó suy ra

Q≥ 556− 3

2=

233

Dấu −” xảy ra khi a = b = c =13

Nhận xét. Đây là một bài toán quen thuộc đã xuất hiện trong đề thi tuyển sinh chuyên củatrường THPT chuyên Phan Bội Châu Nghệ An và trường THPT chuyên Lê Hồng Phong Tp.HCM năm 2012 và bất đẳng thức phụ trên chính là mấu chốt để giải quyết bài toán này. Cácbạn có thể sử dụng bất đẳng thức trên để giải quyết hai bài toán sau:

1. (Vũng Tàu TST 2016) Cho x,y,z không âm thỏa mãn x2 + y2 + z2 = 1. Chứng minh rằng

(x2y+ y2z+ z2x)

(1√

x2 +1+

1√y2 +1

+1√

z2 +1

)≤ 3

2

2. Cho a,b,c > 0 và a2 +b2 + c2 = 3. Chứng minh rằng

ab+

bc+

ca≥ 9

a+b+ c

Bài 11. Cho a,b,c > 0 và a+b+ c≥ 9. Tìm giá trị nhỏ nhất của

A = 2

√a2 +

b2

3+

c2

5+3

√1a+

9b+

25c

(Hải Phòng)

Lời giải

Áp dụng bất đẳng thức Cauchy Schwarz:(a2 +

b2

3+

c2

5

)(1+3+5)≥ (a+b+ c)2⇒ 2

√a2 +

b2

3+

c2

5≥ 2(a+b+ c)

3

1a+

9b+

25c≥ (1+3+5)2

a+b+ c⇒ 3

√1a+

9b+

25c≥ 27√

a+b+ cĐặt t = a+b+ c thì

A≥ 2t3+

27√t=

t6+

t2+

272√

t+

272√

t≥ 9

6+3 3

√t2.

272√

t.

272√

t= 15

Dấu −” xảy ra khi a = 1,b = 3,c = 5.

Page 35: 1 ¾ F - · PDF fileđang chuẩn bị cho kì thi tuyển sinh vào các lớp 10 ... phẩm “Tuyển tập các bài toán trong đề thi ... đam mê với các môn học

THCM

N

2.1 Bất đẳng thức 35

Bài 12. Cho x,y là hai số thực dương. Chứng minh rằng

x√

y+ y√

xx+ y

− x+ y2≤ 1

4

(Tp. HCM)

Lời giải

Áp dụng AM-GM ta có:

x+14≥√

x (2.1.12.1)

y+14≥√y (2.1.12.2)

x+ y≥ 2√

xy (2.1.12.3)

Từ (2.1.12.1) và (2.1.12.2) suy ra

x+ y+12≥√

x+√

y (2.1.12.4)

Từ (2.1.12.3) và (2.1.12.4) suy ra

(x+ y)2 +12(x+ y)≥ 2

√xy(√

x+√

y)

⇔x√

y+ y√

xx+ y

− x+ y2≤ 1

4

Do đó ta thu được điều phải chứng minh.

Bài 13. Cho a,b,c là các số dương thỏa mãn1

a+1+

1b+1

+1

c+1= 2. Chứng minh

18a2 +1

+1

8b2 +1+

18c2 +1

≥ 1

(Lào Cai)

Lời giải

Giả sử tồn tại a,b,c sao cho

18a2 +1

+1

8b2 +1+

18c2 +1

< 1

Suy ra:8a2

8a2 +1>

18b2 +1

+1

8c2 +1≥ 2√

(8b2 +1)(8c2 +1)

Chứng minh tương tự:8b2

b2 +1>

2√(8c2 +1)(8a2 +1)

Page 36: 1 ¾ F - · PDF fileđang chuẩn bị cho kì thi tuyển sinh vào các lớp 10 ... phẩm “Tuyển tập các bài toán trong đề thi ... đam mê với các môn học

THCM

N

36 Chương 2. LỜI GIẢI

8c2

c2 +1>

2√(8a2 +1)(8b2 +1)

Từ đó suy ra

8a2

8a2 +1.

8b2

8b2 +1.

8c2

8c2 +1>

2√(8b2 +1)(8c2 +1)

.2√

(8c2 +1)(8a2 +1).

2√(8a2 +1)(8b2 +1)

Suy ra

abc >18

(2.1.13.1)

Mặt khác từ giả thiết ta có:

1c+1

=a

a+1+

bb+1

≥ 2√

ab√(a+1)(b+1)

Chứng minh tương tự:1

b+1≥ 2

√ca√

(c+1)(a+1)

1a+1

≥ 2√

bc√(b+1)(c+1)

Từ đó suy ra

1a+1

.1

b+1.

1c+1

≥ 2√

ab√(a+1)(b+1)

.2√

ca√(c+1)(a+1)

.2√

bc√(b+1)(c+1)

Suy ra

abc≤ 18

(2.1.13.2)

Từ (2.1.13.1) và (2.1.13.2) ta có mâu thuẫn nên từ đó có điều phải chứng minh.

Nhận xét. Bài toán ở trên có phần hao hao giống bài toán sau: Cho a,b,c > 0 thỏa mãn1

a+1+

1b+1

+1

c+1= 2. Chứng minh rằng

18ab+1

+1

8bc+1+

18ca+1

≥ 1

Bài 14. Cho a,b,c là 3 cạnh của một tam giác. Tìm giá trị nhỏ nhất của

Q =abc

(b+ c−a)(c+a−b)(a+b− c)

(Long An)

Lời giải

Page 37: 1 ¾ F - · PDF fileđang chuẩn bị cho kì thi tuyển sinh vào các lớp 10 ... phẩm “Tuyển tập các bài toán trong đề thi ... đam mê với các môn học

THCM

N

2.1 Bất đẳng thức 37

Áp dụng AM-GM ta được:(b+ c−a)(c+a−b)≤ c2

(b+ c−a)(a+b− c)≤ b2

(a+b− c)(c+a−b)≤ a2

⇒ (b+ c−a)2(c+a−b)2(a+b− c)2 ≤ a2b2c2

⇒ (b+ c−a)(c+a−b)(a+b− c)≤ abc

⇒ Q≥ 1

Dấu −” xảy ra khi a = b = c.

Nhận xét. Bất đẳng thức abc ≥ (a+ b− c)(b+ c− a)(c+ a− b) vẫn đúng với trường hợpa,b,c > 0, và chính là BĐT Schur cho bậc 3. Xin giới thiệu BĐT Schur tổng quát:Với mọi số thực không âm a,b,c,k ta có:

ak(a−b)(a− c)+bk(b− c)(b−a)+ ck(c−a)(c−b)≥ 0

Sau đây là một hướng giải quyết cho BĐT Schur tổng quát. Không mất tính tổng quát, giả sửa≥ b≥ c≥ 0. Khi đó

V T = ck(a− c)(b− c)+(a−b)[ak(a− c)−bk(b− c)

]≥ 0

Khi k = 1 ta có các bất đẳng thức sau:

a3 +b3 + c3 +3abc≥ ab(a+b)+bc(b+ c)+ ca(c+a)

abc≥ (a+b− c)(b+ c−a)(c+a−b)

(a+b+ c)3 +9abc≥ 4(a+b+ c)(ab+bc+ ca)

a2 +b2 + c2 +9abc

a+b+ c≥ 2(ab+bc+ ca)

ab+ c

+b

c+a+

ca+b

+4abc

(a+b)(b+ c)(c+a)≥ 2

Khi k = 2 ta có các bất đẵng thức sau

a4 +b4 + c4 +abc(a+b+ c)≥ ab(a2 +b2)+bc(b2 + c2)+ ca(c2 +a2)

a2 +b2 + c2 +6abc(a+b+ c)

a2 +b2 + c2 +ab+bc+ ca≥ 2(ab+bc+ ca)

Chú ý rằng với k = 2 thì bất đẳng thức Schur đúng với mọi a,b,c ∈ R.Đặt x = a(a−b− c);y = b(b− c−a);z = c(c−a−b). Khi đó bất đẳng thức

x2+y2+z2≥ xy+yz+zx⇔ a4+b4+c4+abc(a+b+c)≥ ab(a2+b2)+bc(b2+c2)+ca(c2+a2)

Bài 15. Cho x,y,z là các số thực thỏa mãn (x− y)(x− z) = 1 và y 6= z. Chứng minh

1(x− y)2 +

1(y− z)2 +

1(z− x)2 ≥ 4

(Nam Định)

Page 38: 1 ¾ F - · PDF fileđang chuẩn bị cho kì thi tuyển sinh vào các lớp 10 ... phẩm “Tuyển tập các bài toán trong đề thi ... đam mê với các môn học

THCM

N

38 Chương 2. LỜI GIẢI

Lời giải

Để ý rằng

1(x− y)2 +

1(z− x)2 =

(x− y)2 +(x− z)2

(x− y)2(x− z)2 =(y− z)2 +2(x− y)(x− z)

(x− y)2(x− z)2

=(y− z)2

(x− y)2(x− z)2 +21

(x− y)(x− z)

Kết hợp sử dụng AM-GM, suy ra:

1(x− y)2 +

1(y− z)2 +

1(z− x)2 =

(y− z)2

(x− y)2(x− z)2 +1

(y− z)2 +2

(x− y)(x− z)≥ 4

(x− y)(x− z)= 4

Nhận xét. Đây chính là dạng biến thể của bài thi học sinh giỏi quốc gia lớp 12 năm 2008 (VMO2008). Xin phát biểu lại đề như sau:Cho a,b,c là các số không âm phân biệt. Chứng minh rằng:

1(a−b)2 +

1(b− c)2 +

1(c−a)2 ≥

4ab+bc+ ca

Để giải quyết bài toán này ta có thể giả sử a = min{a,b,c} và làm tương tự như lời giải trên kếthợp với chú ý rằng ab+bc+ ca≥ (a−b)(a− c).Ngoài ra ta còn có thể sử dụng phương pháp dồn biến về biên như sauNhận thấy rằng bất đẳng thức trên không hề có dấu bằng tại tâm. Ta dự đoán dồn biến về biên.Xét

f (a,b,c) =1

(a−b)2 +1

(b− c)2 +1

(c−a)2 −4

ab+bc+ ca

Không mất tính tổng quát, giả sử c = min{a,b,c}Ta đi chứng minh

f (a,b,c)≥ f(

a+c2,b+

c2,0)

Thật vậy, do:ab+bc+ ca≥

(a+

c2

)(b+

c2

)(b− c)2 ≤

(b+

c2

)2;(a− c)2 ≤

(a+

c2

)2

nênf (a,b,c)≥ f

(a+

c2,b+

c2,0)

Bài toán trên trở thành1

(x− y)2 +1x2 +

1y2 ≥

4xy

hay là1

(x− y)2 +(x− y)2

x2y2 ≥ 2xy

Bất đẳng thức trên đúng theo AM-GM. Do đó ta có điều phải chứng minh.

Page 39: 1 ¾ F - · PDF fileđang chuẩn bị cho kì thi tuyển sinh vào các lớp 10 ... phẩm “Tuyển tập các bài toán trong đề thi ... đam mê với các môn học

THCM

N

2.1 Bất đẳng thức 39

Bài 16. Cho a,b,c là các số thực dương thỏa mãn abc = 1. Chứng minh rằng

1ab+a+2

+1

bc+b+2+

1ca+ c+2

≤ 34

(Ninh Bình)

Lời giải

Áp dụng bất đẳng thức quen thuộc sau1

x+ y≤ 1

4

(1x+

1y

), ta được:

1ab+a+2

=c

1+ac+2c≤ 1

4

(c

c+1+

1a+1

)Chứng minh tương tự ta được:

1bc+b+2

≤ 14

(a

a+1+

1b+1

)1

ca+ c+2≤ 1

4

(b

b+1+

1c+1

)Suy ra:

1ab+a+2

+1

bc+b+2+

1ca+ c+2

≤ 14

(c

c+1+

1a+1

+a

a+1+

1b+1

+b

b+1+

1c+1

)=

34

Do đó ta thu được điều phải chứng minh. Dấu bằng xảy ra khi a = b = c = 1.

Nhận xét. Mở rộng ra, hãy thử chứng minh bài toán sau: Chứng minh rằng với mọi a,b,c dươngta có

12a+b+ c

+1

a+2b+ c+

1a+b+2c

≤ 1a+3b

+1

b+3c+

1c+3a

Bài 17. Cho ba số thực a,b,c thỏa mãn điều kiện ab+bc+ ca = 3. Tìm giá trị nhỏ nhất củabiểu thức

P = a2 +b2 + c2−6(a+b+ c)+2017

(Ninh Thuận)

Lời giải

(a+b+ c−3)2 ≥ 0⇔ a2 +b2 + c2 +2(ab+bc+ ca)−6(a+b+ c)+9≥ 0⇔ a2 +b2 + c2−6(a+b+ c)≥−15⇔ a2 +b2 + c2−6(a+b+ c)+2017≥ 2002⇒MinP = 2002. Dấu bằngxảy ra khi a = b = c = 1

Page 40: 1 ¾ F - · PDF fileđang chuẩn bị cho kì thi tuyển sinh vào các lớp 10 ... phẩm “Tuyển tập các bài toán trong đề thi ... đam mê với các môn học

THCM

N

40 Chương 2. LỜI GIẢI

Bài 18. Cho các số dương x,y. Tìm giá trị nhỏ nhất của biểu thức

P =2√

(2x+ y)3 +1−1+

2√(x+2y)3 +1−1

+(2x+ y)(x+2y)

4− 8

3(x+ y)

(Phú Thọ)

Lời giải

Đặt 2x+ y = a;2y+ x = b khi đó áp dụng AM-GM ta có

P =2√

a3 +1−1+

2√b3 +1−1

+ab4− 8

a+b

=2√

(a+1)(a2−a+1)−1+

2√(b+1)(b2−b+1)−1

+ab4− 8

a+b

≥ 2a+1+a2−a+1

2−1

+2

b+1+b2−b+12

−1+

ab4− 4√

ab=

4a2 +

4b2 +

ab4− 4√

ab

≥ 8ab

+ab4− 4√

ab

Theo đánh giá bằng AM-GM ở trên, ta dự đoán dấu bằng xảy ra khi a = b = 2 do đó ta sẽ chứngminh với t =

√ab > 0 thì

8t2 +

t2

4− 4

t≥ 1⇔ (t−2)2(t2 +4t +8)≥ 0

Bất đẳng thức trên luôn đúng nên ta có điều phải chứng minh là đúng. Vậy giá trị nhỏ nhất của

P = 1 xảy ra khi x = y =23

.

Nhận xét. Hãy thử loại bỏ đi những căn thức "khó chịu"ở mẫu số bằng AM-GM. Sau đây làmột số bài toán tương tự:

1. (Trần Quốc Anh) Cho các số thực không âm a,b,c có tổng bằng 3. Chứng minh rằng

253 3√

4(ab+bc+ ca)≥ a+1

b+1+

b+1c+1

+c+1a+1

2. Cho x,y > 0. Tìm giá trị nhỏ nhất của

P =

√x3

x3 +8y3 +

√4y3

y3 +(x+ y)3

Bài 19. Cho a,b,c là các số thực dương thỏa mãn ab+bc+ ca = 3abc. Chứng minh rằng

1√a3 +b

+1√

b3 + c+

1√c3 +a

≤ 3√2

(Quảng Bình)

Page 41: 1 ¾ F - · PDF fileđang chuẩn bị cho kì thi tuyển sinh vào các lớp 10 ... phẩm “Tuyển tập các bài toán trong đề thi ... đam mê với các môn học

THCM

N

2.1 Bất đẳng thức 41

Lời giải

Áp dụng BĐT AM-GM, ta có:

1√a3 +b

≤ 1√2√

a3b=

1√

2 4√a3b≤ 1

4√

2

(1a+

1a+

1a+

1b

)=

14√

2

(3a+

1b

)Chứng minh tương tự:

1√b3 + c

≤ 14√

2

(3b+

1c

)1√

c3 +a≤ 1

4√

2

(3c+

1a

)Từ đó suy ra

1√a3 +b

+1√

b3 + c+

1√c3 +a

≤ 14√

2

(4a+

4b+

4c

)=

1√2

(ab+bc+ ca

abc

)=

3√2

Do đó ta có điều phải chứng minh. Dấu bằng xảy ra khi a = b = c = 1

Bài 20. Cho ba số thực a,b,c sao cho 0≤ a,b,c≤ 1. Chứng minh

a+b+ c+3abc≥ 2(ab+bc+ ca)

(Quảng Nam)

Lời giải

Bất đẳng thức cần chứng minh tương đương:

c(a−1)(b−1)+b(c−1)(a−1)+a(b−1)(c−1)≥ 0

Bất đẳng thức này luôn đúng nên ta có điều phải chứng minh.

Bài 21. Cho các số thực x,y,z≥ 1 và thỏa mãn 3x2 +4y2 +5z2 = 52. Tìm giá trị nhỏ nhấtcủa biểu thức

F = x+ y+ z

(Thái Bình)

Lời giải

Cách 1. (Lời giải của bạn Đinh Xuân Hùng trường THPT chuyên Lương Văn Tụy Ninh Bình)Đầu tiên ta dự đoán dấu bằng xảy ra khi x = y = 1;z = 3.Do x,y≥ 1 nên

(x−1)(y−1)≥ 0⇔ xy≥ x+ y−1

Page 42: 1 ¾ F - · PDF fileđang chuẩn bị cho kì thi tuyển sinh vào các lớp 10 ... phẩm “Tuyển tập các bài toán trong đề thi ... đam mê với các môn học

THCM

N

42 Chương 2. LỜI GIẢI

Tương tự ta đượcxy+ yz+ zx≥ 2(x+ y+ z)−3

Do đó:5(x+ y+ z)2 = 5(x2 + y2 + z2)+10(xy+ yz+ zx)≥ 52+2x2 + y2 +10 [2(x+ y+ z)−3]

≥ 52+2+1+20(x+ y+ z)−30

Suy rax+ y+ z≥ 5

Dấu bằng xảy ra khi x = y = 1;z = 3.Cách 2. Vẫn dự đoán dấu bằng xảy ra khi x = y = 1;z = 3.Ta thấy rằng khi x = y = 1;z = 3 thì x+ y+ z = 5. Do đó nếu một trong 3 số x,y,z > 3 thìx+ y+ z > 5. Do đó giá trị nhỏ nhất sẽ không xảy ra. Vậy 1≤ x,y,z≤ 3.Ta có đánh giá sau

(x−1)(y−1)(z−1)≥ 0⇔ xyz+ x+ y+ z≥ xy+ yz+ zx+1

(x−3)(y−3)(z−3)≤ 0⇔ 27+3(xy+ yz+ zx)≥ xyz+9

Cộng 2 vế trên lại ta được:

2(xy+ yz+ zx)+26≥ 8(x+ y+ z)

Mặt khác theo cách trên thì x2 + y2 + z2 ≥ 11 do đó

(x+ y+ z)2 ≥ 11+8(x+ y+ z)−26

Do x+ y+ z≥ 3 nên ta suy ra: F = x+ y+ z≥ 5. Dấu bằng xảy ra khi x = y = 1;z = 3.

Nhận xét. Hãy thử sức qua hai bài toán sau:1. (Đề thi HSG TP.HCM 2007) Cho 0 < x ≤ y ≤ z ≤ 1 và 3x+2y+ z ≤ 4. Tìm giá trị lớn

nhất của biểu thứcS = 3x2 +2y2 + z2

2. (HOMC 2013) Cho a,b,c thỏa mãn 0≤ c≤ b≤ a≤ 5; a+b≤ 8 và a+b+ c = 10. Tìmgiá trị lớn nhất của

S = a2 +b2 + c2

Bài 22. Tìm giá trị nhỏ nhất của biểu thức

P =

√x+6

√x−9+

√x−6

√x−9

(Thái Nguyên)

Lời giải

Điều kiện xác định: x≥ 9Ta có:

P =

√x−9+6

√x−9+9+

√x−9−6

√x−9+9 =

√(√x−9+3

)2+

√(√x−9−3

)2

=∣∣∣√x−9+3

∣∣∣+ ∣∣∣3−√x−9∣∣∣=√x−9+3+

∣∣∣3−√x−9∣∣∣≥√x−9+3+3−

√x−9 = 6

⇒MinP = 6. Dấu −” xảy ra khi

{x≥ 93−√

x−9≥ 0⇔ 9≤ x≤ 18.

Page 43: 1 ¾ F - · PDF fileđang chuẩn bị cho kì thi tuyển sinh vào các lớp 10 ... phẩm “Tuyển tập các bài toán trong đề thi ... đam mê với các môn học

THCM

N

2.1 Bất đẳng thức 43

Bài 23. Cho x,y > 0 và x+ y≥ 3. Tìm giá trị nhỏ nhất của

M = 6x2 +4y2 +10xy+4xy+

3yx+2016

(Thừa Thiên - Huế)

Lời giải

Những bài toán như thế này ta cần phải dự đoán giá trị nhỏ nhất xảy ra khi nào? x = y =32

hay

x = 2;y = 1 hay x = 1;y = 2.Trong bài toán này ta dự đoán dấu bằng xảy ra khi x = 1;y = 2cũng tức là ta sẽ chứng minh rằng

A = 6x2 +4y2 +10xy+4xy+

3yx≥ 50

Áp dụng bất đẳng thức Cauchy ta được

xy+4xy≥ 4x

3xy+3yx≥ 6y

Do đó:

A≥ 6x2 +6xy+4y2 +4x+6y = 6x(x+ y)+4y2 +6y+4x≥ 6x.3+4y2 +6y+4x

= 22x+4y2 +6y≥ 22x+4(4y−4)+6y = 22(x+ y)−16≥ 50

Do đóM ≥ 50+2016 = 2066

Dấu -"xảy ra khi và chỉ khi x = 1;y = 2.

Nhận xét. Để giải quyết những bài toán như thế này thông thường ta phải đoán được dấu bằngxảy ra. Nhưng đôi khi dấu bằng không xuất hiện ngay lập tức mà phải đi tìm thông qua một sốphương pháp chẳng hạn như cân bằng hệ số. Sau đây là một số ví dụ về những bài đi tìm giá trịnhỏ nhất không thể giải quyết bằng phương pháp thông thường:

1. Giả sử x,y,z là các số thực không âm thỏa mãn xy+ yz+ zx = 1.Tìm giá trị nhỏ nhất của

P = 10x2 +10y2 + z2

2. (VNTST 2001) Xét các số thực dương a,b,c thỏa mãn 21ab+2bc+8ca≤ 12. Tìm giá trịnhỏ nhất của

P =1a+

2b+

3c

Gợi ý: Bài 2 có thể sử dụng bất đẳng thức Cauchy suy rộng: Cho a1,a2, ...,an không âm và cáchằng số α1,α2, ...,αn dương. Khi đó ta có:

α1a1 +α2a2 + ...+αn.an ≥ (α1 +α2 + ...+αn)(aα11 .aα2

2 ...aαnn )

1α1+α2+...+αn

Page 44: 1 ¾ F - · PDF fileđang chuẩn bị cho kì thi tuyển sinh vào các lớp 10 ... phẩm “Tuyển tập các bài toán trong đề thi ... đam mê với các môn học

THCM

N

44 Chương 2. LỜI GIẢI

Bài 24. Cho a,b,c là các số thực dương thỏa mãn a+b+ c = 3. Chứng minh rằng

4(a2 +b2 + c2)− (a3 +b3 + c3)≥ 9

(Vĩnh Phúc)

Lời giải

Bất đẳng thức cần chứng minh tương đương:

3(a3 +b3 + c3)+27≤ 12(a2 +b2 + c2)

⇔ 3(a3 +b3 + c3)+(a+b+ c)3 ≤ 4(a+b+ c)(a2 +b2 + c2)

⇔ 6abc≤ a2b+b2c+ c2a+ab2 +bc2 + ca2

⇔ 8abc≤ (a+b)(b+ c)(c+a)

Bất đẳng thức cuối cùng luôn đúng theo AM-GM

Nhận xét. Bài toán trên được giải quyết bằng kĩ thuật đồng bậc hóa bất đẳng thức. Sau đây làmột số bài tập về phương pháp này:

1. Cho a,b,c là độ dài ba cạnh tam giác có chu vi bằng 3. Tìm giá trị nhỏ nhất của

P = 3(a2 +b2 + c2)+4abc

2. Cho a,b,c là độ dài ba cạnh tam giác có chu vi bằng 2. Tìm giá trị nhỏ nhất của

P = 4(a3 +b3 + c3)+15abc

2.2 Đại số

Bài 1.1. Giải hệ phương trình:

{x2 +4y2 = 54x2y+8xy2 +5x+10y = 1

2. Giải phương trình:√

5x2 +6x+5 = 64x3+4x5x2+6x+6 .

(Trường THPT chuyên Khoa học Tự nhiên, ĐH KHTN, ĐHQG HN)

Lời giải

1. Hệ phương trình đã cho tương đương với{

x2 +(2y)2 +4xy−4xy = 54xy(x+2y)+5(x+2y) = 1

⇔{

(x+2y)2−4xy = 5(x+2y)(4xy+5) = 1

.

Đặt{

a = x+2yb = 4xy , ta có

{a2−b = 5a(b+5) = 1

⇒ a.a2 = 1⇒ a = 1⇒ b =−4. Đến đây, ta

có hệ{

x+2y = 14xy =−4 ⇔

{x = 1−2yxy =−1 . Thế phương trình trên vào phương trình dưới, ta

có (1−2y)y =−1⇔ 2y2−y−1 = 0⇔[

y = 1y = −1

2. Với y = 1 thì x =−1. Với y =−1

2 thì

x = 2. Thử lại đúng. Như vậy, tập nghiệm của hệ phương trình là S ={(−1;1) ,

(2; −1

2

)}.

Page 45: 1 ¾ F - · PDF fileđang chuẩn bị cho kì thi tuyển sinh vào các lớp 10 ... phẩm “Tuyển tập các bài toán trong đề thi ... đam mê với các môn học

THCM

N

2.2 Đại số 45

2. • Do 5x2 +6x+6 > 5x2 +6x+5 = 2x2 +2+3(x+1)2 > 0∀x nên phương trình luôncó nghĩa với mọi x.• Đặt

√5x2 +6x+5 = a;4x = b, thay vào phương trình trên, ta có a

(a2 +1

)= b3 +

b⇔ a3− b3 + a− b = 0⇔ (a−b)(a2 +ab+b2 +1

)= 0. Vì a2 + ab+ b2 + 1 =(

a+ b2

)2+ 3

4b2+1 > 0∀a;b nên a−b = 0⇔ a = b Vậy√

5x2 +6x+5 = 4x(x≥ 0).

Từ đó 5x2+6x+5= 16x2⇔ 11x2−6x−5⇔[

x = 1x =− 5

11 < 0. Thử lại, ta thấy x= 1

là một nghiệm của phương trình. Như vậy tập nghiệm của phương trình là S = {1}.

Bài 2.1. Giải hệ

{(x−2y)(x+my) = m2−2m−3(y−2x)(y+mx) = m2−2m−3

khi m =−3 và tìm m để hệ có ít nhất một

nghiệm (x0,y0) thỏa x0 > 0;y0 > 0.2. Tìm a≥ 1 để phương trình ax2 +(1−2a)x+1−a = 0 có hai nghiệm phân biệt x1,x2

thỏa x22−ax1 = a2−a−1.

(Trường Phổ thông Năng khiếu - ĐHQG Tp. HCM)

Lời giải

1. • Khi m= 3, ta có hệ{

(x−2y)(x−3y) = 12(y−2x)(y−3x) = 12 ⇒

{x2−5xy+6y2 = 12y2−5xy+6x2 = 12

5(y2− x2)=

0⇒[

x = yx =−y . Lần lượt thay vào, ta tìm được tập nghiệm của hệ phương trình đã

cho là S ={(1;−1) ;(−1;1) ;

(√6;√

6)

;(−√

6;−√

6)}

.

• Với m bất kì, hệ có thể viết lại thành{

x2 +(m−2)xy−2my2 = m2−2m−3y2 +(m−2)xy−2mx2 = m2−2m−3

(2m+1)(y2− x2)= 0.

– Xét m = −12 , hệ phương trình trở thành x2− 5

2xy+ y2 + 74 = 0, có nghiệm(

5+√

22 ,2

)thỏa mãn yêu cầu bài toán.

– Xét m 6= −12 , ta có x = y hoặc x =−y. Trường hợp x =−y loại do không thỏa

mãn yêu cầu bài toán. Với trường hợp x = y, thế vào ta có −(m+1)x2 =m2−2m−3 = (m+1)(m−3).∗ Nếu m = −1 ta có (x−2y)(x− y) = 0. Để phương trình này có nghiệm

thỏa mãn yêu cầu đề bài, ta chỉ cần chọn x = y > 0.∗ Nếu m 6=−1 ta có x2 = 3−m. Để phương trình có nghiệm thì 3−m > 0⇔

m < 3. Khi đó hệ phương trình có nghiệm.Như vậy, các giá trị của m thỏa mãn yêu cầu đề bài là m =−1

2 ;m =−1 và m < 3.2. • Vì ∆ = (1−2a)2− 4a(1−a) = 8a2− 8a+ 1 > 0 nên phương trình luôn có hai

nghiệm phân biệt.• Theo định lý Viète, ta có x1+x2 =

2a−1a ⇒ ax1+ax2 = 2a−1⇒ ax1 = 2a−1−ax2.

Kết hợp với giả thiết, ta có

x22 +ax2−2a+1 = a2−a−1⇔ x2

2 +ax2−a2−a+2 = 0

⇔ ax22 +a2x2−a3−a2 +2a = 0. (2.2.2.1)

Page 46: 1 ¾ F - · PDF fileđang chuẩn bị cho kì thi tuyển sinh vào các lớp 10 ... phẩm “Tuyển tập các bài toán trong đề thi ... đam mê với các môn học

THCM

N

46 Chương 2. LỜI GIẢI

Mà x2 là nghiệm của phương trình nên ta có

ax22 +(1−2a)x2 +1−a = 0. (2.2.2.2)

Lấy (2.2.3.2)−(2.2.3.1), ta có(a2 +2a−1

)x2 = a3+a2−3a+1=

(a2 +2a−1

)(a−1).

– Nhận xét a = 1 thoả mãn điều kiện đề cho.– Với a > 1,x2 = a− 1. Thế vào phương trình ban đầu tìm được a = 0 (không

TMĐK) và a = 3 (TMĐK).Vậy a = 1 hoặc a = 3.

Bài 3. Chứng minh biểu thức sau nhận giá trị nguyên dương với mọi giá trị nguyên dươngcủa n:

P =

(√n2 +(n+1)2 +

√(n−1)2 +n2

)√4n2 +2−2

√4n4 +1

(THPT chuyên Đại học Sư phạm Hà Nội

Lời giải

Xét với n nguyên dương ta có:(√n2 +(n+1)2 +

√(n−1)2 +n2

)2

= n2+(n+1)2+2√

n2 +(n+1)2.

√(n−1)2 +n2+(n−1)2+n2

= 4n2 +2+2√

4n4 +1

⇒√

n2 +(n+1)2 +

√(n−1)2 +n2 =

√4n2 +2+2

√4n4 +1

Vậy:

P =

(√n2 +(n+1)2 +

√(n−1)2 +n2

)√4n2 +2−2

√4n4 +1

=

√4n2 +2+2

√4n4 +1.

√4n2 +2−2

√4n4 +1

=

√(4n2 +2)2−

(2√

4n4 +1)2

=√

16n4 +16n2 +4− (16n4 +4) = 4n.

Vì n nguyên dương nên 4n ∈ N nên ta có điều phải chứng minh.

Bài 4.1. (a) Phân tích đa thức x4 +5x3 +5x2−5x−6 thành nhân tử.

(b) Rút gọn Q =

√x−√

4(x−1)+√

x+√

4(x−1)√x2−4(x−1)

(1− 1

x−1

)với x > 1 và

x 6= 2.2. (a) Giải phương trình: 2(2x−1)−3

√5x−6 =

√3x−8.

(b) Cho bốn số thực a,b,c,d khác 0 thỏa mãn các điều kiện sau: a,b là hai nghiệmcủa phương trình x2− 10cx− 11d = 0; c,d là hai nghiệm của phương trìnhx2−10ax−11b = 0. Tính giá trị của S = a+b+ c+d.

(Bắc Ninh)

Page 47: 1 ¾ F - · PDF fileđang chuẩn bị cho kì thi tuyển sinh vào các lớp 10 ... phẩm “Tuyển tập các bài toán trong đề thi ... đam mê với các môn học

THCM

N

2.2 Đại số 47

Hướng dẫn

1. (a) x4 +5x3 +5x2−5x−6 = (x−1)(x+1)(x+2)(x+3).(b) Trước hết ta tính các hạng tử của Q:√

x−√

4(x−1) =

√(√x−1−1

)2=∣∣∣√x−1−1

∣∣∣√

x+√

4(x−1) =

√(√x−1+1

)2=√

x−1+1√x2−4(x−1) =

√(x−2)

2= |x−2|

Với 1 < x < 2, ta được:

Q =

∣∣√x−1−1∣∣+√x−1+1|x−2|

(1− 1

x−1

)=

1−√

x−1+√

x−1+12− x

.x−2x−1

=2

2− x.x−2x−1

=−2

x−1.

Với x > 2, ta được:

Q =

∣∣√x−1−1∣∣+√x−1+1|x−2|

(1− 1

x−1

)=

√x−1−1+

√x−1+1

x−2.x−2x−1

=2√

x−1x−2

.x−2x−1

=2√

x−1

2. (a) ĐK: x≥ 83

. Biến đổi phương trình ban đầu:

2(2x−1)−3√

5x−6 =√

3x−8

⇔ 4x−12 = 3(√

5x−6−3)+(√

3x−8−1)

⇔ 4(x−3) =15(x−3)√5x−6+3

+3(x−3)√3x−8+1

x−3 = 015√

5x−6+3+

3√3x−8+1

= 4

• x−3 = 0⇔ x = 3 (thỏa ĐK).

• 15√5x−6+3

+3√

3x−8+1= 4⇔

(15√

5x−6+3− 15

6

)+

(3√

3x−8+1− 3

2

)=

0⇔ 156

(3−√

5x−6√5x−6+3

)+

32

(1−√

3x−8√3x−8+1

)= 0⇔ 25

23− x(√

5x−6+3)2 +

92

3− x(√3x−8+1

)2 =

0⇔ (3− x)

(252

1(√5x−6+3

)2 +92

1(√3x−8+1

)2

)= 0⇔ x= 3 (thỏa ĐK).

Vậy phương trình có nghiệm duy nhất x = 3.

Page 48: 1 ¾ F - · PDF fileđang chuẩn bị cho kì thi tuyển sinh vào các lớp 10 ... phẩm “Tuyển tập các bài toán trong đề thi ... đam mê với các môn học

THCM

N

48 Chương 2. LỜI GIẢI

(b) Do a,b là hai nghiệm của phương trình x2−10cx−11d = 0 nên theo định lý Viete,ta có:

a+b = 10c, (2.2.4.1)

ab =−11d. (2.2.4.2)

Tương tự ta cũng có:c+d = 10a, (2.2.4.3)

cd =−11b. (2.2.4.4)

Do a,b,c,d khác 0 nên từ (2.2.4.2) và (2.2.4.4), ta được

ab−11b

=−11d

cd⇔ a

11=

11c⇔ a =

121c

. (2.2.4.5)

Nhân hai vế của (2.2.4.4) với c, phương trình trở thành

c2 + cd = 10ac⇔ c2−11b = 1210⇔ b =c2−1210

11. (2.2.4.6)

Thế (2.2.4.5) và (2.2.4.6) vào (2.2.4.1), phương trình tương đương với:121

c+

c2−121011

= 10c

⇔ c3−110c2−1210c+1331 = 0⇔ (c+11)

(c2−121c+121

)= 0.

⇔[

c =−11c2−121c+121 = 0

Nhận xét: S = a+b+ c+d = 10(a+ c) = 10(

121c

+ c)= 10

(c2 +121

c

).

• Nếu c =−11 thì: S = 10(

112 +121−11

)=−220.

• Nếu c2−121c+121 = 0 thì: S = 10(

c2 +121c

)= 10

(121c

c

)= 1210.

Vậy với giả thiết bài toán như trên thì S =−220 hoặc S = 1210.

Bài 5.

1. Cho biểu thức P =a

a+1+

√1+a2 +

a2

(a+1)2 với a 6= −1. Rút gọn biểu thức P và

tính giá trị của P khi a = 2016.2. Giải các phương trình:

(a) (17−6x)√

3x−5+(6x−7)√

7−3x = 2+8√

36x−9x2−35.(b)√

x2−3x+2 =√

10x−20−√

x−3.

(Đà Nẵng)

Lời giải

Page 49: 1 ¾ F - · PDF fileđang chuẩn bị cho kì thi tuyển sinh vào các lớp 10 ... phẩm “Tuyển tập các bài toán trong đề thi ... đam mê với các môn học

THCM

N

2.2 Đại số 49

1. Với a 6=−1, ta có:

P =a

a+1+

√1+a2 +

a2

(a+1)2 =a

a+1+

√(1+a2 +2a)−2a+

a2

(a+1)2

=a

a+1+

√(a+1)2−2(a+1)

aa+1

+a2

(a+1)2 =a

a+1+

√[(a+1)− a

a+1

]2

=a

a+1+ |(a+1)− a

a+1|.

Nhận xét:

(a+1)− aa+1

=(a+1)2−a

a+1=

a2 +a+1a+1

> 0⇔ a+1 > 0⇔ a >−1

(vì a2 +a+1 =

(a+

12

)2

+34> 0 với mọi x).

Suy ra:• Với a >−1 thì: P =

aa+1

+(a+1)− aa+1

= a+1.

• Với a <−1 thì: P =a

a+1+

aa+1

− (a+1) =−(a2 +1

)a+1

.

• Với a = 2016, khi đó P = a+1 = 2016+1 = 2017.

2. (a) ĐKXĐ:53≤ x≤ 7

3.

Đặt a =√

3x−5 và b =√

7−3x (a,b ≥ 0). Dễ thấy a2 + b2 = 2. Khi đó phươngtrình trên tương đương với:(

2b2 +3)

a+(2a2 +3

)b = a2 +b2 +8ab

⇔ 2ab(a+b)+3(a+b) = (a+b)2 +6ab

⇔ 2ab(a+b−3) = (a+b)(a+b−3)

⇔[

a+b = 3a+b = 2ab

Từ phương trình đầu tiên suy ra: ab =9−a2−b2

272

. Do đó a,b là hai nghiệm của

phương trình X2−3X +73

. Phương trình này vô nghiệm vì ∆ =−5 < 0.

Từ phương trình thứ 2 suy ra: (a+b)2 = 4a2b2⇔ 2+2ab= 4a2b2⇔[

ab = 1+√

3ab = 1−

√3 (loại)

Khi đó: (a+b)2 = a2+b2+2ab = 2+2(1+√

3)=(1+√

3)2⇒ a+b = 1+

√3

(vì a + b ≥ 0). Do đó a,b là hai nghiệm của phương trình X2 −(1+√

3)

X +(1+√

3). Phương trình này vô nghiệm vì ∆ =−12−6

√3 < 0.

Vậy phương trình vô nghiệm.(b) ĐKXĐ: x≥ 3. Khi đó,√

x2−3x+2 =√

10x−20−√

x−3

Page 50: 1 ¾ F - · PDF fileđang chuẩn bị cho kì thi tuyển sinh vào các lớp 10 ... phẩm “Tuyển tập các bài toán trong đề thi ... đam mê với các môn học

THCM

N

50 Chương 2. LỜI GIẢI

⇒ x2−3x+2 = 11x−23−2√

10(x−2)(x−3)

⇒ 2√

10(x−2)(x−3) =−x2 +14x−25

⇒ 40(x−2)(x−3) = (x2−14x+25)2 (x2−14x+25 < 0)

⇒ x4−28x3 +206x2−500x+385 = 0

⇒ (x2−8x+11)(x2−20x+35) = 0.

• x2−8x+11 = 0⇒ x = 4±√

5. Thử lại đúng.

• Nếu x2−20x+35= 0, để ý rằng−x2+14x−25> 0⇒−6x+10> 0⇒ x <53

,không TMĐKXĐ.

Vậy phương trình có hai nghiệm là x = 4+√

5,x = 4−√

5.

Bài 6.1. (a) Giải phương trình x4−2x3 + x−

√2(x2− x) = 0.

(b) Giải hệ phương trình{

x2 +2y−4x = 04x2−4xy2 + y4−2y+4 = 0

.

2. Cho các số thực a,b,c đôi một khác nhau thỏa mãn a3 +b3 + c3 = 3abc và abc 6= 0.Tính

P =ab2

a2 +b2− c2 +bc2

b2 + c2−a2 +ca2

c2 +a2−b2 .

(Hà Nội)

Lời giải

1. (a) • ĐKXĐ: x≤ 0;x≥ 1.• x4−2x3 + x−

√2(x2− x) = 0⇔

(x2− x

)2−(x2− x

)−√

2(x2− x) = 0. Đặt√x2− x = t (t ≥ 0) thì phương trình trở thành

t4− t2−√

2t = 0⇔ t(

t3− t−√

2)= 0

⇔ t(

t−√

2)(

t2 +√

2t +1)= 0⇔

[t = 0t =√

2

– t = 0⇔ x2− x = 0⇔[

x = 0x = 1 . Thử lại đúng.

– t =√

2⇔√

x2− x =√

2⇒ x2− x−2 = 0⇔[

x = 2x =−1 . Thử lai đúng.

Vậy phương trình có tập nghiệm là S = {0;1;2;−1}.

(b){

x2 +2y−4x = 04x2−4xy2 + y4−2y+4 = 0

⇔{

(x−2)2 =−2y+4(2x− y)2−2y+4 = 0

{(x−2)2−2y+4 = 0(2x− y2)2

+(x−2)2 = 0.

Vì(2x− y2)2;(x−2)2 ≥ 0∀x;y⇒

(2x− y2)2

+(x−2)2 ≥ 0∀x;y. Do đó, từ phương

trình trên ta suy được

{(x−2)2 = 0(2x− y2)2

= 0⇔{

x = 22x− y2 = 0

⇔{

x = 2y =±2 . Thử

lại, ta thấy chỉ có (x;y) ∈ {(2;2)} thỏa mãn hệ phương trình. Như vậy, tập nghiệmcủa hệ phương trình là S = {(2;2)}.

Page 51: 1 ¾ F - · PDF fileđang chuẩn bị cho kì thi tuyển sinh vào các lớp 10 ... phẩm “Tuyển tập các bài toán trong đề thi ... đam mê với các môn học

THCM

N

2.2 Đại số 51

2. Do a3+b3+c3−3abc= 0⇔ (a+b+ c)(a2 +b2 + c2−ab−bc− ca

)= 0⇒ a+b+c=

0 (do các số a,b,c đôi một khác nhau)⇒ ab2

a2+b2−c2 =ab2

a2+(b−c)(b+c) =ab2

a2−a(b−c) =b2

a−b+c =

b2

−b−b =−b2 . Tương tự với hai số hạng còn lại, ta có P = −b

2 + −c2 + −a

2 =− (a+b+c)2 = 0.

Bài 7.1. (a) Cho biểu thức

P =

√a3−√

b3

a−b− a√

a+√

b− b√

b−√

a

với a,b > 0 và a 6= b. Thu gọn rồi tính giá trị của P biết (a−1)(b−1)+2√

ab =1.

(b) Cho phương trình x2− x+b = 0 có các nghiệm x1,x2 và phương trình x2−97x+a = 0 có các nghiệm là x4

1;x42. Tìm giá trị của a.

2. (a) Giải phương trình:(9x2−18x+5

)(3x2−4x

)−7 = 0.

(b) Giải hệ phương trình{ √

2x+3y+√

2x−3y = 3√

2y2√

2x+3y−√

2x−3y = 6

(Hải Phòng)

Lời giải

1. (a) Với a,b > 0, đặt x =√

a;y =√

b (lưu ý do a 6= b nên x 6= y ).Theo giả thiết ta có:

(a−1)(b−1)+2√

ab = 1⇔(x2−1

)(y2−1

)+2xy = 1

⇔ x2y2 = x2 + y2−2xy⇔ (xy)2 = (x− y)2⇔ xy = |x− y(x,y > 0)

Khi đó:

P =

√a3−√

b3

a−b− a√

a+√

b− b√

b−√

a=

x3− y3

x2− y2 −x2

x+ y− y2

y− x

=(x− y)

(x2 + y2 + xy

)(x− y)(x+ y)

− x2

x+ y− y2

y− x

=x2 + y2 + xy

x+ y− x2

x+ y− y2

y− x=

y(x+ y)x+ y

− y2

y− x

= y− y2

y− x=−xyy− x

=xy

x− y.

Vậy P = 1 (nếu x > y) và P =−1 (nếu x < y).(b) Do x1,x2 là các nghiệm của phương trình x2−x+b = 0 nên theo định lý Viete, ta có

x1 + x2 = 1, (2.2.7.1)

x1x2 = b. (2.2.7.2)

Tương tự ta cũng cóx4

1 + x42 = 97, (2.2.7.3)

Page 52: 1 ¾ F - · PDF fileđang chuẩn bị cho kì thi tuyển sinh vào các lớp 10 ... phẩm “Tuyển tập các bài toán trong đề thi ... đam mê với các môn học

THCM

N

52 Chương 2. LỜI GIẢI

x41x4

2 = a. (2.2.7.4)

Từ (2.2.7.1) và (2.2.7.2) suy ra

x21 + x2

2 = (x1 + x2)2−2x1x2 = 1−2b

Mặt khác ta cũng suy ra được: 1−2b≥ 0⇔ b≤ 12

.Từ (2.2.7.2), (2.2.7.3) và (2.2.7.5) suy ra:

x41 + x4

2 = 97⇔(x2

1 + x22)2−2(x1x2)

2 = 97

⇔ (1−2b)2−2b2 = 97⇔ 2b2−4b−96 = 0⇔[

b = 8b =−6

Thử lại ta loại b = 8 vì 8 >12

. Với b =−6 suy ra: a = 64 = 1296.2. (a) Ta có:(

9x2−18x+5)(

3x2−4x)−7 = 0⇔ x(3x−5)(3x−1)(3x−4)−7 = 0

⇔ 13.3x(3x−5)(3x−1)(3x−4)−7= 0⇔ 1

3(9x2−15x

)(9x2−15x+4

)−7= 0.

Đặt t = 9x2−15x+2, phương trình trở thành

13(t−2)(t +2)−7 = 0⇔ t2−4−21 = 0⇔ t =±5.

• Với t = 5, ta có 9x2−15x−3 = 0⇔ x =5±√

376

.

• Với t =−5, ta có 9x2−15x+7 = 0 (vô nghiệm).

Vậy nghiệm của phương trình là x =5±√

376

.(b) ĐKXĐ: y≥ 0;2x≥ 3y. Ta có:{ √

2x+3y+√

2x−3y = 3√

2y2√

2x+3y−√

2x−3y = 6 ⇔{ (√

2x+3y+√

2x−3y)2

= 3.6y2√

2x+3y−√

2x−3y = 6

Đặt a =√

2x+3y và b =√

2x−3y (a,b≥ 0) suy ra: a2−b2 = 6y. Khi đó hệ phươngtrình trở thành{

(a+b)2 = 3(a2−b2)

2a−b = 6⇔{

(a+b)(2b−a) = 02a−b = 6

• Với a+b = 0, tìm được a = 2;b =−2 (vô lý vì b≥ 0).• Với 2b−a = 0, tìm được a = 4;b = 2. Ta có hệ phương trình:{ √

2x+3y = 4√2x−3y = 2 ⇔

{x = 5y = 2 (thỏa ĐK)

Vậy hệ có nghiệm duy nhất (x,y) = (5;2).

Page 53: 1 ¾ F - · PDF fileđang chuẩn bị cho kì thi tuyển sinh vào các lớp 10 ... phẩm “Tuyển tập các bài toán trong đề thi ... đam mê với các môn học

THCM

N

2.2 Đại số 53

Bài 8.1. Cho hai số thức a,b sao cho |a| 6= |b| và ab 6= 0 thỏa mãn điều kiện a−b

a2+ab +a+b

a2−ab =3a−ba2−b2 . Tìm giá trị của biểu thức P = a3+2a2b+3b3

2a3+ab2+b3 .2. (a) Giải phương trình x2−6x+4+2

√2x−1 = 0.

(b) Giải hệ phương trình{

x3− y3 = 9(x+ y)x2− y2 = 3

.

(Tp. HCM)

Hướng dẫn

1. Điều kiện đã cho tương đương với⇔ a−ba(a+b)+

a+ba(a−b) =

3a−b(a−b)(a+b)⇔

(a−b)(a−b)+(a+b)(a+b)a(a−b)(a+b) =

a(3a−b)a(a−b)(a+b) ⇔ a2−2ab+b2+a2+2ab+b2 = a(3a−b)⇔ 2a2+2b2−3a2+ab = 0⇔

a2−ab−2b2 = 0⇔ (a+b)(a−2b) = 0⇔[

a =−ba = 2b

• a =−b⇒ P = −b3+2b3+3b3

−2b3−b3+b3 =−2.

• a = 2b⇒ P = 8b3+8b3+3b3

16b3+2b3+b3 = 1

2. (a) • ĐKXĐ: x≥ 12

.

• Phương trình đã cho⇔ x2−4x+4−2x+2√

2x−1= 0⇔ (x−2)2−(2x−1−2

√2x−1+1

)= 0⇔ (x−2)2−

(√2x−1−1

)2= 0⇔

[x−2 =

√2x−1−1

x−2 = 1−√

2x−1. Đến đây tìm

được hai nghiệm của phương trình là x = 2+√

2;x = 4−√

6.

(b) Chia cả hai phương trình vế theo vế, ta được x3−y3

x2−y2 = 3(x+ y)⇔ (x−y)(x2+xy+y2)(x−y)(x+y) =

3(x+ y)⇔ x2 + xy+ y2 = 3(x+ y)2⇔ 2x2 +5xy+2y2 = 0⇔[

x =−12y

x =−2y• Trường hợp x =−1

2y, thế vào phương trình sau, ta được −34y2 = 3⇔ y2 =−4

(vô nghiệm).• Trường hợp x =−2y, thế vào phương trình sau, ta được 3y2 = 3⇔ y= 1;x =−2

hoặc y =−1;x = 2.

Bài 9.1. (a) Đặt a =

√2;b = 3

√2. Chứng minh rằng:

1a−b

− 1b= a+b+

ab+

ba+1.

(b) Cho x = 3√√

28+1− 3√√

28−1+2. Tính giá trị của P = x3−6x2+21x+2016.2. (a) Giải hệ phương trình{

x2y2−2x+ y2 = 02x2−4x+3 =−y3

(b) Giải phương trình(√

2x+5−√

2x+2)(

1+√

4x2 +14x+10)= 3.

(Hưng Yên)

Lời giải

Page 54: 1 ¾ F - · PDF fileđang chuẩn bị cho kì thi tuyển sinh vào các lớp 10 ... phẩm “Tuyển tập các bài toán trong đề thi ... đam mê với các môn học

THCM

N

54 Chương 2. LỜI GIẢI

1. (a) Dễ thấy a2 = b3 = 2 và a+1 =1

a−1.

a+b+ab+

ba+1+

1b=

a2b+ab2 +a2 +b2 +ab+aab

=(a+1)

(b2 +ab+a

)ab

=(a+1)

(ab2 +a2b+a2)

a2b=

(a+1)(ab2 +b4 +b3)

b4

=(a+1)

(a+b2 +b

)b2 =

a+b2 +bb2 (a−1)

=b3 +b2 +ab

b3 (a−1)=

a2 +b2 +abab3−b3 =

a2 +b2 +aba3−b3 =

1a−b

.

Từ đó suy ra:1

a−b− 1

b= a+b+

ab+

ba+1.

(b) Ta có:

x =3√√

28+1− 3√√

28−1+2⇔ x−2 =3√√

28+1− 3√√

28−1

⇔ (x−2)3 =

(3√√

28+1− 3√√

28−1)3

⇔ (x−2)3 = 2−3 3

√(√28+1

)(√28−1

)(3√√

28+1− 3√√

28−1)

⇔ (x−2)3 = 2−9(x−2)⇔ x3−6x2 +21x = 28.

Suy ra:P = x3−6x2 +21x+2016 = 28+2016 = 2044.

Vậy P = 2044.2. (a) Ta xem phương trình thứ nhất như một tam thức bậc 2 theo ẩn x, khi đó ∆′ = 1− y4.

Hệ phương trình có nghiệm

⇔ ∆′ ≥ 0⇔−1≤ y≤ 1. (2.2.9.1)

Mặt khác, xét phương trình thứ hai, ta có:

2x2−4x+3 =−y3⇔ 2(x−1)2 =−1− y3

Dễ thấy:−1− y3 ≥ 0⇔ y≤−1 (2.2.9.2)

(vì 2(x−1)2 ≥ 0 với mọi x).Từ (2.2.9.1) và (2.2.9.2), suy ra: y = −1. Với y = −1 thì: 2(x−1)2 = −1− y3 =

0⇔ (x−1)2 = 0⇔ x = 1.Vậy hệ phương trình có nghiệm duy nhất (x,y) = (1,−1).

(b) ĐKXĐ: x≥−1. Đặt a =√

2x+5 và b =√

2x+2 (a,b≥ 0). Nhận xét:i. a≥

√3 > 1 và a > b.

ii. a2−b2 = 3.iii. 4x2 +14x+10 = a2b2.

Page 55: 1 ¾ F - · PDF fileđang chuẩn bị cho kì thi tuyển sinh vào các lớp 10 ... phẩm “Tuyển tập các bài toán trong đề thi ... đam mê với các môn học

THCM

N

2.2 Đại số 55

Thế (ii) và (iii) vào phương trình ban đầu, ta được:

(a−b)(1+ab) = a2−b2

⇔ (a−b)(1+ab) = (a−b)(a+b)⇔ (a−b)(1+ab−a−b) = 0⇔ (a−b)(a−1)(b−1) = 0⇔ b = 1(do (i))

Nếu b = 1 thì:√

2x+2 = 1⇔ 2x+2 = 1⇔ x =−12

(thỏa ĐK).

Vậy x =−12

là nghiệm duy nhất của phương trình.

Bài 10.

1. (a) Rút gọn biểu thức P =

√1− 1

22

√1− 1

32 ...

√1− 1

20162 .

(b) Cho a là nghiệm của phương trình x2−3x+1 = 0. Không tính giá trị của a, hãy

tính giá trị của biểu thức Q =a2

a4 +a2 +1.

2. (a) Giải phương trình(

x−1x+2

)2

− 15x2−4

+4(

x+1x−2

)2

= 5.

(b) Giải hệ phương trình{ (x2− xy

)(xy− y2)= 25√

x2− xy+√

xy− y2 = 3(x− y)

(Khánh Hoà)

Hướng dẫn

1. (a) Nhận xét:

√1− 1

n2 =

√n−1

√n+1

nvới mọi n≥ 2. Khi đó:

P =

√1− 1

22

√1− 1

32 ........

√1− 1

20162

=1.√

32

.

√2.√

33

........

√2014.

√2015

2015.

√2015.

√2017

2016

=1.√

3.√

2.√

4........√

2014.√

2016.√

2015.√

20172016!

=

√2.√

2016.√

20172.2016

=

√20174032

Vậy P =

√20174032

.

(b) Do a là nghiệm của phương trình x2− 3x+ 1 = 0 nên suy ra: a2− 3a+ 1 = 0⇔a2 +1 = 3a. Khi đó:

Q =a2

a4 +a2 +1=

a2

(a2 +1)2−a2=

a2

(3a)2−a2=

a2

8a2 =18.

Page 56: 1 ¾ F - · PDF fileđang chuẩn bị cho kì thi tuyển sinh vào các lớp 10 ... phẩm “Tuyển tập các bài toán trong đề thi ... đam mê với các môn học

THCM

N

56 Chương 2. LỜI GIẢI

2. (a) ĐKXĐ: x 6=±2. Phương trình ban đầu tương đương với:[(x−1x+2

)2

− 14

]−(

15x2−4

+154

)+4

[(x+1x−2

)2

− 14

]= 0

⇔ 3x(x−4)

4(x+2)2 −15x2

4(x2−4)+

3x(x+4)

(x−2)2 = 0

⇔ x

[x−4

(x+2)2 −5x

x2−4+

4(x+4)

(x−2)2

]= 0

x = 0 (thỏa ĐK)

x−4

(x+2)2 −5x

x2−4+

4(x+4)

(x−2)2 = 0

Giải phương trình thứ hai:x−4

(x+2)2 −5x

x2−4+

4(x+4)

(x−2)2 = 0

⇔ (x−4)(x−2)2 +4(x+4)(x+2)2−5x(x2−4

)= 0

⇔ 24x2 +120x+48 = 0⇔ x2 +5x+2 = 0

x =−5+

√17

2(thỏa ĐK)

x =−5−

√17

2(thỏa ĐK)

Vậy tập nghiệm của phương trình là: S =

{0;−5+

√17

2;−5−

√17

2

}.

(b) ĐKXĐ: x2 ≥ xy và xy≥ y2. Từ phương trình thứ hai, ta thấy: 3(x− y) =V T ≥ 0⇔x ≥ y. Đặt a =

√x2− xy và b =

√xy− y2 (a,b ≥ 0). Dễ thấy: a2− b2 = (x− y)2.

Hệ phương trình tương đương với:{ (x2− xy

)(xy− y2)= 25√

x2− xy+√

xy− y2 = 3(x− y)⇔

{ (x2− xy

)(xy− y2)= 25(√

x2− xy+√

xy− y2)2

= 9(x− y)2

⇔{

(ab)2 = 25(a+b)2 = 9

(a2−b2) ⇔{ (ab)2 = 25

(a+b)(4a−5b) = 0⇔{

ab = 5 (vì a,b≥ 0)(a+b)(4a−5b) = 0

Do ab = 5 nên suy ra: a,b > 0. Khi đó phương trình thứ hai ⇔ 4a− 5b = 0 hay4a = 5b.Ta có: 4a = 5b⇔ 4a2 = 5ab = 25⇔ a =

52=⇒ b = 2. Như vậy,

(x− y)2 =

(52

)2

−22

x2− y2 =

(52

)2

+22⇔

x− y =

32(do x≥ y)

(x− y)(x+ y) =414

x− y =

32

x+ y =416

x =

256

y =83

Page 57: 1 ¾ F - · PDF fileđang chuẩn bị cho kì thi tuyển sinh vào các lớp 10 ... phẩm “Tuyển tập các bài toán trong đề thi ... đam mê với các môn học

THCM

N

2.2 Đại số 57

Thử lại ta thấy (x,y) =(

256,83

)thỏa mãn ĐK. Vậy hệ phương trình có nghiệm duy

nhất (x,y) =(

256,83

).

Bài 11.1. Cho biểu thức P = 1√

x+1 +10

2√

x+1 −5

2x+3√

x+1 với điều kiện x≥ 0.(a) Rút gọn biểu thức P.(b) Tìm tất cả các số tự nhiên x để P là số nguyên tố.

2. Cho phương trình x2−2(m−1)x−2m+5 = 0 (m là tham số). TÌm m để phương trìnhcó hai nghiệm x1,x2 sao cho x1 + x2 +2x1x2 = 26.

3. Giải phương trình 2(x2 +2

)= 5√

x3 +1.

(Long An)

Hướng dẫn

1. • P = 2√

x+1+10(√

x+1)−5(√

x+1)(2√

x+1) = 12√

x+6(√

x+1)(2√

x+1) =6√x+1 .

• Vì P là số nguyên tố nên√

x+1 = 2 hoặc√

x+1 = 3. Khi đó x = 1 hoặc x = 4.

2. • Phương trình có nghiệm⇔ ∆′ ≥ 0⇔ m2−4≥ 0⇔[

x≥ 2x≤−2 .

• Với ∆′ ≥ 0, theo định lý Viète, ta có{

x1 + x2 = 2m−2x1x2 =−2m+5 . Khi đó, x1 + x2 + x1x2 =

26⇔ 2m−2−4m+10 = 26⇔ m =−9.3. • ĐKXĐ: x≥−1.

• 2(x2 +2

)= 5√

x3 +1⇔ 2(x2− x+1+ x+1

)= 5√(x+1)(x2− x+1)⇔ 2 x+1

x2−x+1−

5√

x+1x2−x+1 +2= 0⇔

√ x+1x2−x+1 = 2√

x+1x2−x+1 = 1

2

⇔[

4x2−5x+3 = 0(l)x2−5x−3 = 0

[x = 5+

√37

2x = 5−

√37

2

.

Thử lại đúng. Như vậy, phương trình đã cho có tập nghiệm S ={

5±√

372

}.

Bài 12.1. (a) Đơn giản biểu thức

√x+2+2

√x+1−

√x+2−2

√x+1 với x > 0.

(b) Cho a,b,c là các số thực thỏa mãn điều kiện a+b+c = 6; 1a+b +

1b+c +

1c+a = 47

60 .Tính giá trị của biểu thức a

b+c +b

a+c +c

a+b .2. (a) Giải phương trình

√2x2 +3x+1+

√1−3x = 2

√x2 +1.

(b) Giải hệ phương trình{

x2 +3y2−3x−1 = 0x2− y2− x−4y+5 = 0

.

(Nam Định)

Hướng dẫn

1. (a) A =√

x+1+1+√

x+1−1 = 2√

x+1.(b) (a+b+ c)

( 1a+b +

1b+c +

1a+c

)= a+b+c

a+b + a+b+cb+c + a+b+c

c+a = 1+ ca+b +1+ a

b+c +1+b

a+c = 3+ ca+b +

ab+c +

ba+c = 6.47

60 = 4710 ⇒

ab+c +

ba+c +

ca+b = 47

10 −3 = 1710 .

Page 58: 1 ¾ F - · PDF fileđang chuẩn bị cho kì thi tuyển sinh vào các lớp 10 ... phẩm “Tuyển tập các bài toán trong đề thi ... đam mê với các môn học

THCM

N

58 Chương 2. LỜI GIẢI

2. (a)√

2x2 +3x+1+√

1−3x ≤√

2(2x2 +3x+1+1−3x) = 2√

x2 +1. Vậy phương

trình ban đầu có nghiệm⇔ 2x2 + 3x+ 1 = 1− 3x⇔ 2x2 + 6x = 0⇔[

x = 0x =−3 .

Thử lại, ta thấy x = 0;x =−3 đều thỏa mãn phương trình đã cho.(b) Cộng hai phương trình vế theo vế, ta được 2x2+2y2−4x−4y+4= 0⇔ 2

(x2−2x+1

)+

2(y2−2y+1

)= 0⇔ 2(x−1)2+2(y−1)2 = 0(∗)⇔ x = y = 1. Thử lại đúng. Vậy

(x,y) = (1,1) là nghiệm duy nhất của phương trình.

Bài 13.1. Cho các số a,b thoả mãn 2a2 +11ab−3b2 = 0,b 6= 2a,b 6=−2a. Tính giá trị của biểu

thức T = a−2b2a−b +

2a−3b2a+b .

2. (a) Giải phương trình√

2x+1−√

x−3 = 2.

(b) Giải hệ phương trình{

2x3 + x2y+2x2 + xy+6 = 0x2 +3x+ y = 0

(Phú Thọ)

Lời giải

1. Ta có T = a−2b2a−b +

2a−3b2a+b = (a−2b)(2a+b)+(2a−3b)(2a−b)

(2a−b)(2a+b) = 6a2−11ab+b2

4a2−b2 Từ giả thiết ta suy ra

11ab=−2a2+3b2, thay vào T ta được T = 6a2−11ab+b2

4a2−b2 = 6a2+2a2−3b2+b2

4a2−b2 =2(4a2−b2)

4a2−b2 = 22. (a) • ĐKXĐ: x≥ 3.

• Phương trình đã cho⇔√

2x+1=√

x−3+2⇔ 2x+1= x−3+4+4√

x−3⇔

4√

x−3= x⇔ 16(x−3)= x2⇔ x2−16x+48= 0⇔[

x = 4x = 12 . Thử lại đúng.

(b) Hệ phương trình đã cho⇔{ (

x2 + x)(2x+ y) =−6(

x2 + x)+(2x+ y) = 1

Đặt u = x2 + x;v = 2x+ y,

hệ đã cho trở thành{

uv =−6u+ v = 1 ⇔

{

u =−2v = 3{u = 3v =−2

.

• Với{

u =−2v = 3 ⇒

{x2 + x =−22x+ y = 3

, vô nghiệm.

• Với{

u = 3v =−2 ⇒

{x2 + x = 32x+ y =−2

⇔{

x2 + x−3 = 0y =−2x−2

. Giải hệ phương trình

này, ta được 2 nghiệm

{x = −1−

√13

2y =√

13−1;

{x = −1+

√13

2y =−

√13−1

.

Vậy hệ đã cho có tập nghiệm S ={(−1−

√13

2 ;√

13−1)

;(−1+

√13

2 ;−√

13−1)}

.

Page 59: 1 ¾ F - · PDF fileđang chuẩn bị cho kì thi tuyển sinh vào các lớp 10 ... phẩm “Tuyển tập các bài toán trong đề thi ... đam mê với các môn học

THCM

N

2.2 Đại số 59

Bài 14.1. Cho phương trình x4 +3x3−mx2 +9x+9 = 0 (m là tham số).

(a) Giải phương trình khi m =−2.(b) Tìm tất cả các giá trị của m để phương trình đã cho có ít nhất một nghiệm dương.

2. Giải phương trình 3x2−4x√

4x−3+4x−3 = 0.

(Vĩnh Phúc)

Lời giải

1. (a) Với m =−2, phương trình đã cho trở thành x4 +3x3 +2x2 +9x+9 = 0.• Nhận xét x = 0 là một nghiệm của phương trình.• Với x 6= 0, chia hai vế của phương trình cho x2, ta được

x2 +9x2 +3

(x+

3x

)+2 = 0.

Đặt t = x+ 3x , ta được phương trình t2 +3t−4 = 0⇔ t = 1; t =−4. Với t = 1

thì phương trình vô nghiệm, với t = −4 thì phương trình có hai nghiệm làx =−1;x =−3.

(b) Trong trường hợp tổng quát, với cách đặt như trên, ta có phương trình

t2 +3t−6−m = 0(1) . (2.2.14.1)

Ta cót = x+

3x⇔ x2− tx+3 = 0(2) . (2.2.14.2)

Điều kiện để phương trình (2.2.14.2) có nghiệm dương là t ≥ 2√

3. Xét phươngtrình (2.2.14.1) có ∆ = 4m+33≥ 0⇔ m≥−33

4 . Khi đó t1,2 = −3±√

4m+332 . Do đó

(2.2.14.1) có nghiệm t ≥ 2√

3 khi −3+√

4m+332 ≥ 2

√3⇔ m≥ 6

(1+√

3).

2. • ĐKXĐ: x≥ 34 .

• Phương trình đã cho tương đương với(x−√

4x−3)(

3x−√

4x−3)= 0⇔

[ √4x−3 = x√4x−3 = 3x

.

–√

4x−3 = x⇔{

x≥ 04x−3 = x2 ⇔ x = 1;x = 3.

–√

4x−3 = 3x⇔{

x≥ 04x−3 = 9x2 ⇔

{x≥ 09x2−4x+3 = 0

, vô nghiệm.

Kết hợp điều kiện, ta suy ra các nghiệm của phương trình là x = 1;x = 3.

Page 60: 1 ¾ F - · PDF fileđang chuẩn bị cho kì thi tuyển sinh vào các lớp 10 ... phẩm “Tuyển tập các bài toán trong đề thi ... đam mê với các môn học

THCM

N

60 Chương 2. LỜI GIẢI

2.3 Hình học phẳng

Bài 1. Cho hình vuông ABCD nội tiếp đường tròn tâm (O),P là điểm thuộc cung nhỏ ADcủa đường tròn (O) và P khác A,D. Các đường thẳng PB,PC lần lượt cắt đường thẳng AD tạiM,N. Đường trung trực của AM cắt các đường thẳng AC,PB lần lượt tại E,K. Đường trungtrực của DN cắt các đường thẳng BD,PC lần lượt tại F,L.

1. Chứng minh ba điểm K,O,L thẳng hàng.2. Chứng minh đường thẳng PO đi qua trung điểm EF .3. Giả sử đường thẳng EK cắt đường thẳng BD tại S, các đường thẳng FL và AC cắt nhau

tại T , đường thẳng ST cắt các đường thẳng PC,PB lần lượt tại U,V . Chứng minh rằngbốn điểm K,L,U,V cùng thuộc một đường tròn.

(THPT chuyên KHTN, ĐH KHTN, ĐHQG HN)

Lời giải

1. Xét ∆BAM vuông tại A. Vì trung trực của AM song song AB và đi qua trung điểm của AMnên đường trung trực đó là đường trung bình ∆BAM⇒ K là trung điểm BM.∆BMD có O là trung điểm BD,K là trung điểm BM⇒OK//MD. Tương tự có OL//MD.Vậy theo tiên đề Euclide, ta có O,K,L thẳng hàng (đpcm).

2. • Ta có E thuộc đường trung trực AM⇒ ∆EAM cân tại E, mà EAM = 45o⇒ ∆EAMvuông cân tại E⇒ME⊥AC. Tương tự cũng có NF⊥BD.• MN//BC⇒ PB

MB = PCNC (Định lý Thales). Hạ PX⊥AC,PY⊥BD thì PX//EM//BO

(cùng vuông góc với AC)⇒ XOEO = PB

MB . Mặt khác, PY//NF//CO (cùng vuông gócvới BD)⇒ YO

FO = PCNC ⇒

XOEO = YO

FO ⇒ XY//EF (định lý Thales đảo).

Page 61: 1 ¾ F - · PDF fileđang chuẩn bị cho kì thi tuyển sinh vào các lớp 10 ... phẩm “Tuyển tập các bài toán trong đề thi ... đam mê với các môn học

THCM

N

2.3 Hình học phẳng 61

• Ta có PXO = PYO = XOY = 90o nên PXOY là hình chữ nhật⇒ PO đi qua trungđiểm XY. Mà EF//XY nên PO đi qua trung điểm EF (đpcm).

3. • Ta có KL//AD⇒ KL⊥ES⇒ KOA = OAD = 45o (so le trong) ⇒ KOE = 45o.Mà EOS = 90o nên OK là phân giác của EOS⇒ ∆EOS cân, KS = KE ⇒ KL làtrung trực của ES hay E đối xứng với S qua KL. Tương tự ta có F,T đối xứng quaKL⇒ ∆EOF = ∆SOT ⇒ EFO = STO.• Gọi giao điểm OP và EF là I thì I là trung điểm EF mà ∆EOF vuông tại O⇒

IO= IE = IF.⇒ ∆IOF cân⇒ IOF = IFO= OT S và EOI+ IOF = EOF = 90o⇒EOI + OT S = 90o.• Gọi giao của OP và ST là H⇒ TOH = IOE (đối đỉnh)⇒ TOH + HTO = 90o⇒

T HO = 90o⇒ OP⊥ST .• PLF = PCD (Vì FL//CD cùng vuông góc với AD), PCD = PBD = BPO (OP =

OB) hay PLF = BPO = V PH. Lại có PH⊥UV tại H ⇒ V PH + HV P = 90o. MàPLF + PLK = FLK = 90o⇒ PLK = HV P hay UV P = PKL⇒ Tứ giác KULV nộitiếp (đpcm).

Bài 2. 4ABC nhọn có ABC > 45o. Dựng các hình vuông ABMN,ACPQ (M và C khác phíađối với AB,B và Q khác phía đối với AC). AQ cắt BM tại E,NA cắt CP tại F .

1. Chứng minh4ABE ∼4ACF và tứ giác EFQN nội tiếp.2. Chứng minh trung điểm I của EF là tâm đường tròn ngoại tiếp4ABC.3. MN cắt PQ tại D. Đường tròn ngoại tiếp của hai tam giác DMQ và DNP cắt nhau tại K

khác D. Tiếp tuyến tại B và C của đường tròn ngoại tiếp4ABC cắt nhau tại J. Chứngminh D,A,K,J thẳng hàng.

(Trường Phổ thông Năng khiếu, ĐHQG Tp. HCM)

Lời giải

Page 62: 1 ¾ F - · PDF fileđang chuẩn bị cho kì thi tuyển sinh vào các lớp 10 ... phẩm “Tuyển tập các bài toán trong đề thi ... đam mê với các môn học

THCM

N

62 Chương 2. LỜI GIẢI

1. Ta có: EAB+ BAC = 90o, FAC+ BAC = 90o. Suy ra EAB = FAC. Mặt khác có: ABE =

ACF = 90O. Suy ra ∆ABE ∼ ∆ACF . Suy ra AE.AC = AF.AB mà AC = AQ,AB = AN.Suy ra AE.AQ = AN.AF . Suy ra tứ giác QNEF nội tiếp.

2. Cách 1: Gọi T là giao điểm của MB và CP. Ta có AT BC nội tiếp và AT là đường kínhđường tròn ngoại tiếp ∆ABC. Mắc khác ta có AF ‖ ET,AE ‖ T F nên AET F là hình bìnhhành. Suy ra trung điểm EF cũng là trung điểm AT . Do đó trung điểm I của EF là tâmđường tròn ngoại tiếp ∆ABC.Cách 2: Xét hình thang AEBF , gọi X là trung điểm AB khi đó IX thuộc đường trung bìnhcủa hình thang, suy ra IX ‖ BE hay IX vuông góc AB vậy IX là trung trực của đoạn AB.Chứng minh tương tự thì I cũng thuộc trung trực AC. Vậy I là tâm ngoại tiếp ∆ABC

3. DA cắt EF tại K′ ta có NFK′= NQA (vì NQFE nội tiếp). Mà NQA = NDA (vì AQDN nộitiếp). Suy ra NDA = AFK′. Suy ra NDFK′ nội tiếp. Chứng minh tương tự ta có DK′QEnội tiếp. Do đó K′ là giao điểm của đường tròn ngoại tiếp tam giác DQM và DPN. VậyK ≡ K′. Suy ra D,A,K thẳng hàng.Ta có: BKE = EAB = CAF = CKF . Suy ra BKC = 180o−2.BKE = 2.(90O− EAB) =2.BAC = BIC. Suy ra BKIC là tứ giác nội tiếp. Mà IBJC nội tiếp, suy ra JB = JC nênBKJ = CKJ. Hay KJ là phân giác BKC.Mặt khác BKA = 180o− AEB = 180o− AFC = AKC. Suy ra tia đối tia KA cũng là phângiác của BKC. Do đó A,K,J thẳng hàng. Vậy 4 điểm D,A,K,J thẳng hàng.

Bài 3. Cho tam giác ABC nhọn, AB < AC. Kẻ đường cao AH. Đường tròn (O) đường kínhAH cắt cạnh AB,AC tương ứng tại D,E. Đường thẳng DE cắt đường thẳng BC tại S.

1. Chứng minh rằng BDEC là tứ giác nội tiếp.2. Chứng minh rằng SB.SC = SH2.3. Đường thẳng SO cắt AB,AC tương ứng tại M,N, đường thẳng DE cắt HM,HN tương

ứng tại P,Q. Chứng minh rằng BP,CQ và AH đồng quy.

(THPT chuyên Đại học Sư phạm Hà Nội)

Lời giải

1. Ta có ADHE là tứ giác nội tiếp⇒ AED = AHD = ABH (cùng phụ với BAH)⇒ AED+

DEC = ABH + DEC = 180o⇒ BDEC là tứ giác nội tiếp (Tổng 2 góc đối bằng 180o).

Page 63: 1 ¾ F - · PDF fileđang chuẩn bị cho kì thi tuyển sinh vào các lớp 10 ... phẩm “Tuyển tập các bài toán trong đề thi ... đam mê với các môn học

THCM

N

2.3 Hình học phẳng 63

2. Ta có O là tâm đường tròn đường kính AH nên O là trung điểm AH. Vì AH⊥SH tại Hnên SH là tiếp tuyến của đường tròn tâm O. Khi đó SHD = DEH (góc giữa tiếp tuyến với

dây cung và góc nội tiếp cùng chắn cung_

DH). Xét ∆SDH và ∆SHE có:Góc chung DSH

SHD = DEH

}⇒ ∆SDH ∼ ∆SHE (g.g)⇒ SD

SH = SHSE ⇒ SH2 = SD.SE = SB.SC.

3. Gọi K là giao điểm của BP và CQ. Áp dụng định lý Menelaus cho các tam giác ta có:∆SBM với A,N,C thẳng hàng nên AM

AB .NSNM .CB

CS = 1

∆SBP với K,Q,C thẳng hàng nên KBKP .

QPQS .

CSCB = 1

∆SPM với H,Q,N thẳng hàng nên HPHM .QS

QP .NMNS = 1

Nhân theo vế ta có: AMAB .

KBKP .

HPHM = 1

Áp dụng định lý Menelaus đảo vào ∆BPM với 3 điểm A,K,H ta được A,K,H thẳnghàng.

Bài 4. Từ một điểm M nằm ngoài (O) vẽ hai tiếp tuyến MA,MB đến đường tròn (A,B là haitiếp điểm). Qua A vẽ đường thẳng song song với MB cắt đường tròn tại C; đoạn thẳng MC cắtđường tròn tại D. Hai đường thẳng AD,MB cắt nhau tại E. Chứng minh rằng

1. Tứ giác MAOB nội tiếp.2. ME2 = ED.EA.3. E là trung điểm đoạn MB.

(An Giang)

Lời giải

1. Tứ giác MAOB có:

{MAO = 90o (tiếp tuyến vuông góc với bán kính tại tiếp điểm)MBO = 90o (tiếp tuyến vuông góc với bán kính tại tiếp điểm)

nên

MAOB là tứ giác nội tiếp do có hai góc đối diện bù nhau.

Page 64: 1 ¾ F - · PDF fileđang chuẩn bị cho kì thi tuyển sinh vào các lớp 10 ... phẩm “Tuyển tập các bài toán trong đề thi ... đam mê với các môn học

THCM

N

64 Chương 2. LỜI GIẢI

2. Xét ∆MED và ∆AEM có:• DME = ACM (So le trong)• MAE = ACM (Cùng chắn cung AD)

⇒ DME = MAE. Mặt khác, E chung ta suy ra hai tam giác đó đồng dạng, dẫn đến

MEAE

=EDEM⇒ME2 = ED.EA (2.3.4.1)

3. Ta có ∆BED và ∆AEB đồng dạng do• EBD = BAD (cùng chắn cung BD).• E chung.

Do đóEBEA

=EDEB⇒ EB2 = EA.ED (2.3.4.2)

Từ (2.3.4.1) và (2.3.4.2) suy ra EM = EB hay E là trung điểm MB.

Bài 5. Cho hai đường tròn (O;R),(O′;R′) cắt nhau tại A và B (OO′ > R > R′). Trên nửa mặtphẳng bờ OO′ có chứa điểm A, kẻ tiếp tuyến chung của MN của hai đường tròn trên (với Mthuộc (O) và N thuộc (O′). Biết MB cắt (O′) tại điểm E nằm trong đường tròn (O) và đườngthẳng AB cắt MN tại I.

1. Chứng minh MAN + MBN = 1800 và I là trung điểm MN.2. Qua B, kẻ đường thẳng (d) song song với MN, (d) cắt (O) tại C và cắt (O′) tại

D (với C,D khác B). Gọi P,Q lần lượt là trung điềm của CD và EM. Chứng minh∆AME ∼ ∆ACD và các điểm A,B,P,Q cùng thuộc 1 đường tròn.

3. Chứng minh ∆BIP cân.

(Bà Rịa - Vũng Tàu)

Lời giải

Page 65: 1 ¾ F - · PDF fileđang chuẩn bị cho kì thi tuyển sinh vào các lớp 10 ... phẩm “Tuyển tập các bài toán trong đề thi ... đam mê với các môn học

THCM

N

2.3 Hình học phẳng 65

1. IMA= ABM,MIA= MIB⇒ MBN+MAN = ABM+ABN+MAN = IMA+ INA+MAN =180o. Mặt khác, ∆IMA∼ ∆IBM⇒ IM2 = IA.IB. Tương tự, IN2 = IA.IB. Do đó IM = INhay I là trung điểm MN (đpcm).

2.AEM = ADC (tứ giác AEBD nội tiếp)

AME = ACD

}⇒ ∆AME ∼ ∆ACD⇒

AEQ = ADCAEAD

=EMDC

=EQDP

⇒ ∆AEQ∼ ∆ADP⇒ AQE = APD. Vậy tứ giác ABPQ nội tiếp.3. Gọi K là giao điểm của CM và DN. Do CDNM là hình thang nên các điểm I,K,P thẳng

hàng.MN ‖ BC⇒ OM⊥BC⇒ ∆BMC cân tại M⇒ MCB = MBCDo MN ‖ BC nên MCB = KMN,MBC = BMN. Suy ra KMN = BMN.Chứng minh tương tự ta được KNM = BNM. Do đó ∆BMN = ∆KMN.Đồng thời: MB = MK,NB = NK nên MN là trung trực của KB⇒ KB⊥CD, IK = IB.∆KBP vuông tại B có IK = IB nên I là trung điểm KP.Vậy ∆BIP cân tại I.

Bài 6. Cho ∆ABC nhọn và H là trực tâm. Chứng minh

HABC

+HBCA

+HCAB≥√

3

(Bà Rịa - Vũng Tàu)

Lời giải

Bạn đọc tự vẽ hình.Gọi D,E,F lần lượt là các chân dường cao tương ứng kẻ từ các đỉnh A,B,C của ∆ABC. Đặtx = HA

BC ,y = HBCA ,z =

HCAB . Ta tìm đẳng thức liên hệ giữa x,y,z.

• ∆BHD ∼ ∆ACD ⇒ HBAC = BD

AD ⇒ xy = HABC .HB

CA = HA.BDBC.AD = SAHB

SABC. Tương tự, yz = SBHC

SABC,

zx = SCHASABC⇒ xy+ yz+ zx = SAHB+SBHC+SCHA

SABC= 1.

• (x+ y+ z)2 ≥ 3.(xy+ yz+ zx) nên (x+ y+ z)2 ≥ 3⇒ x+ y+ z≥√

3.Vậy HA

BC + HBCA + HC

AB ≥√

3.

Bài 7. Trên đường tròn (C) tâm O bán kính R vẽ dây cung AB < 2R. Từ A và B vẽ hai tiếptuyến Ax,By với (C). Lấy điểm M bất kì thuộc cung nhỏ AB (M khác A,B). Gọi H,K, I lầnlượt là chân các đường vuông góc hạ từ M xuống AB,Ax,By.

1. Chứng minh rằng MH2 = MK.MI.2. Gọi E là giao điểm của AM và KH,F là giao điểm của BM và HI. Chứng minh EF là

tiếp tuyến chung của hai đường tròn ngoại tiếp các tam giác MEK,MFI.3. Gọi D là giao điểm thứ hai của hai đường tròn ngoại tiếp các tam giác MEK,MFI.

Chứng minh rằng khi M di chuyển trên cung nhỏ AB thì đường thẳng DM luôn đi quamột điểm cố định.

(Bắc Ninh)

Lời giải

Page 66: 1 ¾ F - · PDF fileđang chuẩn bị cho kì thi tuyển sinh vào các lớp 10 ... phẩm “Tuyển tập các bài toán trong đề thi ... đam mê với các môn học

THCM

N

66 Chương 2. LỜI GIẢI

1. Gọi giao điểm hai tiếp tuyến Ax,By của (O) là T . Từ đây ta suy ra:

MAB = MBI. (2.3.7.1)

Mà MH⊥AB,MI⊥BT,MK⊥AT nên ta chứng minh được MIBH,MKAH là các tứ giácnội tiếp. Suy ra MKH = MAB và MBI = MHI. Kết hợp với (2.3.7.1) ta có:

MKH = MHI = MAB. (2.3.7.2)

Tương tự, ta có:MHK = MIH = MBA. (2.3.7.3)

Từ (2.3.7.2) và (2.3.7.3) ta suy ra: ∆MIH ∼ ∆MHK⇒ MHMK

=MIMH⇒MH2 = MI.MK

(đpcm).2. Ta có bổ đề sau:

Bổ đề 3. Cho ∆ABC nội tiếp (O) tâm O có ABC nhọn. Từ C kẻ tia Cx sao cho tia Cx nằmtrên nửa mặt phẳng bờ BC chứa A và ACx = ABC. Khi đó Cx chính là tiếp tuyến của (O).Chứng minh bổ đề khá dễ dàng và xin để dành cho bạn đọc.

Quay trở lại bài toán. Từ (2.3.7.2) và (2.3.7.3) suy ra:

MAB+ MBA = MHK + MHI = IHK = FHE

Mà ∆MBA có BMA+ BAM+ ABM = 180o nên dẫn đến: FHE+ FME = 180o⇒HFMEnội tiếp⇒ EFM = MHK = MIH = MIF (theo (2.3.7.3)).Áp dụng bổ đề, ta suy ra EF là tiếp tuyến đường tròn ngoại tiếp ∆FIM.Chứng minh tương tự với ∆MEK, ta suy ra đpcm.

3. S là giao điểm của OT và AB. Dễ thấy S cố định và là trung điểm đoạn AB. Ta chứng minhMD đi qua điểm S.

Page 67: 1 ¾ F - · PDF fileđang chuẩn bị cho kì thi tuyển sinh vào các lớp 10 ... phẩm “Tuyển tập các bài toán trong đề thi ... đam mê với các môn học

THCM

N

2.3 Hình học phẳng 67

• Gọi G là giao điểm của MD và EF , G′ là giao điểm đoạn EF và MS. Do EFM =

MBA(= MHK) nên EF ‖ AB (hai góc đồng vị). Áp dụng định lý Thales cho ∆MSBvà ∆MSA ta có:

G′FSB

=MG′

MS=

G′ESA

• Do S là trung điểm AB nên SA = SB. Do đó từ tỉ số trên ta suy ra G′F = G′E hay G′

là trung điểm EF .• Xét đường tròn ngoại tiếp ∆IMD có EF là tiếp tuyến nên GFM = FIM = FDG

(Tính chất góc tạo bởi tiếp tuyến và dây cung và tính chất góc nội tiếp). Kết hợp vớiFGM = FGD do góc chung dẫn đến:

∆FGM ∼ ∆DGF ⇒ GFGD

=GMGF⇒ GF2 = GM.GD. (2.3.7.4)

Tương tự:

GE2 = GM.GD. (2.3.7.5)

Từ (2.3.7.4) và (2.3.7.5) suy ra GE = GF hay G là trung điểm EF . Mà G′ cũng là trungđiểm EF nên ta suy ra G≡ G′. Dẫn đến theo cách vẽ, ta phải có MD≡MS. Vậy MD quaS cố định khi M di động.

Bài 8.1. Cho tứ giác ABCD nội tiếp đường tròn tâm O sao cho hai đường thẳng AD và BC cắt

nhau tại T . Đường thẳng (d) ⊥ OT tại T cắt AB và CD lần lượt tại M và N. Chứngminh T M = T N.

2. Cho xOy nhọn và M là một điểm cố định thuộc miền trong xOy. Đường thẳng d qua Mcắt các cạnh Ox,Oy lần lượt tại A,B không trùng với O. Xác định vị trí của A để ∆OABcó điện tích nhỏ nhất.

(Bình Định)

Lời giải

1. Gọi E là giao điểm của AC và BD. Ta xét hai trường hợp:TH1: AB ‖CD. Khi đó, do d ‖ AB ‖CD: nên d không thể giao với AB và CDTH2: AB giao CD tại F . Ta chứng minh bổ đề như sau:

Bổ đề 4. Cho tứ giác ABCD nội tiếp đường tròn tâm O sao cho hai đường thẳng AD vàBC cắt nhau tại T . Gọi E là giao điểm của AC và BD. Khi đó, EF⊥TO.

Page 68: 1 ¾ F - · PDF fileđang chuẩn bị cho kì thi tuyển sinh vào các lớp 10 ... phẩm “Tuyển tập các bài toán trong đề thi ... đam mê với các môn học

THCM

N

68 Chương 2. LỜI GIẢI

Thật vậy, gọi G là điểm nằm trên đường thẳng EF thỏa FA.GB = FE.FG = FD.FC. Tasuy ra EGBA và EGCD là các tứ giác nội tiếp. Dẫn đến: BGC = 360o− BGE−CGE =

180o− BGE +180o−CGE = BAE + BDC = BOC⇒ BGOC là tứ giác nội tiếp. Tươngtự ta có AGOD là tứ giác nội tiếp. Suy ra G là giao điểm đường tròn ngoại tiếp ∆AEB và∆DEC.Lấy G′ thuộc TO sao cho TA.T D = T G′.TO = T B.TC, dẫn đến G′ cũng là giao điểmđường tròn ngoại tiếp ∆AEB và ∆DEC. Suy ra G ≡ G′. Mà ta lại có AGT + FGA =

ADO+ ABD = 90o (AGOD,AEGB là tứ giác nội tiếp) nên FG⊥TO hay EF⊥TO. Bổ đềđược chứng minh.

Quay lại bài toán.

Gọi H là giao điểm của EF và AD. Do MN⊥OT áp dụng bổ đề ta suy ra GF ‖HF ‖MN.

Page 69: 1 ¾ F - · PDF fileđang chuẩn bị cho kì thi tuyển sinh vào các lớp 10 ... phẩm “Tuyển tập các bài toán trong đề thi ... đam mê với các môn học

THCM

N

2.3 Hình học phẳng 69

Khi đó áp dụng định lý Ta lét cho ∆TAN và ∆T MD, ta có:

T NT M

=T NHF

.HFT M

=TAAH

.HDDT

. (2.3.8.1)

Áp dụng định lý Ceva trong ∆AFD với DB,AC,HF đồng quy, ta có:

HAHD

.BFBA

.CDCF

= 1. (2.3.8.2)

Áp dụng định lý Menelaus trong ∆AFD với T,B,F thằng hàng, suy ra:

TAT D

.BFBA

.CDCF

= 1. (2.3.8.3)

Lấy (2.3.8.2) chia (2.3.8.3) theo vế ta suy ra

TAAH

.HDDT

= 1.

Kết hợp với (2.3.8.1) ta suy ra T M = T N.2. Kẻ MC ‖ Oy và MD ‖ Ox. (C ∈ Ox, D ∈ Oy). Đặt d(M;Ox),d(M;Oy) là khoảng cách từ

M tới Ox,Oy.

Ta có: SOAB = SMCOD +SMAC +SMDB = SMCOD +12.d(M;Ox).AC+

12.d(M;Oy).BD.

Do ∆MAC ∼ ∆MBD nên AC.DB = MC.MD.Áp dụng bất đẳng thức Cauchy cho 2 số, ta suy ra

SOAB ≥ SMCOD +2

√12.d(M;Ox).

12.d(M;Oy).AC.BD

≥ T = SMCOD +2

√12.d(M;Ox).

12.d(M;Oy).MC.MD

Page 70: 1 ¾ F - · PDF fileđang chuẩn bị cho kì thi tuyển sinh vào các lớp 10 ... phẩm “Tuyển tập các bài toán trong đề thi ... đam mê với các môn học

THCM

N

70 Chương 2. LỜI GIẢI

Do SMCOD,d(M;Ox),d(M;Oy) không đổi, C,D cố định nên T không thay đổi. Vậy giá trịnhỏ nhất của SOAB là T .Khi đó, SMAC = SMDB hay M là trung điểm AB. Theo định lý đường trung bình, lúc đó Alà điểm trên Ox sao cho C là trung điểm AO.

Nhận xét.• Việc chứng minh bổ đề giúp ta chứng minh được E là trực tâm ∆OT F và dẫn đến OE⊥T F .

Đây chính là kết quả của định lý Brocard và sau đây là một số bài toán liên quan tới định lý:

1. (VMO 2012) Trong mặt phẳng, cho tứ giác lồi ABCD nội tiếp đường tròn tâm O vàcó các cặp cạnh đối không song song. Gọi M,N tương ứng là giao điểm của cácđường thẳng AB và CD, AD và BC. Gọi P,Q,S,T tương ứng là giao điểm các đườngphân giác trong của các cặp MAN và MBN,MBN và MCN,MCN và MDN,MDNvà MAN. Giả sử bốn điểm P,Q,S,T đôi một phân biệt.(a) Chứng minh rằng bốn điểm P,Q,S,T cùng nằm trên một đường tròn. Gọi I là

tâm của đường tròn đó.(b) Gọi E là giao điểm của các đường chéo AC và BD. Chứng minh rằng ba điểm

E,O, I thẳng hàng.2. (PTNK 2015) Cho ∆ABC nhọn nội tiếp (O). Đường tròn (I) qua B và C lần lượt cắt

BA,CA tại E,F .(a) Giả sử các tia BF,CE cắt nhau tại D và T là tâm (AEF). Chứng minh OT ‖ ID.(b) Trên BF,CE lần lượt lấy các điểm G,H sao cho AG ⊥CE và AH ⊥ BF . Các

đường tròn (ABF),(ACE) cắt BC tại các điểm M,N (khác B và C) và cắt EFtại P,Q (khác E và F). Gọi K là giao điểm MP và NQ. Chứng minh DK ⊥ GH.

• Đây là bài toán khá khó cho học sinh cấp 2. Các bạn có ứng dụng tốt việc biến đổi Ceva,Menelaus và biết về mô hình định lý Brocard sẽ có thể giải quyết trong thời gian thi, nếukhông thì bài toán sẽ khó khăn để vừa làm vừa trình bày.

Bài 9.1. Từ một điểm S ở ngoài đường tròn tâm O kẻ các tiếp tuyến SA,SC và cát tuyến SBD (B

nằm giữa S và D). Gọi I là giao điểm của AC và BD. Chứng minh rằng:(a) AB.DC = AD.BC.(b) SB

SD = IBID = AB.CB

AD.CD .2. Cho nửa đường tròn tâm O, đường kính AB = 2R. Điểm M nằm trên nửa đường tròn

sao cho MAB = 60o. Kẻ MH⊥AB tại H, HE⊥AM tại E, HF⊥BM tại F . Các đườngthẳng EF và AB cắt nhau tại K. Tính diện tích tam giác MEF và độ dài các đoạn thẳngKA,KB theo R.

(Bình Định)

Lời giải

Page 71: 1 ¾ F - · PDF fileđang chuẩn bị cho kì thi tuyển sinh vào các lớp 10 ... phẩm “Tuyển tập các bài toán trong đề thi ... đam mê với các môn học

THCM

N

2.3 Hình học phẳng 71

1.(a) Ta có: SAB = ADS (Tính chất tiếp tuyến và dây cung) và ASB = ASD nên ∆ABS∼

∆DAS (g-g). Suy ra:

ABAD

=ASDS

. (2.3.9.1)

Tương tự, ta có:

BCCD

=CSDS

. (2.3.9.2)

Từ (2.3.9.1) và (2.3.9.2) kết hợp với AS =CS (tính chất tiếp tuyến), suy raABAD

=

BCCD⇒ AB.CD = AD.BC. (đpcm)

(b) Gọi H là giao điểm AC và SO. Khi đó H là trung điểm AC và AC⊥SO tại H. Ta cóSB.SD = SH.SO (= SA2) nên BHOD là tứ giác nội tiếp⇒ DHO = DBO = ODB =

BHS⇒ DHO = BHS⇒ 90O− DHO = 90O− BHS⇒ DHA = AHB . Do đó HI là

phân giác trong góc H của ∆DHB và HS là phân giác ngoài⇒ SBSD

=IBID

. Chứng

minh theo các cặp tam giác đồng dạng, suy raIBID

=IBIA

.IAID

=BCDA

.ABCD

, do đóSBSD

=IBID

=AB.CBAD.CD

.

2. Ta có:MEMB

=MFMA

(ME.MA = MB.MF = MH2) và M chung nên ∆MEF ∼ ∆MBA theo

tỉ số k =EFAB

=MHAB

=

√3

4. Mà SMAB =

MH.AB2

=

√3.R2

.2R =√

3.R2 ⇒ SMEF =

SMAB.k2 =3.√

3R2

16.

Page 72: 1 ¾ F - · PDF fileđang chuẩn bị cho kì thi tuyển sinh vào các lớp 10 ... phẩm “Tuyển tập các bài toán trong đề thi ... đam mê với các môn học

THCM

N

72 Chương 2. LỜI GIẢI

Ta có: AE = AM−ME = R− 34.R =

R4

. Mà ∆KAE cân (KEA = 30o; KAE = 1202)⇒

KA = AE =R4⇒ KB =

9R4

.

Bài 10. 4ABC nội tiếp đường tròn (O),AB < AC. Phân giác trong góc BAC cắt (O) tại Dkhác A. Trên tia AB lấy M tuỳ ý sao cho đường tròn ngoại tiếp4ADM cắt AC tại N khác A,C.

1. Chứng minh4BDM =4CDN.2. Khi BN không song song với MC, đường trung trực của đoạn BN cắt đường trung trực

của đoạn MC tại P. Chứng minh A,C,P,M cùng thuộc một đường tròn.3. Xác định vị trí tâm I của đường tròn ngoại tiếp4ADM để độ dài đoạn thẳng MN nhỏ

nhất.

(Cần Thơ)

Lời giải

1. ∆ABC nội tiếp (O) có AD cắt (O) tại D nên DB = DC. Đồng thời ABDC,AMDN là tứ giácnội tiếp nên MBD = NCD và BMD = DNC⇒ BDM = NDC. Kết hợp với MBD = NCD,ta suy ra ∆BDM = ∆CDN.

2. Từ câu 1. suy ra BM =CN, mà theo cách vẽ ta có BP = NP và PN = PC nên ∆PBM =

∆PNC⇒ PCN = PMB hay PCA = PMA. Do đó tứ giác APCM nội tiếp dẫn đến đpcm.

Page 73: 1 ¾ F - · PDF fileđang chuẩn bị cho kì thi tuyển sinh vào các lớp 10 ... phẩm “Tuyển tập các bài toán trong đề thi ... đam mê với các môn học

THCM

N

2.3 Hình học phẳng 73

3. Kẻ đường kính ME của đường tròn ngoại tiếp ∆ADM. Khi đó MN = sin(MEN).ME =

sin(BAC).ME. Do đó MN nhỏ nhất khi ME nhỏ nhất. Mà ME = 2.ID = IA+ ID≥ ADnên ME nhỏ nhất khi I ∈ AD.Vậy khi I thuộc đường thẳng AD thì MN nhỏ nhất.

Bài 11. Cho tam giác ABC có BAC > 90o,AB < AC và nội tiếp đường tròn tâm O. Trungtuyến AM của tam giác ABC cắt (O) tại điểm thứ hai D. Tiếp tuyến của (O) tại D cắt đườngthẳng BC tại S. Trên cung nhỏ DC của (O) lấy điểm E, đường thẳng SE cắt (O) tại điểm thứhai là F . Gọi P,Q lần lượt là giao điểm của các đường thẳng AE,AF với BC.

1. Chứng minh rằng MODS là tứ giác nội tiếp.2. Chứng minh rằng QB = PC.

(Đà Nẵng)

Lời giải

1. Vì (O) có M là trung điểm dây cung BC nên OMS = 90o. Mà (O) có DS là tiếp tuyến nênODS = 90o. Ta suy ra OMS+ ODS = 180o hay MODS là tứ giác nội tiếp.

Page 74: 1 ¾ F - · PDF fileđang chuẩn bị cho kì thi tuyển sinh vào các lớp 10 ... phẩm “Tuyển tập các bài toán trong đề thi ... đam mê với các môn học

THCM

N

74 Chương 2. LỜI GIẢI

2. Lấy N là trung điểm EF . Khi đó chứng minh được N thuộc đường tròn ngoại tiếp tứ giácMODS nên END = SND = SMD = AMQ. Đồng thời, theo tính chất góc chắn cung trong(O) ta có: QAM = NED. Từ đó suy ra

∆QAM ∼ ∆DEN⇒ QMND

=AMNE⇒ QM =

AM.NDNE

. (2.3.11.1)

Chứng minh tương tự, ta được:

PM =AM.ND

NF. (2.3.11.2)

Từ (2.3.11.1) và (2.3.11.2), kết hợp với NE = NF , ta suy ra: QM = MP⇒ BM−QB =CM−PC⇒ QB = PC (do BM = MC).

Nhận xét. Bài toán trên có mô hình khá tương đồng với bài toán trong đề thi chọn đội tuyểnPTNK 2014: Cho ∆ABC không cân và I là trung điểm BC. Đường tròn (I) tâm I và qua đỉnh Acắt AB,AC lần lượt tại M,N. MI,NI lần lượt cắt (I) tại P,Q. Gọi K là giao điểm của PQ và tiếptuyến (I) tại A. Chứng minh K thuộc đường thẳng BC.

Bài 12. Từ điểm M nằm ngoài đường tròn (ω) tâm O, vẽ đến (ω) hai tiếp tuyến MA,MB vàcát tuyến MCD (C nằm giữa M và D).Gọi H là giao điểm MO và AB.

1. Chứng minh MA2 = MC.MD.2. Chứng minh tứ giác CDOH nội tiếp.3. Chứng minh đường thẳng AB và hai tiếp tuyến của (ω) tại C,D đồng quy.4. Đường thẳng CH cắt (ω) tại điểm thứ hai E khác C. Chứng minh AB//DE.

(Đồng Nai)

Lời giải

1. Ta có:MAC = MDAAMC = AMD

}⇒ ∆MAC ∼ ∆MDA⇒ MA

MD=

MCMA⇒MA2 = MC.MD.

2. MA,MB là tiếp tuyến của (ω) nên AH⊥MO và MA2 = MC.MD. Do đó, AH là đường cao∆AMO nên theo hệ thức lượng, ta có AM2 = MH.MO.Mà MA2 = MC.MD nên MC.MD = MH.MO. Do đó tứ giác CDOH nội tiếp.

Page 75: 1 ¾ F - · PDF fileđang chuẩn bị cho kì thi tuyển sinh vào các lớp 10 ... phẩm “Tuyển tập các bài toán trong đề thi ... đam mê với các môn học

THCM

N

2.3 Hình học phẳng 75

3. Tiếp tuyến tại C và D giao nhau tại T và S là giao của TO và CD. Theo hệ thức lượngtrong ∆TOC, ta suy ra

CO2 = OS.OT (2.3.12.1)

Mặt khác, theo hệ thức lượng trong ∆AMO, ta có:

AO2 = OH.OM .(2.3.12.2)

Từ (2.3.12.1) và (2.3.12.2), kết hợp với AO = OC, suy ra: OS.OT = OH.OM⇒ OSOM

=

OHOT

. Mà MOS = TOH ⇒ ∆T HO ∼ ∆MSO⇒ T HO = MSO = 90o hay T H⊥MO. Mặt

khác, AH⊥MO nên T,A,H thẳng hàng. Dẫn đến đường thẳng AB và hai tiếp tuyến của(ω) tại C,D đồng quy.

4. Dễ thấy MO⊥AB. Nên AB ‖ DE sẽ tương đương với chứng minh MO⊥DE. Theo hệ thứclượng trong ∆MAO, ta có:

AH2 = AH.HB = MH.HO (2.3.12.3)

Mà tứ giác ACBE có AB giao CE tại H nên

AH.HB =CH.HE (2.3.12.4)

Từ (2.3.12.3),(2.3.12.4) suy ra MH.HO =CH.HE, dẫn đến tứ giác MCOE nội tiếp, suyra

MOE = MCH. (2.3.12.5)

Page 76: 1 ¾ F - · PDF fileđang chuẩn bị cho kì thi tuyển sinh vào các lớp 10 ... phẩm “Tuyển tập các bài toán trong đề thi ... đam mê với các môn học

THCM

N

76 Chương 2. LỜI GIẢI

Từ kết quả câu 2., ta suy raMOD = MCH (2.3.12.6)

Từ (2.3.12.5),(2.3.12.6)⇒ MOD = MOE. Kết hợp với OD = OE và MO chung ta suyra MD = ME. Dẫn đến MO là trung trực đoạn CE nên MO⊥DE, và ta có đpcm.

Nhận xét. Từ câu 4 của bài hình ta liên hệ với bài hình sau:

Từ điểm M nằm ngoài đường tròn (ω) tâm O, vẽ đến (ω) hai tiếp tuyến MA,MB và cát tuyếnMCD (C nằm giữa M và D). Gọi E và F lần lượt là điểm đối xứng với C vả D qua trung trựcAB. Chứng minh M,E,F thẳng hàng.

Bài 13. 4ABC có bán kính đường tròn nội tiếp r và độ dài các đường cao là x,y,z.

1. Chứng minh1x+

1y+

1z=

1r

.

2. Biết r = 1 và x,y,z là các số nguyên dương. Chứng minh4ABC đều.

(Đồng Nai)

Lời giải

1. Bạn đọc tự vẽ hình.Gọi S là diện tích ∆ABC. Không mất tính tổng quát, ta coi các đường cao có độ dài x,y,zsẽ tương ứng vuông góc với các cạnh BC,CA,AB. Khi đó:

2.S = x.BC = y.CA = z.AB. (∗)

Trong đẳng thức đề bài, ta nhận thấy các đại lượng đều liên quan tới S, do đó ta nhân Svào hai vế đẳng thức để chia các đường cao.

1x+

1y+

1z=

1r⇔ S

x+

Sy+

Sz=

Sr=

AB+CB+CA2

. (2.3.13.1)

Đồng thời, ta chứng minh được đẳng thức quen thuộc: S =AB+BC+CA

2.r. Từ đây ta

suy ra (2.3.13.1) đúng và ta có đpcm.2. Với r = 1, ta có:

1x+

1y+

1z= 1. (2.3.13.2)

Để giải, ta tìm nghiệm nguyên dương (x,y,z) của phương trình (2.3.13.2) có thể tạo thành3 đường cao tam giác.

Ta có nhận xét sau: với x,y,z như đề bài,1x,1y,1z

là bộ độ dài 3 cạnh một tam giác nào đó.

Ta chứng minh được bằng kết hợp sử dụng bất đẳng thức các cạnh ∆ABC và (*).Không mất tình tổng quát, ta giả sử x≥ y≥ z. Kết hợp với (2.3.13.2) ta suy ra 1≤ 3

z ⇔z≤ 3.Nếu một trong 3 số x,y,z bằng 1 thì hai số còn lại phải bằng 0, không thỏa mãn. Do đó2≤ z≤ 3:

Page 77: 1 ¾ F - · PDF fileđang chuẩn bị cho kì thi tuyển sinh vào các lớp 10 ... phẩm “Tuyển tập các bài toán trong đề thi ... đam mê với các môn học

THCM

N

2.3 Hình học phẳng 77

• Nếu z = 2 thì từ (2.3.13.2) ta có:1x+

1y=

12

. Do1x≤ 1

ynên 1

2 ≤2y

hay y≤ 4. Do

y ≥ z = 2 nên y ∈ {2;3;4}. Thế vào đẳng thức trên để tính ta thu được (x,y,z) ∈{(4,2,2);(6,3,2)}. Do

12+

12=

14

và13+

16=

12

, nên mỗi bộ trong 2 bộ trên không

thể là 3 cạnh một tam giác, vô lý theo nhận xét ở trên.

• Với z = 3 thì ta có:1x+

1y=

23

. Làm tương tự như trường hợp z = 2, ta thu được bộ

nghiệm duy nhất thỏa: x = y = z = 3. Kết hợp với (*) ta suy ra AB = BC =CA tứctam giác ∆ABC đều (đpcm).

Bài 14. Cho tam giác nhọn ABC có AB < AC và nội tiếp đường tròn (O). Các đường caoBB′,CC′ cắt nhau tại điểm H. Gọi M là trung điểm BC. Tia MH cắt đường tròn (O) tại điểmP.

1. Chứng minh ∆BPC′ ∼ ∆CPB′.2. Các đường phân giác của các góc BPC′,CPB′ lần lượt cắt AB,AC tại E,F . Gọi O′ là

tâm đường tròn ngoại tiếp ∆AEF ; K là giao điểm của HM và AO′.(a) Chứng minh tứ giác PEKF nội tiếp.(b) Chứng minh các tiếp tuyến tại E và F của đường tròn (O′) cắt nhau tại 1 điểm

nằm trên đường tròn (O).

(Hà Nội)

Lời giải

1. Cho AO cắt (O) tại D thì AD là đường kính của (O)⇒ ABD = ACD = 90o⇒ BD//CHvà CD//BH⇒ Tứ giác HDBC là hình bình hành⇒ DH cắt BC tại trung điểm của BC vàcũng là trung điểm của DH⇒D,M,N,P thẳng hàng⇒ APD = 90o hay APH = 90o⇒ Pthuộc đường tròn đường kính AH. Lại có AB′H = AC′H = 90o⇒ B′,C′ thuộc đường trònđường kình AH. Vậy A,P,B′,C′,H cùng thuộc đường tròn đường kính AH.

Page 78: 1 ¾ F - · PDF fileđang chuẩn bị cho kì thi tuyển sinh vào các lớp 10 ... phẩm “Tuyển tập các bài toán trong đề thi ... đam mê với các môn học

THCM

N

78 Chương 2. LỜI GIẢI

⇒ AB′P = AC′PPC′B+ PC′A = 180o

PB′C+ PB′A = 180o

⇒ PC′B = PB′C

Xét đường tròn (O) : PCB′ = PBC′ (Góc cùng chắn cung AP). Mà PC′B = PB′C ⇒∆PB′C ∼ ∆PC′B (g-g) (đpcm).

2. (a) PE,PF lần lượt là phân giác của BPC′ và CPB′⇒

{EPC′ = 1

2 BPC′

FPB′ = 12CPB′

. Mà BPC′ =

CPB′ do ∆EPC′ ∼ ∆CPB′⇒ EPC′ = FPB′.Xét ∆PEC′ và ∆PFB′ có:

EPC′ = FPB′ (cmt)PC′E = PB′F (∆BPC′ ∼ ∆CPB′)

}⇒ ∆PEC′ ∼ ∆PFB′ (g-g)⇒ PEC′ = PFB′ hay

PEA = PFA⇒ Tứ giác APEF nội tiếp. Mà O′ là tâm ngoại tiếp ∆AEF nên là tâmngoại tiếp tứ giác APEF .Gọi K′ là giao của AO′ và (O′) thì AK′ là đường kính⇒ APK′ = 90o⇒ K′ ∈ PDvì APD = 90o⇒ K′ là giao của PD và AO′ hay K′ là giao của MH và AO′⇒ K ≡K′⇒ K ∈ (O), hay tứ giác PEKF nội tiếp.

(b) Lấy D′ là trung điểm cungBC không chứa A. Chú ý rằng do PE,PF lần lượt là phân

giác của BPC′ và CPB′; và do ∆PEC′ ∼ ∆PFB′ (g-g) nênEC′

FB′=

PC′

PB′=

C′BB′C

. Khi

đó, ta sẽ chứng minh được HE,HF lần lượt là phân giác C′HB, B′HC⇒ ∆AEF cântại A. Từ đó, ta chứng minh được A,O′,K,D′ thẳng hàng. Từ đây, bài toán tươngđương với chứng minh D′E,D′F là tiếp tuyến của (O′) tại E,F .Kéo dài CC′ cắt (O) tại C1. Ta chứng minh được C1 đối xứng với C qua AB. Để từđó, ta suy ra E là tâm đường tròn nội tiếp ∆BC1H nên C1E là phân giác BC1C. MàCD′ là phân giác BC1C nên C1,E,D′ thẳng hàng.

Do EK và C1H cùng vuông góc AB nên C1H ‖ EK ⇒ D′EK = D′C1H =BAC

2=

EAK⇒ DE là tiếp tuyến của (O′). Tương tự với DF , ta có đpcm.

Nhận xét. Một bài toán khá hay và khó, đòi hỏi ta phải linh hoạt trong việc vẽ thêm và sử dụngcác kiến thức liên quan tới trực tâm, đường tròn và tiếp tuyến. Sau đây là một số bài toán với môhình có điểm tương đồng.

1. (T2/THPT - THTT 440) Cho ∆ABC có H là trực tâm và M là trung điểm BC. P là mộtđiểm thuộc đường thẳng HM, đường tròn (K) đường kính AP cắt CA,AB lần lượt tạiE,F 6= A. Chứng minh tiếp tuyến tại E,F của (K) cắt nhau trên trung trực BC.

2. (Lạng Sơn 2016) Cho4ABC nhọn nội tiếp (O). Các đường cao AD,BE,CF cắt nhau tạiH (D ∈ BC,E ∈CA,F ∈ AB). Gọi M là trung điểm của BC. Hai đường tròn (DEF) và(HBC) cắt nhau tại X và Y .

(a) Chứng minh AX = AY .(b) Gọi R là trung điểm của XY . AR cắt HM tại S. Chứng minh tứ giác HDSR nội tiếp.

Page 79: 1 ¾ F - · PDF fileđang chuẩn bị cho kì thi tuyển sinh vào các lớp 10 ... phẩm “Tuyển tập các bài toán trong đề thi ... đam mê với các môn học

THCM

N

2.3 Hình học phẳng 79

Bài 15. Cho tam giác đều ABC nội tiếp đường tròn tâm O. Điểm E thay đổi trên cung nhỏAB (E khác A và B). Từ B và C lần lượt kẻ các tiếp tuyến với đường tròn (O), các tiếp tuyếnnày cắt đường thẳng AE theo thứ tự tại M và N. Gọi F là giao điểm của BN và CM.

1. Chứng minh rằng MB.CN = BC2.2. Khi điểm E thay đổi trên cung nhỏ AB. Chứng minh rằng đường thẳng EF luôn đi qua

một điểm cố định.

(Hà Tĩnh)

Lời giải

1. E thuộc cung nhỏ AB nên MN giao BC tại 1 điểm gọi là T . MBA = 90o− ABO = 60o =

BAC⇒MB ‖ AC. Tương tự, ta cũng có NM ‖ AB. Từ hai điều trên, áp dụng định lý Thales

trong ∆TAC và ∆T NC, ta có

MBAC

=T BTC

ABNC

=T BTC

⇒ MBAC

ABNC⇒MB.NC = AB.AC = BC2(∆ABC

đều).2. Gọi D là giao điểm hai tiếp tuyến tại B và C. MBA = ABC = ACB = ACN = 60o nên

MBC = BCN. Mà từ câu 1., ta cóMBBC

=BCCN

, nên ∆MBC∼ ∆BCN. Khi đó BMC = NCB,

và từ đó chứng minh được BFC = 120o. Mà BDC = 60o nên BFCD là tứ giác nội tiếp.Ta cũng có MEB = ABC = 180o−120o = 180o− BFC = BFM nên tứ giác MEFB nộitiếp, tương tự NEFC nội tiếp.EF giao (O) tại G. Khi đó BGE = EBM = EMF = GFC (tứ giácMEFB nội tiếp) ⇒BG ‖ FC. Tương tự ta có GC ‖ GB. Vậy EF đi qua trung điểm BC cố định.

Page 80: 1 ¾ F - · PDF fileđang chuẩn bị cho kì thi tuyển sinh vào các lớp 10 ... phẩm “Tuyển tập các bài toán trong đề thi ... đam mê với các môn học

THCM

N

80 Chương 2. LỜI GIẢI

Bài 16. Cho đường tròn tâm O đường kính BC, A là điểm di động trên đường tròn (O) (Akhác B và C). Kẻ AH⊥BC tại H. M là điểm đối xứng của điểm A qua điểm B.

1. Chứng minh điểm M luôn nằm trên một đường tròn cố định.2. Đường thẳng MH cắt (O) tại E và F (E nằm giữa M và F). Gọi I là trung điểm HC,

đường thẳng AI cắt (O) tại G (G 6= A). Chứng minh AF2+FG2+GE2+EA2 = 2.BC2.3. Kẻ HP⊥AB tại P. Tìm vị trí điểm A sao cho bán kính đường tròn ngoại tiếp tam giác

BPC đạt giá trị lớn nhất.

(Hải Dương)

Lời giải

1. Lấy K là điểm đối xứng của O qua B, vì B và O cố định nên K cố định. Vì OAKM là hình

bình hành nên KM = OA, mà OA =BC2

không đổi⇒M nằm trên đường tròn tâm K, bán

kínhBC2

.

2. Xét ∆AHB và ∆CHA có BHC = CHA= 90o, BAH = ACB (cùng phụ với ABC)⇒∆AHB∼∆CHA.Gọi S là trung điểm AH, I là trung điểm HC nên ∆ABS ∼ ∆CAI ⇒ ABS = CAI. Ta lạicó BS là đường trung bình ∆AMH ⇒ BS ‖ MH ⇒ ABS = AMH ⇒ AMH = CAI. MàCAI + MAI = 90o⇒ AMH + MAI = 90o⇒ AI⊥MF .Xét tứ giác AEGF nội tiếp (O) có AG⊥EF . Kẻ đường kính AD. Do GD⊥AG và EF⊥AGnên EF ‖ GD, do đó tứ giác nội tiếp EFGD là hình thang cân ⇒ FG = ED⇒ AE2 +FG2 = AE2 +ED2 = AD2 = BC2. Tương tự ta chứng minh được AF2 +EG2 = BC2. VậyAF2 +FG2 +GE2 +EA2 = 2.BC2.

Page 81: 1 ¾ F - · PDF fileđang chuẩn bị cho kì thi tuyển sinh vào các lớp 10 ... phẩm “Tuyển tập các bài toán trong đề thi ... đam mê với các môn học

THCM

N

2.3 Hình học phẳng 81

3. Gọi Q là hình chiếu của H lên AC⇒ APHQ là hình chữ nhật có tâm S⇒ AQP = AHP =ABC nên tứ giác BPQC nội tiếp.Đường trung trực của các đoạn PQ,BC,QC cắt nhau tại O′ thì O′ là tâm đường tròn ngoạitiếp ∆BCP. Ta có: OO′ ‖ AH và cùng vuông góc với BC.

OA⊥PQ và O′S⊥PQ⇒ O′S ‖ OA nên ASO′O là hình bình hành ⇒ OO′ = AS =AH2

.

Trong trường hợp A nằm chính giữa cung BC thì ta vẫn có: OO′ = AS =AH2

.

∆OO′C vuông tại O nên O′C =

√OC2 +

AH2

4. Do OC không đổi nên O′C nhỏ nhất khi

AH lớn nhất⇔ A chính giữa cung BC.

Bài 17. Cho ∆ABC nhọn nội tiếp đường tròn tâm O có AB < AC. Các đường cao BD,CE cắtnhau tại H (D thuộc AC, E thuộc AB). Gọi M là trung điểm của BC, tia MH cắt đường tròn(O) tại N.

1. Chứng minh rằng năm điểm A,D,H,E,N cùng thuộc 1 đường tròn.2. Lấy điểm P trên đoạn BC sao cho BHP = CHM, Q là hình chiếu vuông góc của A trên

đường thẳng HP. Chứng minh rằng tứ giác DENQ là hình thang cân.3. Chứng minh rằng đường tròn ngoại tiếp tam giác MPQ tiếp xúc với đường tròn (O).

(Hải Phòng)

Lời giải

1. Cho AO cắt (O) tại S thì AS là đường kính của (O)⇒ ABS = ACS = 90o ⇒ BD ‖ CSvà CE ‖ BS⇒ Tứ giác HSBC là hình bình hành ⇒ SH cắt BC tại trung điểm M củaBC⇒N,H,M,S thẳng hàng⇒ ANS = 90o hay ANH = 90o⇒N thuộc đường tròn đườngkính AH hay A,D,H,E,N cùng thuộc 1 đường tròn.

Page 82: 1 ¾ F - · PDF fileđang chuẩn bị cho kì thi tuyển sinh vào các lớp 10 ... phẩm “Tuyển tập các bài toán trong đề thi ... đam mê với các môn học

THCM

N

82 Chương 2. LỜI GIẢI

2. Dễ thấy Q cũng thuộc đường tròn đường kính AH. Theo tính chất góc đối đỉnh, ta có:BHP = QHD và MHC = NHE. Kết hợp với giả thiết, do đó đường tròn (O′) đường kínhAH có hai dây cung EN và QD chắn hai góc chắn cung bằng nhau nên NQ ‖ ED. Từ đósuy ra DENQ là hình thang cân.

3. Ta chứng minh đường tròn (MPQ) ngoại tiếp ∆MPQ tiếp xúc (O) tại N. Gọi T là giaođiểm của AH với BC. Khi đó: QPM = HPM = 90o− PHT = 90o− AHQ = 90o− ANQ =

QNH = QNM. Do đó (MPQ) đi qua N.• EAO = BAO = 90o− ACB = 90o− AED⇒ AO⊥ED.• Gọi F là giao của AS và đường tròn đường kính AH, NQ giao (O) tại G. Từ câu

2, ta có NQ ‖ ED mà AO⊥ED nên NG⊥AF . Từ đó suy ra AS là trung trực NG⇒MSF = ASN = AGN = ANQ = AFQ⇒ Q,J,M thẳng hàng⇒ HF ‖ NG.• Kẻ tiếp tuyến Nx của (O) sao cho Nx nằm trên nửa mặt phẳng bờ NG chứa A.

Ta chứng minh được xNG = GSN = ANG+ ASN = AFQ+ ASN = MFS+ ASN =

NMQ = xNQ. Suy ra Nx là tiếp tuyến của (MNQ) nên (MPQ) tiếp xúc (O).

Bài 18.1. Cho ∆ABC nhọn có các đường cao AA1,BB1,CC1. Gọi K là hình chiếu của A trên A1B1;

L là hình chiếu của B lên B1C1. Chứng minh rằng: A1K = B1L.2. Cho tứ giác nội tiếp ABCD có AC cắt BD tại E. Tia AD cắt tia BC tại F . Dựng hình

bình hành AEBG.(a) Chứng minh FD.FG = FB.FE(b) Gọi H là điểm đối xứng của E qua AD. Chứng minh 4 điểm F,H,A,G cùng thuộc

một đường tròn.

(Tp. HCM)

Lời giải

1. Ta chứng minh được LBB1 = BAA1 (= BB1A1) và AA1B = BLB1 = 90o nên

∆LBB1 ∼ ∆A1AB⇒ LBAA1

=BB1

AB⇒ KA1 =

BB1.AA1

AB. (2.3.18.1)

Đồng thời, ta chứng minh tương tự được ∆KAA1 ∼ ∆B1AB nên ta có:

KA1 =BB1.AA1

AB. (2.3.18.2)

Từ (2.3.18.1) và (2.3.18.2) ta suy ra; KA1 = B1L.

Page 83: 1 ¾ F - · PDF fileđang chuẩn bị cho kì thi tuyển sinh vào các lớp 10 ... phẩm “Tuyển tập các bài toán trong đề thi ... đam mê với các môn học

THCM

N

2.3 Hình học phẳng 83

2. (a) Tứ giác nội tiếp ABCD có AC giao BD tại E nên suy ra:

∆FBA∼ ∆FDC∆ABE ∼ ∆DCE

}⇒

FBFD

=ABCD

ABCD

=EAED

⇒ FBFD

=EAED

=GBED

(AGBE là hình bình hành).

Mặt khác ADE = ACF = GBF (GB ‖ EA) nên suy ra:

∆FED∼ ∆FGB⇒ FEFG

=FDFB⇒ FD.FG = FB.FE (đpcm)

(b) Gọi G′ là điểm đối xứng của G qua AF . Khi đó, hai tứ giác HAGF và EAG′F đốixứng với nhau qua đường thẳng AF . Do đó, chứng minh F,H,A,G cùng thuộc 1đường tròn tương đương với chứng minh tứ giác AEFG′ nội tiếp.

Mà tương tự với chứng minh ∆FED∼ ∆FGB , ta cũng chứng minh được: ∆AFG∼∆CFE⇒ CEF = AGF = AG′F .Khi đó, do CEF bù với góc AEF nên AG′F bù với góc AEF . Suy ra AEFG′ là tứgiác nội tiếp.Vậy ta có đpcm.

Page 84: 1 ¾ F - · PDF fileđang chuẩn bị cho kì thi tuyển sinh vào các lớp 10 ... phẩm “Tuyển tập các bài toán trong đề thi ... đam mê với các môn học

THCM

N

84 Chương 2. LỜI GIẢI

Bài 19. Cho hai đường tròn (O) và (O′) cắt nhau tại A,B. Tiếp tuyến chung gần B của haiđường tròn lần lượt tiếp xúc (O),(O′) tại C và D. Qua A kẻ đường thẳng song song CD lầnlượt cắt (O),(O′) tại M,N. Các đường thẳng CM,DN cắt nhau tại E. Gọi P là giao điểm củaBC với MN, Q là giao điểm của BD và MN. Chứng minh:

1. AE ⊥CD.2.

BDBO

+BCBP

=MNPO

.3. 4EPQ cân.

(Hưng Yên)

Lời giải

1. AE giao CD tại F . Khi đó ta chứng minh được APDF là hình chữ nhật nên DF = AP =

PN⇒ FDAN

=12

. Áp dụng định lý Thales cho ∆EAN với FD ‖ AN, ta có:

DEEN

=DFAN

=12⇒ DN

EN=

12=

NPAN

.

Áp đụng định lý Thales đảo, suy ra PD ‖ AE. Do đó AE⊥CD.2. Do CD ‖MN và 2.CD = MN nên áp dụng định lý Thales ta suy ra:

BDBQ

+BCBP

=2.BDBQ

=2.CDPQ

=MNPQ

.

3. Gọi I là giao điểm AB và CD. Bằng tam giác đồng dạng, ta suy ra được: IC = ID =√IA.IB⇒ AP = AQ (theo định lý Thales). Và do AE⊥MN (AE⊥CD), nên suy ra ∆EQP

cân tại E.

Page 85: 1 ¾ F - · PDF fileđang chuẩn bị cho kì thi tuyển sinh vào các lớp 10 ... phẩm “Tuyển tập các bài toán trong đề thi ... đam mê với các môn học

THCM

N

2.3 Hình học phẳng 85

Bài 20. Cho hai đường tròn (O) và (O′) cắt nhau tại A,B. Từ điểm E nằm trên tia đối của tiaAB kẻ đến (O′) các tiếp tuyến EC,ED (C và D là các tiếp điểm phân biệt). Các đường thẳngAC,AD theo thứ tự cắt (O) lần lượt tại P,Q khác A. Chứng minh:

1. 4BCP∼4BDQ.2. CA.DQ =CP.DA.3. Ba điểm C,D và trung điểm I của PQ thẳng hàng.

(Khánh Hoà)

Lời giải

1. Ta có: AQB = CPB (góc chắn cung AB) và PCB = ADB (góc ngoài tại C của tứ giác nộitiếp ACBD) nên suy ra4BCP∼4BDQ.

2. Chứng minh các cặp ∆EAD∼ ∆EDB và ∆ECA∼ ∆EBD, ta chứng minh được tỉ số sau:ADAC = BD

BC . Mà theo câu a, ta suy ra QDPC = BD

BC nên suy ra DADQ = CA

CP ⇒CA.DQ =CP.DA.3. Giả sử CD giao với PQ tại T , ta cần chứng minh T P = T Q. Xét ∆QAP có T,C,D thẳng

hàng. Theo định lý Menelaus, ta có

T QT P

.DADQ

.CPCA

= 1.

Kết hợp với kết quả câu 2, ta có: T Q = T P⇒ T ≡ I. Từ đó ta suy ra đpcm.

Page 86: 1 ¾ F - · PDF fileđang chuẩn bị cho kì thi tuyển sinh vào các lớp 10 ... phẩm “Tuyển tập các bài toán trong đề thi ... đam mê với các môn học

THCM

N

86 Chương 2. LỜI GIẢI

Bài 21. Cho ∆ABC (AB < AC) nội tiếp đường tròn (O). Gọi H là chân đường cao kẻ từ Ađến BC. Gọi P,Q lần lượt là chân đường cao kẻ từ H đến AB,AC. Hai đường thẳng PQ và BCcắt nhau tại M, đường thẳng MA cắt đường tròn (O) tại K với K 6= A. Gọi I là tâm đường trònngoại tiếp ∆BCP.

1. Chứng minh các tứ giác APHQ,BPQC nội tiếp;2. Chứng minh MP.MQ = MB.MC và MB.MC = MK.MA;3. Chứng minh AKPQ là tứ giác nội tiếp;4. Chứng minh I,H,K thẳng hàng.

(Lào Cai)

Lời giải

1. Do APH = AQH = 90o⇒ APH + AQH = 180o nên APHQ là tứ giác nội tiếp.Do AHPQ nội tiếp nên APQ = AHQ, mà AHQ+ QHC = 90o và QHC+ QCH = 90o nênAHQ = QCH hay APQ = QCB. Vậy BPQC là tứ giác nội tiếp.

2. Xét ∆MPB và ∆MCQ có

{MPB = MCQPMB = CMQ.

nên ∆MPB ∼ ∆MCQ nên MPMC = MB

MQ hay

MP.MQ = MC.MB. Hoàn toàn tương tự ta có: MB.MC = MK.MA.3. Từ ý 2 ta có MK.MA = MP.MQ nên MK

MP = MQMA . Hai tam giác ∆MKP và ∆MQA lại có góc

M chung nên đồng dạng, suy ra MKP = MQA nên tứ giác KPQA nội tiếp.4. Do P,Q nằm trên đường tròn đường kính AH và KPQA là tứ giác nội tiếp nên K cũng

thuộc đường tròn đường kính AH, suy ra

AKH = 90o⇒ KH⊥AK. (2.3.21.1)

Page 87: 1 ¾ F - · PDF fileđang chuẩn bị cho kì thi tuyển sinh vào các lớp 10 ... phẩm “Tuyển tập các bài toán trong đề thi ... đam mê với các môn học

THCM

N

2.3 Hình học phẳng 87

Gọi J là trung điểm AH. Vẽ đường kính AD của đường tròn (O). Ta có AJ⊥BC. Vì OI làtrung trực của BC nên OI⊥BC. Suy ra AJ ‖ OI.Vì IJ là trung trực của PQ nên

IJ⊥PQ. (2.3.21.2)

Mặt khác,

{AQP = ABC (tứ giác BPQC nội tiếp).

ABC = ADC (hai góc nội tiếp chắn cung_AC).

⇒ OAQ+AQP= OAQ+ADC =

90o, tức làAO⊥PQ. (2.3.21.3)

Từ (2.3.21.2) và (2.3.21.3) ta có AO ‖ IJ. Mặt khác, OI ‖ AJ (⊥BC) nên AJIO là hìnhbình hành, nên AJ = IO suy ra JH = OI. Do đó JHIO là hình bình hàng nên HI ‖ OJ.Vì OJ là trung trực của AK nên OJ⊥AK, suy ra:

HI⊥AK (2.3.21.4).

Từ (2.3.21.3) và (2.3.21.4) ta có: I,H,K thẳng hàng.

Bài 22. Cho ∆ABC nhọn có đường cao BE và nội tiếp (O). Tiếp tuyến của (O) tại B,C của(O) cắt nhau tại S,BC và OS cắt nhau tại M. Chứng minh:

1. AB.BM = AE.BS2. AME = ASB

(Long An)

Lời giải

1.

{AEB = BMS = 90O (Tính chất hai tiếp tuyến cắt nhau).

BAE = SBM (cùng bằng 12

_BC)

⇒ ∆ABE ∼ ∆BSM⇒ ABBS =

AEBM hay AB.BM = AE.BS (đpcm).

Page 88: 1 ¾ F - · PDF fileđang chuẩn bị cho kì thi tuyển sinh vào các lớp 10 ... phẩm “Tuyển tập các bài toán trong đề thi ... đam mê với các môn học

THCM

N

88 Chương 2. LỜI GIẢI

2. Vì ABBS = AE

BM ;MB = ME = BC2 nên

ABBS

=AEME

. (2.3.22.1)

Vì ∆ABE ∼ ∆BSM nên ABE = BSM = OBM. Suy ra ABO = EBM = BEM. Mà ABS =

90O + ABO; AEM = 90o + BEM.

⇒ ABS = AEM. (2.3.22.2)

Từ (2.3.22.1) và (2.3.22.2) suy ra ∆ABS∼ ∆AEM⇒ AME = ASB (đpcm).

Bài 23. Cho tứ giác ABCD có BAD = 60o; BCD = 90o. Đường phân giác trong của BAD cắtBD tại E. Đường phân giác trong của BCD cắt BD tại F . Chứng minh:

√3

AE+

√2

CF=

1AB

+1

BC+

1CD

+1

DA

(Long An)

Lời giải

Gọi K là hình chiếu vuông góc của E lên AB. Với ∆ABC bất kì, đặt sABC là diện tích tam giác đó.Khi đó:

SABE = KE.AB2 = AE.sin(30o).AB

2 = AE.AB4

SADE = AE.AD4

SABD = AB.sin(60o).AD2 =

√3.AB.AD

4

Page 89: 1 ¾ F - · PDF fileđang chuẩn bị cho kì thi tuyển sinh vào các lớp 10 ... phẩm “Tuyển tập các bài toán trong đề thi ... đam mê với các môn học

THCM

N

2.3 Hình học phẳng 89

Mà SABD = SABE +SADE nên √3

AE=

1AB

+1

DA. (2.3.23.1)

Tương tự ta cũng tìm được √2

CF=

1BC

+1

CD. (2.3.23.2)

Từ (2.3.23.1) và (2.3.23.2) ta có:√

3AE +

√2

CF = 1AB + 1

BC + 1CD + 1

DA .

Bài 24. Cho ∆ABC nhọn, nội tiếp đường tròn (O). Các đường cao AK,BM,CN của ∆ABCcắt nhau tại H.

1. Chứng minh: NKH = MKH2. Đường thẳng MN cắt đường tròn (O) tại hai điểm I,J. Chứng minh AO đi qua trung

điểm của IJ.3. Gọi P là trung điểm BC, diện tích tứ giác AMHN là S. Chứng minh 2.OP2 > S.

(Nam Định)

Lời giải

1. Ta có: AK,BM,CN là ba đường cao giao nhau tại H nên BKNH,CKHM là các tứ giác nộitiếp.

{NKH = NBHHKM = HCM

. Mà NBH = HCM (cùng phụ với BAC), nên suy ra NKH = MKH.

2. Ta có BMC = BNC = 90o, nên BNMC là tứ giác nội tiếp. Suy ra ANJ = ANM = ACB =

AOB2

= 90O− BAO = 90o− NAO. Từ đó, ta chứng minh được AO⊥MN hay AO⊥IJ. Mà

IJ là dây cung của (O) nên AO đi qua trung trực IJ.3. Do ∆ABC có H là trực tâm và P là trung điểm BC nên ta chứng minh được tính chất quen

thuộc: AH = 2.OP.

Ta có: S = SANH +SAHM =AN.NH +AM.MH

2.

Page 90: 1 ¾ F - · PDF fileđang chuẩn bị cho kì thi tuyển sinh vào các lớp 10 ... phẩm “Tuyển tập các bài toán trong đề thi ... đam mê với các môn học

THCM

N

90 Chương 2. LỜI GIẢI

Áp dụng bất đẳng thức AM - GM, ta có:AN.NH ≤ AN2 +NH2

2=

AH2

2

AM.MH ≤ AM2 +MH2

2=

AH2

2

⇒ S ≤

AH2

2=

(2.OP)2

2= 2.OP2. Dấu -"xảy ra khi AN = NH và AM = MH, khi đó ∆ABC cân

tại A, vô lý. Vậy 2.OP2 > S.

Bài 25. Cho đường tròn (O), bán kính R, dây BC cố định khác đường kính. A là một điểm diđộng trên cung lớn BC sao cho4ABC nhọn. Các đường cao BE,CF của4ABC cắt nhau tạiH.

1. Chứng minh tứ giác BECF nội tiếp và AO⊥ EF .2. Tia EF cắt (O) tại I, tia AO cắt (O) tại G. Gọi M là trung điểm BC,D là giao điểm hai

đường thẳng AH và BC. Chứng minh AI2 = 2AD.OM.3. Trong trường hợp4ABC cân tại A, goi x là khoảng cách từ (O) đến BC. Tìm x để chu

vi4ABC lớn nhất.

(Ninh Bình)

Lời giải

1. Do BFC = BEC = 90o nên BCEF là tứ giác nội tiếp.Cho AO giao EF tại N. Do BCEF là tứ giác nội tiếp nên:

AFE = ACB. (2.3.25.1)

Mà ∆ABC nhọn có tâm ngoại tiếp là O nên:

ACB = 90o− BAO = 90o− FAN. (2.3.25.2)

Từ (2.3.25.1) và (2.3.25.2), ta suy ra: AFE + FAN = 90o, nên ∆FAN vuông tại N nênAO⊥EF tại N.

Page 91: 1 ¾ F - · PDF fileđang chuẩn bị cho kì thi tuyển sinh vào các lớp 10 ... phẩm “Tuyển tập các bài toán trong đề thi ... đam mê với các môn học

THCM

N

2.3 Hình học phẳng 91

2. Gọi giao của AO và (O) là G. Khi đó, ta có: ACB = AGB.Kết hợp với (2.3.25.1), ta suy ra: AGB = AFE = AFN. Do đó FNGB là tứ giác nội tiếpcó FB giao NG tại A và:

AN.AG = AF.AB = AH.AD

Đồng thời, ta chứng minh được tính chất sau: AH = 2.OM. Do đó:

AN.AG = 2MO.AD (2.3.25.3)

Ta có AG là đường kính (O) nên AIG = 90o. Dẫn đến ∆AIG vuông tại I có IN là đườngcao (AO⊥EF tại N), nên kết hợp với (2.3.25.3), suy ra:

AI2 = AN.AG = 2.MO.AD.

3. Lúc này, ta chứng minh được AD là đường cao đi qua trunng điểm D của BC. Ta tính đượcBC = 2.

√R2− x2 và AB = AC =

√2Rx+R2. Khi đó, chu vi ∆ABC là:

P = AB+AC+BC = 2[√

R2− x2 +√

2Rx+R2]

Vì các giá trị có căn, ta áp dụng bất đẳng thức Cosi để khử căn, khử x, và tìm ra max củaP. Ta tìm hằng số a dương và áp dụng như sau:

P = 2.√

a.(R+ x)√

R− x.1√a+2.

√2Rx+R2.

Áp dụng bất đẳng thức Cosi, ta có:

2.√

a.(R+ x)√

R− x.1√a≤ a.(R+ x)+R− x√

a=

R(a+1)+(a−1).x√a

Dấu ’=’ xảy ra khi:√

a.(R+ x) =√

R− x⇒ x =R(1−a)

a+1⇒ x+R =

2Ra+1

.

Ta nhận thấy 0 < a < 1. Với biểu thức T = 2.√

2Rx+R2 = 2.√

2R.(x+R), ta tìm cáchnhân hệ số theo a sao cho dấu ’=’ phía trên vẫn bảo toàn. Dựa vào đó, ta nhân và chia biểuthức cho

√a+1 như sau:

T = 2.

√2R

a+1.√(x+R).

√a+1≤ (

2Ra+1

+ x+R).√

a+1

Do đó:

P≤ R(a+1)+(a−1).x√a

+

(2R

a+1+ x+R

).√

a+1≤ x.(

a−1√a

+√

a+1)+M.

Với M =R(a+1)√

a+(

2Ra+1

+R).√

a+1. Do đó, ta tìm a để khử x.

⇒ a−1√a

+√

a+1 = 0⇒ a =13

(do 0 < a < 1).

Vậy giá trị lớn nhất của chu vi P là M = 3√

3.R khi x =R(1−a)

a+1=

R2

.

Page 92: 1 ¾ F - · PDF fileđang chuẩn bị cho kì thi tuyển sinh vào các lớp 10 ... phẩm “Tuyển tập các bài toán trong đề thi ... đam mê với các môn học

THCM

N

92 Chương 2. LỜI GIẢI

Bài 26. Cho đường tròn (O;R) và dây cung BC cố định. Gọi A là điểm di động trên cung lớnBC sao cho ∆ABC nhọn. Bên ngoài ∆ABC dựng các hình vuông ABDE,ACFG và hình bìnhhành AEKG.

1. Chứng minh rằng AK = BC và AK⊥BC.2. DC cắt BF tại M. Chứng minh rằng A,K,M thẳng hàng.3. Chứng minh rằng khi A thay đổi trên cung lớn BC của (O;R) thì K luôn thuộc một

đường tròn cố định.

(Phú Thọ)

Lời giải

1. Ta dễ dàng chứng minh được KG = AB (= AE) và AG = AC. Mà bằng cộng góc, ta cũngchứng minh được: KGA = 180o− EAG = BAC.

⇒ ∆KAG = ∆BCA⇒ AK = BC và KAG = ACBKhi đó, kẻ tia đối tia AK là Ax, ta chứng minh được: xAC+ ACB = 90o⇒ AK⊥BC.

2. Theo câu 1. ta suy ra AK = BC và KAB = ABC + 90o = DBC. Mà AB = BD nên suyra ∆KAB = ∆CBD. Từ đây, ta chứng minh được DC⊥KB hay CD là đường cao ∆KBC.Tương tự, ta có BF là đường cao ∆KBC. Mà BF giao CD tại M, nên M là trực tâm ∆KBChay KM⊥BC. Mà KA⊥BC nên K,A,M thẳng hàng (đpcm).

3. Vẽ hình bình hành AKO′O. Khi đó, ta chứng minh được O′ cố định do O cố định, OO′⊥BCvà OO′ = BC. Dẫn đến chứng minh được K thuộc đường tròn (O′;R) cố định nên ta cóđpcm.

Page 93: 1 ¾ F - · PDF fileđang chuẩn bị cho kì thi tuyển sinh vào các lớp 10 ... phẩm “Tuyển tập các bài toán trong đề thi ... đam mê với các môn học

THCM

N

2.3 Hình học phẳng 93

Bài 27. 4ABC nhọn (AB < AC) nội tiếp đường tròn (O). Phân giác của góc BAC cắt BC tạiD, cắt (O) tại E. Gọi M là giao điểm của AB,CE. Tiếp tuyến tại C của (O) cắt AD tại N, tiếptuyến tại E của (O) cắt CN tại F .

1. Chứng minh tứ giác MACN nội tiếp.2. Gọi K là điểm trên cạnh AC sao cho AB = AK. Chứng minh AO⊥ DK.

3. Chứng minh1

CF=

1CN

+1

CD.

(Quảng Bình)

Lời giải

1. Theo tính chất tiếp tuyến dây cung và tính chất phân giác, ta có: MCN = ECN = EAC =

MAE = MAN. Vậy nên tứ giác MACN nội tiếp (đpcm).2. Vì AB = AK nên ∆BAK cân tại A. Khi đó AD là phân giác BAK nên AD là đường trung

trực đoạn BK. Suy ra: ABD = AKD. Kết hợp với BC =COA

2= 90o− OCA = 90o− OCK,

ta sẽ chứng minh được nếu S là giao của AO và DK thì ASD = 90o.Ta suy ra AO⊥DK (đpcm).

Page 94: 1 ¾ F - · PDF fileđang chuẩn bị cho kì thi tuyển sinh vào các lớp 10 ... phẩm “Tuyển tập các bài toán trong đề thi ... đam mê với các môn học

THCM

N

94 Chương 2. LỜI GIẢI

3. Từ câu 1., ta suy raECN = MAE = BCE. (2.3.25.1)

Do đó EC là phân giác ∆DCN. Mà tiếp tuyến tại E và C giao nhau tại F nên EF = FC.Từ đó chứng minh được:

ECN = CEF . (2.3.25.2)

Từ (2.3.25.1) và (2.3.25.2) ta suy ra BCE = CEF Hay EF ‖DC. Áp dụng định lý Thalescho ∆DCN, suy ra:

CFCN

= 1− NFNC

= 1− EFDC

= 1− CFCD⇒ 1 =

CFCN

+CFCD⇒ 1

CF=

1CN

+1

CD.

Bài 28. Cho hình bình hành ABCD có góc A tù và AB = AC, gọi H là hình chiếu của C lênAB. Trên cạnh AB lấy E sao cho H là trung điểm BE. Gọi F là điểm đối xứng với D qua E,Glà điểm đối xứng với A qua B. Chứng minh:

1. EC là tia phân giác của DEB.2. 4CFG cân.

(Quảng Nam)

Lời giải

1. Do HC⊥EB và H là trung điểm EB nên ∆EBC cân hay EBC = BEC. Đồng thời, CA=CDnên ∆ADC cân tại C hay DAC = ADC. Từ đó, ta chứng minh được: ABC = BEC = ADC =

DAC. Do đó AECD là từ giác nội tiếp nên suy ra EDC = DAC = BEC, dẫn đến EC làphân giác DEB (đpcm).

2. Tứ giác AECD nội tiếp có AE ‖ DC nên AECD là hình thang cân, do đó AC = DE. Màtheo cách vẽ của F và G, ta suy ra EF = DE = AC = AB = BG.Đồng thời, do DEC = DCE = CEB = CBE ⇒ 180o− DEC = 180o− CBE ⇒ FEC =

GBCTừ đó, xét trong ∆EFC và ∆BGC có EF = BG, EC = BC, và FEC = GBC, ta suy ra∆EFC = ∆BGC⇒ FC =CG. Do đó ∆CFG cân tại C (đpcm).

Page 95: 1 ¾ F - · PDF fileđang chuẩn bị cho kì thi tuyển sinh vào các lớp 10 ... phẩm “Tuyển tập các bài toán trong đề thi ... đam mê với các môn học

THCM

N

2.3 Hình học phẳng 95

Bài 29. Cho đường tròn (O) đường kính AB, dây CD vuông góc với AB tại H nằm giữa O vàA. Lấy điểm E bất kì trên cung nhỏ BD, gọi M là hình chiếu của B lên CE.

1. Chứng minh rằng HM//AE.2. Đường tròn ngoại tiếp4DEM đi qua trung điểm N của dây AE trong (O).

(Quảng Nam)

Lời giải

1. Theo giả thiết, ta có CMB = CHB = 90o⇒CHMB là tứ giác nội tiếp. Suy ra:

BHM = BCM = BCE. (2.3.29.1)

Mà ACBE nội tiếp nên BCE = BAE. Kết hợp với (2.3.29.1) và do H ∈ AB, ta suy raBHM = BAE sẽ dẫn đến HM ‖ AE (đpcm).

2. Từ kết quả câu 1., ta suy ra: CMH = CEA = ADC. Mà dây CD vuông góc AB nênCA =CD, suy ra ADC = ACD = AED. Kết hợp lại, ta suy ra:

CMH = AED. (2.3.29.2)

Đồng thời, ta cũng có CEA = CDA (cùng chắn cung AC). Nên kết hợp với (2.3.29.1), ta

suy ra: ∆ADE ∼ ∆CHM (g-g)⇒ ADCH

=AECM

. Mà CD = 2CH mà AE = 2AN nên dẫn đếnADCD

=ANCM

. Kết hợp với điều kiện DAN = DCM (Cùng chắn cung DE), suy ra:

∆ADN ∼ ∆CDM⇒ ADN = CDM⇒ ADC = NDM.

Mà ADC = AEC = NEM nên suy ra NDM = NEM⇒ DNME là tứ giác nội tiếp. Ta suyra đpcm.

Page 96: 1 ¾ F - · PDF fileđang chuẩn bị cho kì thi tuyển sinh vào các lớp 10 ... phẩm “Tuyển tập các bài toán trong đề thi ... đam mê với các môn học

THCM

N

96 Chương 2. LỜI GIẢI

Bài 30. Cho ∆ABC vuông tại A có đường cao AH (H thuộc cạnh BC). Cho BH = 2 vàCH = m. Xác định m để đường thẳng BC tiếp xúc với đường tròn tâm A bán kính R = 4. Khiđó tính độ dài các đoạn AB và AC.

(Tây Ninh)

Lời giải

Bạn đọc tự vẽ hình Để BC tiếp xúc với đường tròn tâm A bán kính R = 4, thì khoảng cách tứ Atới BC phải bằng 4. Ta suy ra AH = 4.Khi đó, áp dụng hệ thức lượng trong tam giác vuông, ta suy ra:

AH2 = BH.CH⇒ m =CH =AH2

BH=

42

2= 8.

Áp dụng hai lần định lý Pytago, ta suy ra:{AB =

√BH2 +AH2 =

√22 +42 = 2.

√5

AC =√

CH2 +AH2 =√

82 +42 = 4.√

5

Bài 31. Cho ∆ABC cân tại A và nội tiếp (O) tâm O. Gọi M là một điểm bất kì trên cung nhỏAC của (O) (M khác A và C). Trên tia BM lấy điểm E sao cho ME = MC (E ở ngoài đoạnBM). Chứng minh rằng đường tròn tâm A bán kính AE luôn đi qua B và C.

(Tây Ninh)

Lời giải

Page 97: 1 ¾ F - · PDF fileđang chuẩn bị cho kì thi tuyển sinh vào các lớp 10 ... phẩm “Tuyển tập các bài toán trong đề thi ... đam mê với các môn học

THCM

N

2.3 Hình học phẳng 97

Theo đề bài, ta suy ra MABC là tứ giác nội tiếp nên AMC+ ABC = 180o và AMB = ACB.Mà ABC = ACB (∆ABC cân tại A)⇒ AMC = 180o− AMB = AME.Từ đây, kết hợp với ME = MC và AM chung ta suy ra ∆AMC = ∆AME ⇒ AE = AC = AB(∆ABC cân tại A). Vậy đường tròn tâm A bán kính AE luôn đi qua B và C.

Bài 32. Từ một điểm M nằm ngoài (O) kẻ hai tiếp tuyến MA,MB với (O) (A,B là các tiếpđiểm). Gọi C là giao điểm của MO và AB, lấy D thuộc đoạn AC (D khác A,C). Đường thẳngMD cắt (O) tại E,F (ME < MF). Chứng minh rằng:

1. MA2 = ME.MF .2. E,C,O,F cùng thuộc 1 đường tròn.

(Tây Ninh)

Hướng dẫn

1. Chứng ∆MAF ∼ ∆MEA và dùng tỉ lệ đồng dạng để suy ra MA2 = ME.MF .2. Chứng minh ME.MF = MA2 = MC.MO, dẫn đến ∆MEC ∼ ∆MOF . Từ đó suy ra ECOF

là tứ giác nội tiếp.

Bài 33. Từ một điểm I nằm ngoài đường tròn (O), vẽ các tiếp tuyến IA, IB (A,B là các tiếpđiểm) và cát tuyến ICD không qua tâm O của (O) (C nằm giữa I và D).

1. Chứng minh AC.BD = AD.BC.2. Gọi K là giao điểm của CD và AB,E là trung điểm OI. Chứng minh KA.KB = OE2−

EK2.3. Gọi H là trung điểm AB. Chứng minh ADH = IDB.

(Thái Bình)

Lời giải

Page 98: 1 ¾ F - · PDF fileđang chuẩn bị cho kì thi tuyển sinh vào các lớp 10 ... phẩm “Tuyển tập các bài toán trong đề thi ... đam mê với các môn học

THCM

N

98 Chương 2. LỜI GIẢI

1. Ta có: IAC = ADI (Tính chất tiếp tuyến và dây cung) và AIC = AID nên ∆ACI ∼ ∆DAI(g-g). Suy ra:

ACAD

=AIDI

(2.3.33.1)

Tương tự, ta có:

BCBD

=BIDI

(2.3.33.2)

Từ (2.3.33.1) và (2.3.33.2) kết hợp với AI = BI (tính chất tiếp tuyến), suy raACAD

=BCBD⇒

AC.BD = AD.BC (đpcm)2. Không mất tính tổng quát, giả sử cát tuyến ICD đi qua đoạn AB⇒ K ∈ AB. Xét (O) có

H là giao của AB và OI nên H là trung điểm AB và AB⊥OI tại H, suy ra: KA.KB =(AH−KH).(BH +KH) = AH2−KH2 = (OA2−OH2)− (OK2−OH2) = OA2−OK2.Để ý rằng OE2−EK2 = OE2− (KH2 +HE2) = OE2− [KH2 +(OE−OH)2] = OE2−[KH2 +OE2 +OH2−2.OE.OH] = OH.OI− (KH2 +OH2), mà OH.OI = OA2 (hệ thứclượng ∆OAI) và KH2+OH2 = OK2 (Định lý Pythagoras)⇒OE2−EK2 = OA2−OK2 =KA.KB.

3. Ta có: IC.ID = IH.OH (= IA2), dẫn đến CHOD là tứ giác nội tiếp⇒ CDH = COI. MàADI = IAC, nên suy ra ADH = ADI+CDH = IAC+COI = BAI−CAB+AOI−AOC. MàAOB = AOI + BOI = BAI + AOI, CAB = IDB, COB = 2IDB⇒ ADH = AOB− AOC−IDB = COB− IDB = IDB.

Bài 34. Cho đường tròn tâm O và dây cung AB. Từ một điểm M bất kì trên đường tròn(M 6= A,B), kẻ MH⊥AB tại H. Gọi E,F lần lượt là hình chiếu vuông góc với H trên MA,MB.Qua M kẻ đường thẳng vuông góc với EF , cắt dây cung AB tại D. Chứng minh MA2

MB2 =AHBD .

ADBH .

(Thái Nguyên)

Page 99: 1 ¾ F - · PDF fileđang chuẩn bị cho kì thi tuyển sinh vào các lớp 10 ... phẩm “Tuyển tập các bài toán trong đề thi ... đam mê với các môn học

THCM

N

2.3 Hình học phẳng 99

Lời giải

Gọi E ′,F ′ lần lượt là hình chiếu vuông góc của D trên MA,MB.

Ta có:AHBD

=SMAH

SMBD=

HE.MADF ′.MB

;ADBH

=SMAD

SMBH=

DE ′.MAHF.MB

⇒ ADBD

.AHBH

=MA2

MB2 .HE.DE ′

DF ′.HF. (2.3.34.1)

Ta có: EHF = E ′DF ′ (vì cùng bù với AMB)Ta lại có: HFE = DMF ′ (Hai góc có cạnh tương ứng vuông góc). Mà DMF ′ = DE ′F ′ (cùngchắn cung DF ′).⇒ HFE = DE ′F ′⇒ ∆HEF ∼ ∆DF ′E ′

⇒ HEHF

=DF ′

DE ′⇒ HE.DE ′

HF.DF ′= 1. (2.3.34.2)

Từ (2.3.34.1) và (2.3.34.2) ta được:MA2

MB2 =AHBD

.ADBH

.

Bài 35. Cho ∆ABC vuông tại A, đường cao AH. Gọi (O) là đường tròn ngoại tiếp ∆AHC.Trên cung nhỏ AH của (O) lấy điểm M bất kì khác A và H. Trên tiếp tuyến tại M của (O) lấyhai điểm D,E sao cho BD = BE = BA. Đường thẳng BM cắt (O) tại điểm thứ hai N. Chứngminh rằng:

1. Tứ giác BDNE nội tiếp.2. Đường tròn ngoại tiếp tứ giác BDNE và đường tròn (O) tiếp xúc nhau.

(Thái Nguyên)

Lời giải

Page 100: 1 ¾ F - · PDF fileđang chuẩn bị cho kì thi tuyển sinh vào các lớp 10 ... phẩm “Tuyển tập các bài toán trong đề thi ... đam mê với các môn học

THCM

N

100 Chương 2. LỜI GIẢI

1. Vì AH⊥BC⇒ AHC = 90o⇒ AC là đường kính đường tròn (O). Mà ∆ABC vuông tại Anên AB là tiếp tuyến của (O).Xét ∆BAM và ∆BNA có: ABN chung và BAM = BNA vì có số đo bằng nửa số đo cung_

AM . Vậy ∆BAM ∼ ∆BNA⇒ BMBA

=BABN⇒ BM.BN = BA2. Theo giả thiết BA = BD⇒

BM.BN = BD2 ⇒ BMBD

=BDBN

. Mặt khác DBM là góc chung của ∆BDM và ∆BND⇒

∆BDM ∼ ∆BND⇒ BDM = BND.Ta có: BD = BE ⇒ ∆BDE cân tại B⇒ BDM = BED⇒ BND = BED⇒ BDNE là tứgiác nội tiếp.

2. Kí hiệu (O′) là đường tròn ngoại tiếp tứ giác BDNE. Giả sử (O) và (O′) không tiếp xúcnhau. Vì (O) và (O′) có điểm chung N nên chúng có điểm chung thứ hai N′ và BN′ cắtDE tại M′ 6= M.

Các tam giác ∆BDM′ và ∆BN′D có:

{DBM′ = DBN′

BN′D = BDE(= BED)⇒ ∆BDM′ ∼ ∆BN′D⇒

BM′

BD=

BDBN′⇒ BM′.BN′ = BD2.

Ta có: BM.BN = BD2 nên BM′.BN′ = BM.BN⇒ BMBN′

=BM′

BN. Xét ∆BMM′ và ∆BN′N có

NBN′ chung vàBMBN′

=BM′

BNnên chúng đồng dạng⇒ BMM′= BN′N⇒ BN′N+M′MN =

180o⇒M′MNN′ là tứ giác nội tiếp.Đường tròn ngoại tiếp M′MNN′ và (O) có chung 3 điểm M,N,N′ nên chúng trùng nhau⇒M′ là điểm chung thứ hai của (O) và tiếp tuyến DE. Điều này vô lý nên điều giả sử sai.Vậy đường tròn ngoại tiếp tứ giác BDNE và đường tròn (O) tiếp xúc nhau.

Page 101: 1 ¾ F - · PDF fileđang chuẩn bị cho kì thi tuyển sinh vào các lớp 10 ... phẩm “Tuyển tập các bài toán trong đề thi ... đam mê với các môn học

THCM

N

2.3 Hình học phẳng 101

Bài 36. Cho hình bình hành ABCD với BAD < 90o. Tia phân giác góc BCD cắt đường trònngoại tiếp ∆BCD tại O (O khác C). Kẻ đường thẳng (d) đi qua A và vuông góc với CO. Đườngthẳng (d) cắt các đường thẳng CB,CD lần lượt tại M,N.

1. Chứng minh rằng: OBM = ODC2. Chứng minh ∆OBM = ∆ODC và O là tâm đường tròn ngoại tiếp ∆CMN.3. Gọi K là giao điểm của OC và BD; I là tâm đường tròn ngoại tiếp ∆BCD. Chứng minh

rằng: NDMB = IB2−IK2

KD2 .

(Thanh Hóa)

Lời giải

1. Do cùng bù với OBC nênOBM = ODC (2.3.36.1)

2. Do CO là phân giác BCD nênBO = DO. (2.3.36.2)

AB ‖ CN ⇒ N = BAM, mà ∆CMN có CO vừa là đường cao và đường phân giác nên∆CMN cân tại C⇒ BMA = N

⇒ BMA = BAM. (2.3.36.3)

Từ (2.3.36.11),(2.3.36.2),(2.3.36.3)⇒ ∆OBM = ∆ODC (c-g-c). Suy ra

OM = OC (2.3.36.4)

Vì CO là trung trực MN nênOM = ON. (2.3.36.5)

Từ (2.3.36.4) và (2.3.36.5)⇒ O là tâm đường tròn ngoại tiếp ∆CMN.

Page 102: 1 ¾ F - · PDF fileđang chuẩn bị cho kì thi tuyển sinh vào các lớp 10 ... phẩm “Tuyển tập các bài toán trong đề thi ... đam mê với các môn học

THCM

N

102 Chương 2. LỜI GIẢI

3. Gọi H là hình chiếu của I lên BD⇒ H là trung điểm BD.Ta có: KD2 = (DH−HK)2 = DH2 +HK2−2DH.HK = (ID2−HI2)+(IK2− IH2)−2.DH(DH −KD) = ID2 + IK2 + 2DH.KD− 2(IH2 +DH2) = ID2 + IK2 +BD.DK −2.ID2 = IK2− ID2 +BD.KD⇒ ID2− IK2 = BD.DK−KD2. Mà IB = ID nên

IB2− IK2

KD2 =ID2− IK2

KD2 =BD.KD−KD2

KD2 =BD−DK

DK=

BKDK

(2.3.36.6)

Mặt khác, CK là phân giác ∆CBD nên

BKDK

=CBCD

. (2.3.36.7)

Do CM =CN và MB =CD nên

CMMB

=CNCD⇔ CM−MB

MB=

CN−CDCD

⇔ CBMB

=NDCD⇔ CB

CD=

NDMB

(2.3.36.8)

Từ (2.3.36.6),(2.3.36.7),(2.3.36.8) ta có: NDMB = IB2−IK2

KD2 (đpcm).

Bài 37. Cho hai đường tròn (O1) và (O2) có bán kính khác nhau, cắt nhau tại A,B saocho O1,O2 thuộc hai nửa mặt phẳng có bờ là đường thẳng AB. Đường tròn (O) ngoại tiếp4BO1O2 cắt (O1),(O2) lần lượt tại K,L khác A và B. Đường thẳng AO cắt (O1),(O2) lầnlượt tại M,N khác A. Hai đường thẳng MK,NL cắt nhau tại P sao cho P,B thuộc hai nửa mặtphẳng có bờ là đường thẳng KL. Chứng minh:

1. Tứ giác BKPL nội tiếp.2. Điểm A cách đều hai đường thẳng BK,BL.3. Điểm P thuộc đường thẳng AB khi và chỉ khi4PKL cân.

(Thừa Thiên - Huế)

Lời giải

Page 103: 1 ¾ F - · PDF fileđang chuẩn bị cho kì thi tuyển sinh vào các lớp 10 ... phẩm “Tuyển tập các bài toán trong đề thi ... đam mê với các môn học

THCM

N

2.3 Hình học phẳng 103

1. Tứ giác KABM nội tiếp nên MKB = MAB. Tứ giác ABLN nội tiếp nên MAB = BNL. Kếthợp lại, suy ra MKB = BNL. Do đó tứ giác BKPL có góc trong tại L bằng góc ngoài tại Knên nó nội tiếp. Ta có đpcm.

2. Dễ thấy rằng A cách đều hai đường thẳng BK,BL khi và chỉ khi ta chứng minh được BA làphân giác của KBL.Từ câu 1., ta suy ra P thuộc đường tròn ngoại tiếp ∆O1BO2. Suy ra MPB = KPB =

180o− KO1B = 180o−2.PMB = 180o−(PMB+ PBM). Suy ra PMB = PBM hay ∆PBMcân tại P nên PM = PB. Tương tự, ta có ∆PBN cân tại P nên PB = PN. Dẫn đến PM = PNhay ∆PMN cân tại P nên:

PMN = PNM (2.3.37.1).

Mà PMN = KMA = KBA và PNM = ABL, nên kết hợp với (2.3.37.1), ta suy ra KBA =

ABL nên BA là phân giác của KBL. Ta suy ra đpcm.3. Bài toán tương đương với chứng minh mệnh đề sau:

P ∈ AB⇔ ∆PKL cân.

⇒ Giả sử P ∈ AB. Khi đó do ∆PBM cân tại P nên dễ chứng minh được ∆PKA cân tại Phay PK = PA. Tương tự ta cũng chứng minh được PA = PL⇒ PK = PL⇒ ∆PKLcân tại P.

⇐ Giả sử ∆PKL cân. Khi đó do tính chất góc chắn cung trong (O) nên suy ra PKL =

PLK = PBL = PBK. Từ đó suy ra BP và BA cùng là phân giác KBL nên K ∈ AB.Từ đây ta suy ra đpcm.

Bài 38. 4ABC nhọn (AB < AC) nội tiếp đường tròn (O),M là trung điểm BC. AM cắt (O)tại D khác A. Đường tròn ngoại tiếp4MDC cắt đường thẳng AC tại E khác C. Đường trònngoại tiếp4MDB cắt đường thẳng AB tại F khác B.

1. Chứng minh4BDF ∼4CDE và E,M,F thẳng hàng.2. Chứng minh OA⊥ EF .3. Phân giác góc BAC cắt EF tại N. Phân giác góc CEN và BFN lần lượt cắt CN,BN tại

P,Q. Chứng minh rằng PQ//BC.

(Vĩnh Phúc)

Lời giải

1. Do các tứ giác MECD,MBFD nội tiếp nên

DEC = DMC = DFB. (2.3.38.1)

Tứ giác ABDC nội tiếp nên

DCE = DCA = DBF . (2.3.38.2)

Từ (2.3.38.1) và (2.3.38.2) suy ra ∆BDF ∼ ∆CDE (g-g)⇒ EDC = BDF . Mà EMC =

EDC; BMF = BDF ⇒ EMC = BMF . Vậy E,M,F thẳng hàng.

Page 104: 1 ¾ F - · PDF fileđang chuẩn bị cho kì thi tuyển sinh vào các lớp 10 ... phẩm “Tuyển tập các bài toán trong đề thi ... đam mê với các môn học

THCM

N

104 Chương 2. LỜI GIẢI

2. Do MECD,MBFD là hai tứ giác nội tiếp nên AB.AF = AM.AD = AE.AC, suy ra tứ giácBECF nội tiếp. Do đó AFE = ACB.Vẽ tiếp tuyến Ax của (O) thì ACB = BAx. Do đó BAx = AFE, suy ra Ax ‖ EF . VậyOA⊥EF .

3. Ta có ∆BDF ∼ ∆CDE nênSBDF

SCDE=

BF2

CE2 .

1 =MBMC

=SDAB

SDAC=

SDAB

SBDF.SBDF

SCDE.SCDE

SDAC=

ABBF

.BF2

CE2 .CEAC

=AB.BFCE.AC

. Từ đó:

BFCE

=ACAB

=AFAE

=NFNE⇒ EN

EC=

FNFB

. (2.3.38.3)

Theo tính chất phân giác ta có:

PNPC

=ENEC

;QNQB

=FNFB

. (2.3.38.4)

Từ (2.3.38.3) và (2.3.38.4) suy raPNPC

=QNQB

. Do đó PQ ‖ BC.

2.4 Số học

Bài 1.1. Với x,y là các số nguyên dương thỏa mãn x2−1

2 = y2−13 .

Chứng minh rằng x2− y2 ... 402. Tìm tất cả các cặp số nguyên (x,y) thỏa mãn x4 +2x2 = y3

(THPT chuyên KHTN, ĐH KHTN, ĐHQG HN)

Page 105: 1 ¾ F - · PDF fileđang chuẩn bị cho kì thi tuyển sinh vào các lớp 10 ... phẩm “Tuyển tập các bài toán trong đề thi ... đam mê với các môn học

THCM

N

2.4 Số học 105

Lời giải

1. Ta có x2−12 = y2−1

3 ⇔ 3x2−2y2 = 1. Từ đây suy ra x lẻ.Ta thấy: với a nguyên thì a2 ≡ 0,4 (mod 8) nếu a chẵn và a2 ≡ 1 (mod 8) với a lẻ. Đồng

thời a2 ≡ 0,1,4 (mod 5). Khi đó 2(x2−y2) = 1−x2 ≡ 0 (mod 8) do x lẻ nên x2−y2 ... 4suy ra x,y cùng lẻ. Do đó

x2 ≡ y2 ≡ 1 (mod 8)⇒ x2− y2 ... 8. (2.4.1.1)

Ta có 3x2 ≡ 0,3,2 (mod 5) và 2y2 ≡ 0,2,3 (mod 5) mà 3x2−2y2 = 1 nên

x2 ≡ y2 ≡ 1 (mod 5)⇒ x2− y2 ... 5. (2.4.1.2)

Để ý rằng (8;5) = 1 nên từ (2.4.1.1),(2.4.1.2), ta có x2− y2 ... 402. Dễ thấy y≥ 0.

• Nhận xét (0;0) là một nghiệm nguyên của phương trình.• Với y > 0, ta có (y+1;y2−y+1) = (y+1;(y+1)2−3y) = (y+1;3y) = (y+1;3).

Mặt khác x2+1≡ 1,2 (mod 3)⇒ y+1 6 ... 3 nên (y+1;y2−y+1) = (y+1;3) = 1.Do đó y+1 và y2− y+1 là các số chính phương.Với y > 1 ta có (y−1)2 < y2− y+1 < y2 nên y2− y+1 không là số chính phương.Do đó y = 1. suy ra y+1 = 2 không là số chính phương.

Vậy không tồn tại (x;y) = (0;0) là cặp số nguyên duy nhất thỏa mãn đề bài.

Bài 2. Cho x,y là hai số nguyên dương thỏa mãn x2 + y2 +10... xy.

1. Chứng minh rằng x,y là hai số lẻ và nguyên tố cùng nhau.2. Chứng minh k = x2+y2+10

xy chia hết cho 4 và k ≥ 12.

(Trường Phổ thông Năng khiếu, ĐHQG Tp. HCM)

Lời giải

1. Nếu x chẵn suy ra xy chẵn nên x2 + y2 +10 chẵn. Do đó y chẵn. Khi đó xy,x2,y2 chia hết

cho 4 suy ra x2 + y2 +10... xy

... 4⇒ 10... 4 vô lý. Nếu y chẵn tương tự ta có điều vô lý. Vậy

x,y cùng lẻ.

Đặt (x;y) = d⇒ xy,x2,y2 ... d2⇒ x2 + y2 +10... d2⇒ 10

... d2. Mà 10 = 2.5 suy ra d = 1.Ta có đpcm.

2. Vì x,y lẻ nên x2 ≡ y2 ≡ 1 (mod 4)⇒ x2 + y2 +10 ≡ 12 ≡ 0 (mod 4). Mặt khác x,y lẻ

nên k... 4.

• Nếu x... 3 ta có xy

... 3⇒ x2+y2+10... 3⇒ y2+1

... 3, vô lý do a2≡ 0,1 (mod 3))∀a∈Z.• Nếu y

... 3 tương tự có điều vô lý.Do đó x,y cùng không chia hết cho 3 nên x2 + y2 +10≡ 1+1+1≡ 0 (mod 3). Suy ra

k... 3. Từ đó k

... 12⇒ k ≥ 12.

Page 106: 1 ¾ F - · PDF fileđang chuẩn bị cho kì thi tuyển sinh vào các lớp 10 ... phẩm “Tuyển tập các bài toán trong đề thi ... đam mê với các môn học

THCM

N

106 Chương 2. LỜI GIẢI

Bài 3. Tìm các số nguyên dương x,y thỏa mãn x3− y3 = 95(x2 + y2).

(THPT chuyên Đại học Sư phạm Hà Nội)

Lời giải

Để ý rằng chúng ta có bổ đề sau:

Bổ đề 5. Với x,y là số nguyên dương,

x2 + xy+ y2 ... 5⇔ x,y... 5.

Thật vậy, vì x2+xy+y2 ... 5 nên x3−y3 ...5. Ta có a3 ≡ 0,1,3,2,4 (mod 5) ∀a ∈Z mà x3−y3 ... 5,thử lần lượt các cặp số dư ta thấy chỉ có x≡ y (mod 5) thỏa mãn đề bài. Tiếp tục thử các cặp

x ≡ y (mod 5) thì ta chỉ thấy x ≡ y ≡ 0 (mod 5) thỏa x2 + xy+ y2...5. Vậy bổ đề được chứngminh.

Quay lại bài toán. Đặt (x;y) = d thì khi đó x = da;y = db với (a;b) =−1.Mà từ đề bài ta suy ra

95(a2 +b2)...a2 +ab+b2

Gọi q là số nguyên tố sao cho (a2 +b2;a2 +ab+b2)...q⇒ ab

...q. Mà có được a2 +b2...q nên a,b...q

vô lí do (a;b) = 1Vậy

95...a2 +ab+b2

Mà do (a;b) = 1 nên 19...a2 + ab+ b2. Xét các trường hợp, ta tìm được d = 65 và (x;y) =

(195;130).

Bài 4. Cho S là tập hợp các số nguyên dương n có dạng n = x2 +3y2 trong đó x,y nguyêndương. Chứng minh rằng

1. Nếu a,b ∈ S thì ab ∈ S

2. Nếu N ∈ S và N chẵn thì N... 4 và N

4 ∈ S

(THPT chuyên Đại học Sư phạm Hà Nội)

Lời giải

1. Ta viết a = x2 +3y2,b = z2 +3t2 thì

ab = (x2 +3y2)(z2 +3t2) = (xz)2 +3(xt)2 +3(yz)2 +9(yt)2

⇒ ab = (xz+3yt)2 +3(xt− yz)2

Do đó ab ∈ S

Page 107: 1 ¾ F - · PDF fileđang chuẩn bị cho kì thi tuyển sinh vào các lớp 10 ... phẩm “Tuyển tập các bài toán trong đề thi ... đam mê với các môn học

THCM

N

2.4 Số học 107

2. Ta viết N = x2 +3y2. Do N chẵn nên x,y cùng tính chẵn lẻ.

Nếu x,y cùng chẵn thì x2,y2 cùng chia hết cho 4 suy ra N... 4.

Nếu x,y cùng lẻ thì x2,y2 cùng chia 4 dư 1 nên N ≡ 1+3.1≡ 0 (mod 4) hay N... 4.

Do x,y cùng tính chẵn, lẻ nên tồn tại u,v là các số nguyên sao cho x = u+3v, y = v−u

hoặc x = u+3v, y = u− v. Khi đóN4∈ S (bạn đọc dễ dàng kiếm chứng bằng phép biến

đổi tương đương).

Bài 5. Tìm tất cả các cặp số nguyên tố (p;q) thỏa

p2−5q2 = 4.

(Bà Rịa - Vũng Tàu)

Lời giải

Dễ thấy p,q đều lẻ.Ta có p2−4 = 5q2⇔ (p−2)(p+2) = 5q2. Gọi d = gcd(p−2; p+2). Khi đó d|4, mà do p lẻnên d = 1Với mọi q là số nguyên tố lẻ thì ta luôn có q2 > 5. Do đó ta có hệ phương trình{

p−2 = 5p+2 = q2 .

Khi đó p = 7⇒ q = 3. Vậy (p;q) = (7;3).

Bài 6. Tìm ba số nguyên tố a,b,c thỏa mãn a < b < c , bc−1... a, ca−1

... b, ab−1... c.

(Bắc Ninh)

Lời giải

Xét N = ab+bc+ ca−1 ta thấy N chia hết cho a,b,c. Mà do a,b,c đôi một nguyên tố cùng

nhau nên N... abc. Đặt ab+ bc+ ca− 1 = kabc với k ∈ N∗. Vì a < b < c nên kabc < 3bc⇒

ka < 3⇒ ka≤ 2. Mà a nguyên tố nên a = 2 và k = 1. Từ đây ta có 2b+bc+2c−1 = 2bc⇔bc− 2b− 2c+ 4 = 3⇔ (b− 2)(c− 2) = 3 mà b < c⇒ b− 2 < c− 2 suy ra b− 2 = 1 vàc−2 = 3. Do đó b = 3,c = 5.Vậy a = 2,b = 3,c = 5.

Bài 7. Tìm nghiệm nguyên của phương trình

x2− (y+5)x+5y−2 = 0

(Bình Định)

Lời giải

Page 108: 1 ¾ F - · PDF fileđang chuẩn bị cho kì thi tuyển sinh vào các lớp 10 ... phẩm “Tuyển tập các bài toán trong đề thi ... đam mê với các môn học

THCM

N

108 Chương 2. LỜI GIẢI

Xét phương trình bậc 2 : x2− (y+5)x+5y−2 = 0. Để phương trình có nghiệm nguyên thì ∆

của nó phải chính phương, hay ∆ = (y+5)2−4(5y−2) = y2−10y+33 = z2⇔ (y−5)2 +8 =z2⇔ (z− y+ 5)(z+ y− 5) = 8. Mà z− y+ 5+ z+ y− 5 = 2z chẵn nên z− y+ 5 và z+ y− 5cùng tính chẵn lẻ. Do đó ta có các hệ phương trình sau:{

z− y+5 = 2z+ y−5 = 4 ;

{z− y+5 = 4z+ y−5 = 2 ;

{z− y+5 =−2z+ y−5 =−4 ;

{z− y+5 =−4z+ y−5 =−2

Tìm được các nghiệm y= 6;4, thay vào ta được các cặp nghiệm là (x;y)= (7;6);(4;6);(6;4);(3;4).

Bài 8. Cho 2 số nguyên dương lẻ m,n nguyên tố cùng nhau và thỏa :{m2 +2

... n

n2 +2... m

Chứng minh rằng: m2 +n2 +2... 4mn.

(Bình Thuận)

Lời giải

Dễ thấy m2 +n2 +2... m và m2 +n2 +2

... n mà (m;n) = 1 nên m2 +n2 +2... mn.

Lại do m,n lẻ nên m2≡ n2≡ 1 (mod 4) suy ra m2+n2+2≡ 4≡ 0 (mod 4) hay m2+n2+2... 4.

Mặt khác m,n lẻ nên (mn;4) = 1.

Từ đó m2 +n2 +2... 4mn

Bài 9. Tìm tất cả các cặp số nguyên (x;y) thỏa mãn 2x2−2x−6y2 +3y+ xy+7 = 0.

(Cần Thơ)

Hướng dẫn

2x2− 2x− 6y2 + 3y + xy + 7 = 0⇔ 2x2− 3xy + 4xy− 6y2− 2x + 3y = −7⇔ x(2x− 3y) +2y(2x−3y)− (2x−3y) =−7⇔ (2x−3y)(x+2y−1) =−7. Ta có các trường hợp:TH1: 2x−3y = 1 và x+2y−1 =−7. Trường hợp này không có nghiệm nguyên.TH2: 2x−3y =−1 và x+2y−1 = 7. Trường hợp này không có nghiệm nguyên.TH3: 2x−3y =−7 và x+2y−1 = 1. Trường hợp này không có nghiệm nguyên.TH4: 2x−3y = 7 và x+2y−1 =−1⇒ x = 2,y =−1.

Bài 10. Tìm tất cả các số nguyên dương x và số nguyên tố p sao cho

x5 + x4 +1 = p2

(Đà Nẵng)

Lời giải

Page 109: 1 ¾ F - · PDF fileđang chuẩn bị cho kì thi tuyển sinh vào các lớp 10 ... phẩm “Tuyển tập các bài toán trong đề thi ... đam mê với các môn học

THCM

N

2.4 Số học 109

x5 + x4 +1 = p2⇔ (x2 + x+1)(x3− x+1) = p2. Xét các trường hợp:TH1: x2 + x+1 = x3− x+1 = p⇒ x3− x2−2x = 0⇔ x2− x−2 = 0⇒ x = 2⇒ p = 7.TH2: x2 + x+1 = p2,x3− x+1 = 1. Trường hợp này vô nghiêm, bạn đọc dễ dàng kiểm chứng.TH3: x2 + x+1 = 1,x3− x+1 = p2. Trường hợp này cũng vô nghiệm.Vậy (x; p) là (2;7).

Bài 11. Tìm tất cả các cặp số tự nhiên (x;y) thỏa

2x.x2 = 9y2 +6y+16

(Hà Nội)

Lời giải

Ta thấy x dương và không chia hết cho 3, do đó x2 ≡ 1 (mod 3) nên 2x.x2 ≡ (−1)x (mod 3).Mặt khác 9y2 +6y+16≡ 1 (mod 3) nên (−1)x ≡ 1 (mod 3). Vì vậy x chẵn. Đặt x = 2k ta có

(2k.2k)2 = (3y+1)2 +15⇔ (2k.2k−3y−1)(2k.2y+3y+1) = 15

Do đó ta có các trường hợp:TH1: 2k.2k− 3y− 1 = 1 và 2k.2k+ 3y+ 1 = 15 suy ra 2k.2k = 8 và 3y+ 1 = 7. Không có k

thỏa mãn trường hợp này.TH2: 2k.2k−3y−1 = 3 và 2k.2k+3y+1 = 5 suy ra 2k.2k = 4 và 3y+1 = 1. Do đó k = 1 và

y = 0.Vậy (x;y) cần tìm là (2;0).

Bài 12. Tìm các bộ số nguyên dương (x;y;z) thỏa{x+ y− z = 0

x3 + y3− z2 = 0

(Hà Tĩnh)

Lời giải

Chúng ta có bổ đề hết sức quen thuộc sau:

Bổ đề 6. Cho x,y > 0. Khi đóx3 + y3 ≥ xy(x+ y).

Dấu bằng xảy ra⇔ x = y.

Quay lại bài toán. Ta có z = x+ y nên thay xuống phương trình cuối, ta được

x3 + y3 = (x+ y)2 ≥ xy(x+ y)⇔ x+ y≥ xy.

Mà x+ y− xy = x(1− y)− (1− y)+1 = (x−1)(1− y)+1≤ 1Do đó nếu x+ y = xy⇒ x = y = 2Nếu x+ y− xy = 1 thì có 1 trong 2 số bằng 1Do đó nghiệm của phương trình là (x;y;z) = (2;2;4),(1;2;3);(2;1;3).

Page 110: 1 ¾ F - · PDF fileđang chuẩn bị cho kì thi tuyển sinh vào các lớp 10 ... phẩm “Tuyển tập các bài toán trong đề thi ... đam mê với các môn học

THCM

N

110 Chương 2. LỜI GIẢI

Bài 13.1. Tìm dạng tổng quát của số nguyên dương n biết M = n.4n +3n chia hết cho 72. Tìm số các cặp số (x;y) nguyên dương thỏa mãn:

(x2 +4y2 +28)2−17(x4 + y4) = 238y2 +833.

(Hải Dương)

Lời giải

1. Ta có 4≡−3 (mod 7) nên n(−3)n ≡−3n (mod 7).TH1: n chẵn ta có n.3n ≡ −3n (mod 7) mà (3;7) = 1 nên n ≡ −1 (mod 7). Do đó n =

14k+6 ∀k ∈ N.TH2: n lẻ ta có −n.3n ≡ −3n (mod 7) mà (3;7) = 1 nên n ≡ 1 (mod 7). Do đó n =

14k+1 ∀k ∈ N2. (x2 + 4y2 + 28)2 − 17(x4 + y4) = 238y2 + 833 ⇔ −16x4 − y4 + 784 + 8x2y2 + 56x2 +

224y2 = 238y2 + 833⇔ 16x4 + y4 + 49− 8x2y2− 56x2 + 14y2 = 0⇔ (4x2− y2− 7) =0⇔ 4x2− y2 = 7⇔ (2x− y)(2x+ y) = 7. Vì 2x− y < 2x+ y nên ta có 2x− y = 1 và2x+ y = 7. Suy ra x = 2 và y = 3.

Bài 14. Tìm tất cả các số nguyên m,n với m ≥ n ≥ 0 sao cho (m+2n)3 là ước của 9(m2 +mn+n2 +16).

(Hải Phòng)

Lời giải

Do (m+2n)3 là ước của 9(m2 +mn+n2 +16) nên

(m+2n)3 ≤ 9(m2 +mn+n2 +16)

⇔ m3 +6m2n+12mn2 +8n3 ≤ 9m2 +9mn+9n2 +144

Nếu m≥ n≥ 3 thì n3 ≥ 3n2. Ta có:

m3 +8n3 ≥ 9n3 ≥ 27n2 > 9n2 +144

6m2n≥ 18m2 > 9m2

12mn2 ≥ 36mn > 9mn

Suy ra m3 +6m2n+12mn2 +8n3 > 9m2 +9mn+9n2 +144 vô lý nên n ∈ {0;1;2}.• Với n = 0 ta có m3 ≤ 9(m2 +16)⇒ m≤ 10. Thử các giá trị ta thấy m = 0;m = 1 thỏa• Với n = 1 ta có m3 +6m2 +12m+8≤ 9m2 +9m+153⇒ m3−3m2 +3m−145≤ 0⇒(m−1)3 ≤ 144⇒ m−1≤ 5⇒ m≤ 6 nên m ∈ {1;2;3;4;5;6}. Thử lại đều không thỏamãn.• Với n = 2 ta có m3 +12m2 +48m+64≤ 9m2 +18m+180⇒ m3−6m2 +30m−116≤

0⇒ (m− 2)3 ≤ 108− 18m < 108⇒ m− 2 ≤ 4⇒ m ≤ 5 nên m ∈ {1;2}. Thử lại đềukhông thỏa mãn.

Vậy (m;n) = (0;0);(1;0) thỏa mãn đề bài.

Page 111: 1 ¾ F - · PDF fileđang chuẩn bị cho kì thi tuyển sinh vào các lớp 10 ... phẩm “Tuyển tập các bài toán trong đề thi ... đam mê với các môn học

THCM

N

2.4 Số học 111

Bài 15. Cho m,n là các số nguyên dương sao cho 5m+n chia hết cho 5n+m. Chứng minh

rằng m...n.

(Tp. HCM)

Lời giải

Do 5m+n chia hêt cho 5n+m nên tồn tại k nguyên dương và k < 5 sao cho:

5m+n = k(5n+m)

Tức làm = n.

5k−15− k

Thử các giá trị của k = 1;2;3;4 ta lần lượt có m = n;m = 3n;m = 7n;m = 19n. Do đó m...n.

Bài 16. Tìm tất cả các nghiệm nguyên dương (x;y;z) của phương trình

xyz+ xy+ yz+ zx+ x+ y+ z = 2015 thỏa x≥ y≥ z≥ 8.

(Hưng Yên)

Hướng dẫn

Vì x≥ y≥ z nên 2015 = xyz+xy+yz+ zx+x+y+ z≥ z3+3z2+3z suy ra 2016≥ (z+1)3⇒z+1 < 13⇒ z≤ 11 mà z≥ 8 nên z ∈ {8;9;10;11}.Với z = 8 ta có

8xy+ xy+8y+8x+ x+ y+8 = 2015⇔ 9xy+9x+9y = 2007⇔ xy+ x+ y = 223

⇔ (x+1)(y+1) = 224.

Bài 17. Tìm tất cả số nguyên tố p sao cho 8p2 +1 và 8p2−1 là số nguyên tố.

(Khánh Hoà)

Lời giải

Ta có x2 ≡ 0;1 (mod 3) nên nếu 3 không là ước của p thì p2 ≡ 1 (mod 3)⇒ 8p2 + 1 ≡ 0(mod 3). Do đó p = 3. Thử lại ta thấy p = 3 thỏa yêu cầu bài toán.

Page 112: 1 ¾ F - · PDF fileđang chuẩn bị cho kì thi tuyển sinh vào các lớp 10 ... phẩm “Tuyển tập các bài toán trong đề thi ... đam mê với các môn học

THCM

N

112 Chương 2. LỜI GIẢI

Bài 18.1. Chứng minh rằng tồn tại vô hạn bộ ba số nguyên (x;y;z) thỏa xyz 6= 0 và x5+8y3+7z2 =

02. Tìm tất cả các số nguyên không âm (a;b;c) thỏa và{

(a−b)2 +(b− c)2 +(c−a)2 = 6abc

a3 +b3 + c3 +1... a+b+ c+1

(Nam Định)

Lời giải

1. Ta thấy với mọi a ∈ N∗ thì bộ (x;y;z) là (a6;−a10;a15) thỏa mãn đề bài. Do đó tồn tại vôhạn bộ (x;y;z) thỏa mãn.

2. Ta có:(a−b)2 +(b− c)2 +(c−a)2 = 6abc

⇔ a2 +b2 + c2−ab−bc− ca = 3abc

Do đó(a+b+ c)3 = a3 +b3 + c3 +3(a+b+ c)(ab+bc+ ca)−3abc

= a3 +b3 + c3 +3(a+b+ c)(ab+bc+ ca)−a2−b2− c2 +ab+bc+ ca.

Khi đó

a3+b3+c3+1=(a+b+c)3+1−3(a+b+c)(ab+bc+ca)+a2+b2+c2−ab−bc−ca

=(a+b+c+1)((a+b+c)2−(a+b+c)+1)−3(ab+bc+ca)(a+b+c+1)+(a+b+c)2

Suy ra (a+b+c)2 ... a+b+c+1 mà (a+b+c,a+b+c+1)= 1 nên suy ra a+b+c+1=1. Từ đó a = b = c = 0

Bài 19. Tìm tất cả các căp số nguyên (x;y) thỏa

1+ x+ x2 + x3 + x4 = y2.

(Ninh Bình)

Lời giải

Ta có4y2 = 4x4 +4x3 +4x2 +4x+4 = (2x2 + x)2 +2x2 +(x+2)2

Từ đó suy ra4y2 > (2x2 + x)2⇒ 4y2 ≥ (2x2 + x+1)2

Tức là

4x4 +4x3 +4x2 +4x+4≥ 4x4 + x2 +1+4x3 +4x2 +2x⇔ x2−2x−3≤ 0

Do đó −1≤ x≤ 3. Thử các giá trị, ta nhận các nghiêm S = (−1;1);(0;1);(3;11).

Page 113: 1 ¾ F - · PDF fileđang chuẩn bị cho kì thi tuyển sinh vào các lớp 10 ... phẩm “Tuyển tập các bài toán trong đề thi ... đam mê với các môn học

THCM

N

2.4 Số học 113

Bài 20. Tìm các số nguyên (x;y) thỏa mãn

2x3 +2x2y+ x2 +2xy = x+10

(Phú Thọ)

Lời giải

Biến đổi tương đương, ta thu được

y(2x2 +2x)+2x3 + x2− x−10 = 0

Dễ thấy x = 0;−1 không thỏa. Do đó

2y =−2x3 + x2− x−10x2 + x

=−(

2x−1− 10x2 + x

).

Để y nguyên thì x2+x|10, mà x nguyên nên x2+x = 2. Thay vào ta được các nghiệm của phươngtrình là (x;y) = (−2;5),(x;y) = (1;2).

Bài 21. Hãy tìm bộ ba số nguyên dương a≤ b≤ c thỏa mãn đẳng thức sau:

abc = 2(a+b+ c).

(Quảng Nam)

Lời giải

Vì a≤ b≤ c nên 2(a+b+ c)≤ 2.3c⇒ abc≤ 6c⇒ ab≤ 6. Xét các trường hợp:TH1: ab = 1 suy ra a = b = 1⇒ c = 2(c+2) vô lý.TH2: ab = 2 suy ra a = 1, b = 2⇒ 2c = 2(c+3) vô lý.TH3: ab = 3 suy ra a = 1, b = 3⇒ 3c = 2(c+4)⇔ c = 8.TH4: ab = 4 suy ra hoặc a = 1b = 4 hoặc a = b = 2.

– Với a = b = 2 ta có 4c = 2(c+4)⇔ c = 4.– Với a = 1, b = 4 ta có 4c = 2(c+5)⇔ c = 5.

TH5: ab = 5 suy ra a = 1, b = 5⇒ 5c = 2(c+6)⇔ c = 4 loại.TH6: ab = 6 suy ra:

– Với a = 1, b = 6 ta có 6c = 2(c+7)⇒ c 6∈ Z.– Với a = 2, b = 3 ta có 6c = 2(c+5)⇒ c 6∈ Z.

Vậy các bộ ba số (a;b;c) thỏa mãn đề là (1;3;8),(2;2;4),(1;4;5).

Page 114: 1 ¾ F - · PDF fileđang chuẩn bị cho kì thi tuyển sinh vào các lớp 10 ... phẩm “Tuyển tập các bài toán trong đề thi ... đam mê với các môn học

THCM

N

114 Chương 2. LỜI GIẢI

Bài 22.1. Tìm tất cả các nghiệm nguyên của phương trình 2xy+ y+ x = 83.2. Tìm tất cả các số có 5 chữ số abcde sao cho

3√abcde = ab

3. Cho 3 số nguyên dương a,b,c nguyên dương, nguyên tố cùng nhau thỏa điều kiện1a+

1b=

1c

. Chứng minh a+b là số chính phương.

(Thái Nguyên)

Lời giải

1. 2xy+y+x = 83⇔ 4xy+2x+2y+1 = 167⇔ (2x+1)(2y+1) = 167. Không giảm tổngquát giả sử x≤ y thì 2x+1≤ 2y+1. Ta có các trường hợp:

TH1: 2x+1 = 1 và 2y+1 = 167. Suy ra x = 0 và y = 83.TH2: 2x+1 =−167 và 2y+1 =−1. Suy ra x =−84 và y =−1.Vậy phương trình có các nghiệm nguyên (x;y) là (0;83),(83;0),(−84;−1),(−1;−84)

2. Ta thấy 3√104 ≤ ab≤ 3√105 suy ra 22≤ ab≤ 46. Do đó a ∈ {2;3;4}.• Với a = 2: Ta có 3√2.104 ≤ 2b ≤ 3√3.104 ⇒ 28 ≤ 2b ≤ 31 nên b ∈ {8;9} thử lại

không đúng.• Với a = 3: Ta có 3√3.104 ≤ 3b≤ 3√4.104⇒ 32≤ 3b≤ 34 nên b ∈ {2;3;4} thử lại

chỉ có b = 2 thỏa mãn.• Với a = 4: Ta có 3√4.104 ≤ 4b≤ 3√5.104⇒ 35≤ 4b≤ 36 vô lý.

Vậy số cần tìm là 32768.3. Giả thiết⇔ ab = c(a+b) và a,b > c. Đặt (a;c) = m và (b;c) = n thì (m;n) = 1. Do đó

c... mn.

Viết c = mnt, a = mx, b = ny với x,y nguyên dương thì (nt;x) = (mt;y) = 1 suy ra

(n;x) = (t;x) = (m;y) = (t;y) = 1. Mặt khác ab... c⇒ xy

... t suy ra t = 1.

Với c = mn, a = mx, b = ny ta có xy = mx+ ny. Từ đây suy ra mx... y và ny

... x. Lại có

(n;x) = (m;y) = 1 nên x... y và y

... x suy ra x = y. Vì vậy a+b = xy = x2 là số chính phương.

Bài 23. Tìm các số nguyên x,y thỏa:

9x2 +3y2 +6xy−6x+2y−35 = 0

(Thái Bình)

Lời giải

9x2 +3y2 +6xy−6x+2y−35 = 0

⇔ (9x2 + y2 +1+6xy−6x−2y)+2(y2 +2y+1) = 38⇔ (3x+ y−1)2 +2(y+1)2 = 38

Từ đây suy ra 3x+ y−1 chẵn và không vượt quá 6. Thử với 0,2,4,6 ta chỉ có 38 = 62 +2.12

nên 3x+ y−1 = 6 và y+1 = 1. Suy ra y = 0 và 3x = 7 (loại).Vậy phương trình không có nghiệm nguyên.

Page 115: 1 ¾ F - · PDF fileđang chuẩn bị cho kì thi tuyển sinh vào các lớp 10 ... phẩm “Tuyển tập các bài toán trong đề thi ... đam mê với các môn học

THCM

N

2.4 Số học 115

Bài 24. Tìm các bộ số nguyên dương (x;y;z) thỏa mãn 1x +

!y +

1z = 1 và

√x− y+ z =√

x−√y+√

z.

(Thừa Thiên - Huế)

Lời giải

Ta có √x− y+ z =

√x−√y+

√z⇔√

x− y+ z+√

y =√

x+√

z

⇔ x− y+ z+ y+2√

(x− y+ z)y = x+ z+2√

xz⇔ (x− y+ z)y = xz

⇔ xy+ yz = y2 + zx.

Lại có1x+

!y+

1z= 1⇔ xy+ yz+ zx = xyz

⇔ y2 +2zx = xyz⇔ y2 = zx(y−2)

Suy ra y2 ... y−2⇒ y2−4+4... y−2⇒ 4

... y−2. Vậy y−2 = 1, y−2 = 2 hoặc y−2 = 4 suy ray ∈ {3;4;6}.• Với y = 3 ta có zx = 9 và 1

x +1z =

23 . Suy ra x+ z = 6. Vì vậy x và z là nghiệm của phương

trình X2−6X +9 = 0 nên x = z = 3.• Với y = 4 ta có zx = 8 và 1

x +1z =

34 . Suy ra x+ z = 6. Vì vậy x và z là nghiệm của phương

trình X2−6X +8 = 0 nên x = 4,z = 2 hoặc x = 2,z = 4.• Với y = 6 ta có zx = 9 và 1

x +1z =

56 . Suy ra x+ z = 15

2 , loại.Vậy các bộ (x;y;z) cần tìm là (3;3;3),(2;4;4),(4;4;2).

Bài 25. Tìm tất cả nghiệm nguyên x,y của phương trình x2 = y2(x+ y4 +2y2).

(Vĩnh Phúc)

Lời giải

x2 = y2(x+ y4 +2y2)⇔ x2− xy2− y6−2y4 = 0. (2.4.10.1)

Coi phương trình trên là phương trình bậc 2 với ẩn x thì

∆ = y4 +4(y6 +2y4) = y4(4y2 +9)

Phương trình (2.4.10.1) có nghiệm nguyên khi và chỉ khi ∆ là số chính phương. Điều này tươngđương với 4y2 +9 là số chính phương.Đặt 4y2 +9 = k2 với k ∈ N. Ta có:

k2−4y2 = 9⇔ (k−2y)(k+2y) = 9.

Không giảm tổng quát giả sử y≥ 0. Từ đây suy ra các trường hợp:TH1: k−2y = 1 và k+2y = 9. Nên k = 5 và y = 2. Ta có x2 = 4(x+24)⇔ x2−4x−96 = 0⇒

x = 12 hoặc x =−8. Lần lượt thay x =−8 và x = 12 vào phương trình ban đầu đều chokết quả y = 2 hoặc y =−2 (bạn đọc dễ dàng kiểm tra, chú ý y nguyên).

TH2: k−2y = k+2y = 3⇒ k = 3 và y = 0 nên x = 0.TH3: k−2y = k+2y =−3⇒ k =−3, loại.Tương tự với y≤ 0. Vậy phương trình có các nghiệm nguyên (x;y) là (0;0), (12;2), (12;−2),(−8;2),(−8;−2).

Page 116: 1 ¾ F - · PDF fileđang chuẩn bị cho kì thi tuyển sinh vào các lớp 10 ... phẩm “Tuyển tập các bài toán trong đề thi ... đam mê với các môn học

THCM

N

116 Chương 2. LỜI GIẢI

2.5 Tổ hợp

Bài 1. Chứng minh rằng với mọi số nguyên n ≥ 3 luôn tồn tại một cách sắp xếp bộ n số1,2, ...,n thành x1,x2, ...xn sao cho x j 6= xi+xk

2 với mọi bộ số chỉ số (i, j,k) mà 1≤ i< j < k≤ n.

(Trường THPT chuyên Khoa học Tự nhiên, ĐH KHTN, ĐHQG HN)

Lời giải

Chúng tôi xin trích đăng lời giải của thầy Hồng Trí Quang, trung tâm Hocmai.vn.• Với n = 3, ta có cách sắp xếp 1,3,2.• Ta chứng minh nếu bài toán đúng với n, nó cũng sẽ đúng với 2n. Thật vậy, giả sử ta có cách

sắp xếp đúng với n thì cách sắp xếp đó có dạng x1,x2, ...,xn thoả mãn ∀1≤ i < j < k ≤ nthì x j 6= xi+xk

2 .Xét dãy sau:

2x1,2x2, ...,2xn,2x1−1,2x2−1, ...,2xn−1.

Dễ thấy dãy gốm tất cả các số từ 1 đến 2n. Xét a < b bất kì cùng thuộc dãy.

TH1: Nếu a,b khác tính chẵn lẻ thìa+b

2không thuộc dãy (thoả mãn).

TH2: Nếu a,b cùng tính chẵn lẻ thìa+b

2thuộc dãy. Không mất tính tổng quát, giả sử a,b

cùng chẵn (trường hợp cùng lẻ chứng minh tương tự).

∗ a+b2

lẻ thìa+b

2không thể nằm giữa a,b theo cách xây dựng dãy (thoả mãn).

∗ Nếua+b

2chẵn, giả sử

a+b2

nằm giữa a,b trong dãy, Khi đó a = 2xi,b =

2xk,a+b

2= 2x j(i < j < k)⇒ x j =

xi + xk

2⇒ Dãy ban đầu x1,x2, ...,xn là cách

sắp xếp không thoả mãn đề bài, mâu thuẫn.

Vậy điều giả sử là sai, do đóa+b

2không thể nằm giữa a,b trong dãy.

Vậy với mọi trường hợp, trung bình cộng của a,b không thể nằm giữa a,b suy ra đã xâydựng được cách xếp thoả mãn cho trường hợp 2n. Mặt khác, nếu bài toán đã đúng với n,nó cũng sẽ đúng với n−1 (bằng cách bỏ đi số đầu tiên hoặc số cuối cùng). Theo nguyên líquy nạp, ta có đpcm.

Bài 2. Giả sử mỗi điểm của mặt phẳng được tô bởi một trong ba màu : xanh,đỏ,vàng . Chứngminh rằng tồn tại ba điểm cùng màu là ba đỉnh của một tam giác cân.

(Trường THPT chuyên Đại học Sư phạm HN)

Lời giải

Nhận xét.1. Chọn 3 đỉnh bất kì của ngũ giác thì sẽ có 2 đỉnh kề nhau.(Dễ thấy)2. 3 đỉnh bất kì của ngũ giác đều là 1 tam giác cân. Thật vậy, xét ngũ giác đều ABCDE, ta

chọn ra 3 đỉnh bất kì, theo nhận xét 2 thì chắc chắn sẽ có 2 đỉnh được chọn kề nhau. Giảsử là D và C. Điểm còn lại chỉ có thể là A, B, E. Mà4EDC,4BCD,4ADC đều là tamgiác cân.

Page 117: 1 ¾ F - · PDF fileđang chuẩn bị cho kì thi tuyển sinh vào các lớp 10 ... phẩm “Tuyển tập các bài toán trong đề thi ... đam mê với các môn học

THCM

N

2.5 Tổ hợp 117

3. Nếu các đỉnh của một ngũ giác đều chỉ được tô bằng 2 màu thì ta luôn tìm được 1 tamgiác cân có đỉnh cùng màu từ các đỉnh của ngũ giác đó. Thật vậy, theo ngyên lí Dirichletthì tồn tại 3 điểm được tô cùng 1 màu. Mà theo nhận xét 3 , đó là tam giác cân có đỉnhcùng màu.

Xét 1 điểm trên mặt phẳng, giả sử đó là điểm O và được tô màu đỏ. Vẽ đường tròn tâm O bánkính 1 và ngũ giác đều A1B1C1D1E1 nội tiếp đường tròn tâm O đó. Thì 1 trong các đỉnh của ngũgiác đều sẽ là màu đỏ. Nếu không thì theo nhận xét 3, thì bài toán chứng minh xong.Vẽ ngũ giác đều A2B2C2D2E2 đối xứng với ngũ giác đều A1B1C1D1E1 qua tâm O. Tương tự thìcũng có 1 điểm màu đỏ. Vậy, bài toán cũng chứng minh xong vì có 2 điểm màu đỏ đó và tâm Ocũng màu đỏ lập thành tam giác cân có đỉnh cùng màu.

Bài 3. Với mỗi số nguyên dương m > 1, kí hiệu s(m) là ước nguyên dương lớn nhất của mvà khác m. Cho số tự nhiên n > 1, đặt n0 = n và lần lượt tính các số n1 = n0− s(n0);n2 =n1− s(n1); . . . ;ni+1 = ni−S(ni); . . . . Chứng minh rằng tồn tại số nguyên dương k để nk = 1và tính k khi n = 216.1417.

(Trường Phổ thông Năng khiếu, ĐHQG Tp. HCM)

Lời giải

Ta có, dãy ni+1 = ni− s(ni), mà n > s(ni)> 0 nên 0 < ni+1 < ni. Vậy n = n0 > n1 > . . . > ni >. . . > 0.Nếu không tồn tại nk = 1 thì ta sẽ xây dựng được 1 dãy gồm vô hạn số dương giảm và nhỏ hơnn, vô lí. Vậy tồn tại k sao cho nk = 1.Khi n= 216.1417 = 233.717, ta có n1 = 233.717−232.717 = 232.717. Tương tự, n2 = 231.717, . . . ,n33 =20.717 = 717. Đặt n33 = m0. Ta có m1 = 6.716;m2 = 3.716;m3 = 2.716;m4 = 716. Tương tự sẽcó m8 = 715, . . . ,m68 = 70 = 1. vậy k = 33+68 = 101.

Bài 4. Người ta dùng một số quân cờ tetromino gồm 4 ô vuông kích thước 1 x 1, hình chữ L,có thể xoay hoặc lật ngược như hình 1 để ghép phủ kín một bàn cờ hình vuông kích thước n xn (n là số nguyên dương) gồm n2 ô vuông kích thước 1 x 1 theo quy tắc:

i. Với mỗi quân cờ sau khi ghép vào bàn cờ, các ô vuông của nó phải trùng với các ôvuông của bàn cờ.

ii. Không có hai quân cờ nào mà sau khi ghép vào bàn cờ chúng kê lên nhau.1. Khi n = 4, hãy chỉ ra một cách ghép phủ kín bàn cờ.2. Tìm tất cả giá trị của n để có thể ghép phủ kín bàn cờ.

(Đà Nẵng)

Hình 1

Lời giải

Page 118: 1 ¾ F - · PDF fileđang chuẩn bị cho kì thi tuyển sinh vào các lớp 10 ... phẩm “Tuyển tập các bài toán trong đề thi ... đam mê với các môn học

THCM

N

118 Chương 2. LỜI GIẢI

1. Hình vuông 4 x 4 được phủ bởi 4 ô tetromino (các ô đánh số 1 là tetromino thứ nhất, . . .các ô đánh số 4 là tetromino thứ 4).

1 4 4 41 4 3 31 1 2 32 2 2 3

2. Ta sẽ chứng minh là chỉ có hình vuông 4n x 4n mới được phủ bởi các tetromino. Thật vậy:• Nếu hình vuông có cạnh lẻ, suy ra số ô là lẻ, không chia hết cho 4. Vậy không thể

nào có thể phủ hết bằng các ô tetromino.• Ta xét trường hợp (4n+ 2) x (4n+ 2) = 16n2 + 16n+ 4, ta sẽ cần 4n2 + 4n+ 1 ô

tetromino. Đánh số ô vuông (4n+2) x (4n+2) bằng các số 1 và −1 xen kẽ theohàng.Ví dụ với n = 6:

Vậy sẽ có 8n2+8n+2 số 1 và 8n2+8n+2 số −1. Nhận xét: ô tetromino khi đặt lênbàn sẽ phủ ba số 1, một số -1 hoặc ba số -1, một số 1. Gọi x là số ô tetromino phủ ba số1, một số −1 ; y là số tetromino phủ ba số −1, một số 1. Ta có 3x+y = 8n2+8n+2(số số 1) là số chẵn, suy ra x+ y chẵn (do 2x chẵn). Lại có tổng số ô tetromino là4n2 +4n+1 là lẻ, suy ra x+ y lẻ (mâu thuẫn). Vậy với hình vuông cạnh bằng 4n+2không thể được phủ bởi tetromino.• Với hình 4n x 4n, ta sẽ phủ bởi các ô 4 x 4 (hình 2). Suy ra là bảng 4n x 4n luôn phủ

được.

Bài 5. Cho 2017 số hữu tỷ dương được viết trên một đường tròn. Chứng minh tồn tại hai sốđược viết cạnh nhau trên đường tròn sao cho khi bỏ 2 số đó thì 2015 số còn lại không thểchia thành hai nhòm mà tổng các số ở mỗi nhóm bằng nhau.

(Hà Nội)

Lời giải

Ta có thể đưa về bài toán mà 2017 số là các số nguyên dương (vì ta có thể quy đồng mẫu số củatất cả các số hữu tỉ trên đường tròn thì ta được tự số là các số nguyên dương).Gọi ước chung lớn nhất của 2017 số này là d, chia tất cả các số cho d ta đưa về bài toán với 2017số nguyên dương có ước chung lớn nhất là 1.

Page 119: 1 ¾ F - · PDF fileđang chuẩn bị cho kì thi tuyển sinh vào các lớp 10 ... phẩm “Tuyển tập các bài toán trong đề thi ... đam mê với các môn học

THCM

N

2.5 Tổ hợp 119

TH1: Tổng 2017 số là số lẻ. Nhận xét rằng tồn tại 2 số cùng tính chẵn lẽ kề nhau. Thật vậy, dotrên đường tròn có 2017 số nguyên dương, vì 2017 là số lẻ nên sẽ có 2 số nằm kề nhautrên đường tròn cùng tính chẵn lẻ. Thế nên, chỉ cần bỏ đi 2 số đó là tổng thì tổng 2015 sốlúc sau là số lẻ nên không thể chia thành 2 nhóm có tổng bằng nhau.

TH2: Tổng 2017 số là số chẵn. Nhận xét rằng tồn tại 1 số lẻ, vì nếu tất cả đều chẵn thì mâuthuận với ƯCLN của chúng là 1 theo giả sử. Ngoài ra, để ý rằng tồn tại 1 số chẵn nằmcạnh 1 số lẻ. Thật vậy, vì nếu không tồn tại số chẵn nào nằm cạnh 1 số lẻ, cũng có nghĩalà không có số chẵn nào trên đường tròn thì tổng 2017 số trên đường tròn là lẻ, mâu thuẫnvới điều giả sử. Thế nên, từ 2 nhận xét trên, ta sẽ bỏ đi 2 số khác tính chẵn lẻ nằm kề nhautrên đường tròn, lúc đó ta được tổng 2015 số lúc sau là số lẻ nên cũng không chia ra đượcthành 2 nhóm bằng nhau.

Bài 6. Trên một đường tròn, lấy 1000 điểm phân biệt, các điểm được tô màu xanh và đỏ xenkẽ nhau. Mỗi điểm được gán với một giá trị là một số thực khác 0. Giá trí của mỗi điểm xanhbằng tổng giá trị của hai điểm đỏ kề với nó, giá trị của mỗi điểm đỏ bằng tích giá trị của haiđiểm xanh kề với nó. Tính tổng giá trị của 1000 điểm trên.

(Hà Tĩnh)

Lời giải

Trong 1000 điểm đó, ta chọn một điểm đỏ làm mốc và có giá trị a; điểm xanh bên cạnh có giátrị b (điểm bên cạnh sẽ được xác định theo chiều kim đồng hồ). Khi đó các điểm tiếp theo nó tacũng sẽ gọi thứ tự và theo công thức được xác định như đầu bài thì ta sẽ lần lượt xác định đượcmàu và giá trị của các điểm tiếp theo. Cụ thể là

a;b;b−a;b−a

b;b−a

b− (b−a);1−b;

a−abb

;ab

;a;b.

Nhìn vào dãy trên ta thấy rõ ràng giá trị các điểm sẽ lặp lại theo chu kì 8. Do đường tròn có 1000điểm nên chu kì trên sẽ lặp lại đúng 125 lần. Ta có

a+b+b−a+b−a

b+

b−ab− (b−a)+1−b+

a−abb

+ab= 3.

Vậy tổng 1000 số trên là 125.3 = 375.

Bài 7. Trong dãy số thực a1;a2;a3; . . . ;a2016 ta đánh dấu tất cả các số dương và số mà có ítnhất một tổng của nó với một số các số liên tiếp liền ngay sau nó là một số dương (ví dụ trongdãy−6;5;−3;3;1;−1;−2;−3; . . . ;2011 ta đánh dấu các số a2 = 5;a3 =−3;a4 = 3;a5 = 1).Chứng minh rằng nếu trong dãy đã cho có ít nhất một số dương thỉ tổng tất cả các số đượcđánh dấu là một số dương.

(Hải Phòng)

Lời giải

• Xét trường hợp các số đánh dấu chỉ toàn các số không âm, bài toán đã được chứng minh.

Page 120: 1 ¾ F - · PDF fileđang chuẩn bị cho kì thi tuyển sinh vào các lớp 10 ... phẩm “Tuyển tập các bài toán trong đề thi ... đam mê với các môn học

THCM

N

120 Chương 2. LỜI GIẢI

• Xét trường hợp các số đánh dấu có ít nhất một số âm. Xét số âm được đánh dấu đầu tiên,gọi là ai. Tổng của ai với một số các số liền tiếp phía sau (gọi số cuối của dãy đó là a j). làsố dương (giả thiết). Vậy

ai +ai+1 + . . . +a j > 0.

Từ i dến j, bỏ các số không được đánh dấu (nếu có) đi. Vì các số không được đánh dấu chỉcó thể là số âm nên khi bỏ đi, ta sẽ có tổng của các số được đánh dấu từ i tới j sẽ lớn hơn0. Sau đó xét tiếp tới số âm sau a j đầu tiên được đánh dấu, gọi là ak, làm tương tự. Cònnhững số được đánh dấu giữa a j và ak chắc chắn lớn hơn 0 (do nếu bé hơn 0 thì ak khôngphải là số âm sau a j đầu tiên được đánh dấu). Như vậy, ta sẽ có tổng các số được đánh dấulớn hơn 0 (đpcm).

Bài 8. Nam cắt 1 tờ giấy ra làm 4 miếng hoặc 8 miếng rồi lấy một số miếng nhỏ đó cắt ralàm 4 miếng hoặc 8 miếng nhỏ hơn, và Nam cứ tiếp tục việc cắt như thế nhiều lần. Hỏi vớiviệc cắt như vậy, Nam có thể cắt được 2016 miếng lớn, nhỏ được hay không? Vì sao?

(Tp. HCM)

Hướng dẫn

Để ý rằng cắt một mảnh giấy ra làm 4 sẽ làm số mảnh tăng 3, cắt một mảnh giấy ra làm 8 sẽ làmsố mảnh tăng 7.Gọi x là số lần cắt làm 4,y là số lần cắt làm 8. Ban đầu có 1 mảnh giấy. Giả sử nếu cắt đượcthành 2016 mảnh thì ta cần tìm x,y thỏa 1+3x+7y = 2016.

y 0 1 2x Loại vì x không phải là số tự nhiên 669 Loại vì x không phải là số tự nhiên

Vậy ta cắt mảnh giấy thành 8 mảnh nhỏ, sau đó lấy 1 mảnh nhỏ, cắt làm 4 phần. Làm như vậy669 lần thì ta sẽ có 2016 mẩu giấy nhỏ.

Bài 9. Trong mặt phẳng cho 10 điểm đôi một phân biệt sao cho bất kỳ 4 điểm nào trong 10điểm đã cho cũng có 3 điểm thẳng hàng. Chứng minh rằng ta có thể bỏ đi một điểm trong 10điểm đã cho để có 9 điểm còn lại thuộc một đường thẳng.

(Khánh Hòa)

Lời giải

Ta xét các trường hợp sau:TH1: Tất cả các điểm trong 10 điểm đó đều nằm trên 1 đường thẳng. Trong trường hợp này, ta

bỏ điểm nào đi cũng thỏa mãn yêu cầu bài toán.TH2: Các điểm không đồng thời nằm trên 1 đường thẳng. Vậy tồn tại 3 trong 10 điểm không

thẳng hàng. Giả sử đó là điểm A,B,C.

Page 121: 1 ¾ F - · PDF fileđang chuẩn bị cho kì thi tuyển sinh vào các lớp 10 ... phẩm “Tuyển tập các bài toán trong đề thi ... đam mê với các môn học

THCM

N

2.5 Tổ hợp 121

Xét 1 điểm nào đó khác A,B,C, giả sử đó là D. Theo giả thiết thì D phải thuộc 1 trong 3đường thẳng AB,BC,CA, giả sử đó là đường BC. Nhận xét rằng, với mọi điểm X bất kìthuộc 6 điểm còn lại khác A,B,C,D thì điểm X đó cũng phải thuộc đường thẳng BC. Thậtvậy, nếu X thuộc đoạn AC thì 4 điểm X ,A,B,D không có 3 điểm nào thẳng, mâu thuẫnvới giả thiết. Còn nếu X thuộc AB thì 4 điểm X ,A,C,D cũng không có 3 điểm nào thẳnghàng, cũng mâu thuẫn với giả thiết. Do đó X sẽ phải thuộc vào BC. Vậy, ngoài điểm A ra,tất cả các điểm còn lại đều thẳng hàng. Do đó, ta chỉ cần bỏ điểm A đi là thỏa mãn yêucầu bài toán.

Bài 10. Số A được tạo thành bởi các chữ số viết liền nhau bao gồm các số nguyên dương từ 1đến 60 theo thứ tự từ nhỏ đến lớn: A = 12345678910. . .585960. Ta xóa 100 chữa số của Asao cho số tạo thành bởi các chữ số còn lại là số nhỏ nhất (không thay đổi trật tự của các chữsố ban đầu). Hãy tìm số nhỏ nhất được tạo thành đó.

(Long An)

Lời giải

Số chữ số của A là 9+(60−10+1).2 = 111 chữ số, suy ra xóa 100 chữ số thì còn lại 11 chữ số.Để số mới có 11 chữ số nhỏ nhất thì ta phải chọn 5 chữ số đầu tiên bên trái là 5 chữ số 0 (nghĩa làxóa hết các chữ số khác 0 từ 1 đến 50). 11 chữ số còn lại sau khi xóa 100 chữ số là 00000123450.Vậy số cần tìm là 123450.

Bài 11. Trên bảng ban đầu ghi số 2 và số 4. Ta thực hiện cách viết thêm các số lên bảngnhư sau: trên bảng đã có 2 số, giả sử là a,b (a khác b), ta viết thêm lên bảng số có giá trịlà a+b+ab. Hỏi với cách thực hiện như vậy, trên bảng có thể xuất hiện số 2016 được haykhông ? Giải thích.

(Nam Định)

Lời giải

Muốn xuất hiện số 2016, thì trên bảng phải có hai số a,b thỏa: a+b+ab = 2016 với a,b ∈ N∗.Ta có: a+b+ab = 2016⇒ ab+a+b+1 = 2017 hay (a+1)(b+1) = 2017. Mà 2017 là sốnguyên tố, vậy a+1 = 1 hoặc b+1 = 1⇒ a = 0 hoặc b = 0, vô lí vì a,b ∈ N∗.Vậy không thể xuất hiện số 2016 trên bảng.

Page 122: 1 ¾ F - · PDF fileđang chuẩn bị cho kì thi tuyển sinh vào các lớp 10 ... phẩm “Tuyển tập các bài toán trong đề thi ... đam mê với các môn học

THCM

N

122 Chương 2. LỜI GIẢI

Bài 12. Cho 19 diểm phân biệt nằm trong 1 tam giác đều có cạnh bằng 3, trong đó không có3 điểm nào thẳng hàng. Chứng minh rằng luôn tìm được 1 tam giác có 3 đỉnh là 3 trong 19

điểm đã cho mà có diện tích không lớn hơn

√3

4.

(Phú Thọ

Lời giải

Gọi tam giác đều có cạnh bằng 3 đó là ABC. Chia tam giác ABC thành 9 tam giác đều, có

cạnh bằng 1 (gọi là tam giác nhỏ) thì mỗi tam giác đó có diện tích

√3

4. Vì có 19 điểm và 9

tam giác nhỏ nên theo nguyên lý Dirichlet, tồn tại 1 tam giác có chứa ít nhất 3 điểm. Giả sử 3

điểm đó là M,N,P. Khi đó, vì4MNP nằm trong một tam giác nhỏ nên SMNP ≤√

34

, và ta cóđpcm.

Bài 13. Trong 100 số tự nhiên từ 1 đến 100 hãy chọn n số (n≥ 2) sao cho 2 số phân biệt bấtkì được chọn có tổng chia hết cho 6. Hỏi có thể chọn n số thỏa mãn điều kiện trên với n lớnnhất là bao nhiêu?

(Quảng Bình)

Hướng dẫn

Xét 17 số: 3;9;15;21; . . . ;87;93;99. 2 số bất kì trong dãy trên đều có tổng chia hết cho 6.Giả sử ta có thể tìm n = 18 số a1;a2; . . . ;an thỏa điều kiện đề bài. Ta có:

a2 ≡−a1 (mod 6)

a3 ≡−a1 (mod 6)

. . .

an ≡−a1 (mod 6)

Suy ra a2 ≡ a3 ≡ . . . ≡ an (mod 6). Tương tự sẽ có a1 ≡ a3 ≡ . . . ≡ an (mod 6). Vậy a1 ≡a2 ≡ . . . ≡ an (mod 6). Mà a2 ≡ −a1 (mod 6)⇒ a1 ≡ −a1 (mod 6) hay 2a1

...6. Vậy a1 ≡a2 ≡ . . . ≡ an ≡ 0 (mod 6) hoặc a1 ≡ a2 ≡ . . . ≡ an ≡ 3 (mod 6).Giả sử có n≥ 18 số thỏa đề thì sẽ có 18 số chia hết cho 6 trong khoảng [1;100] hoặc 18 số chia6 dư 3 trong khoảng [1;100], vô lý vì lấy tất cả số chia hết cho 6 trong khoảng [1;100] là 16 số,còn lấy tất cả các số chia 6 dư 3 trong khoảng [1;100] là 17 số. Mà n = 17 thỏa. Vậy n = 17 làsố lớn nhất thỏa đề bài.

Page 123: 1 ¾ F - · PDF fileđang chuẩn bị cho kì thi tuyển sinh vào các lớp 10 ... phẩm “Tuyển tập các bài toán trong đề thi ... đam mê với các môn học

THCM

N

2.5 Tổ hợp 123

Bài 14. Tập hợp A ={

1;2;3; ...;3n−1;3n}

với n là số nguyên dương được gọi là tập hợpcân đối nếu có thể chia A thành n tập hợp con A1,A2, ...,An và thỏa mãn hai điều kiện sau:

i. Mỗi tập hợp Ai(i = 1;2; ...;n) gồm 3 số phân biệt và có một số bằng tổng hai số còn lại.ii. Mỗi tập hợp A1,A2, ...,An đôi một không có phần tử chung.

.Chứng minh rằng:

1. Tập A ={

1;2;3; ..;92;93}

không là tập cân đối.2. Tập A =

{1;2;3; ...;830;831

}là tập cân đối.

(Vĩnh Phúc)

Lời giải

1. Giả sử A = {1;2;3; . . . ;92;93} là tập hợp cân đối, khi đó mội tập Ai (với i = 1,2, . . . ,31)có dạng {x;y;x + y}, như vậy tổng ba phần tử trong Ai là số chẵn. Do đó tổng cácphần tử của tập A là số chẵn. Mà với A = {1;2;3; . . . ;92;93} có tổng các phần tử bằng93.94

2= 93.47 là số lẻ, mâu thuẫn. Vậy A là tập không cân đối.

2. Chúng ta có nhận xét sau:

Nhận xét. Nếu tập Sn = {1;2; . . . ;n} với n chia hết cho 3 là tập hợp cân đối thì tập S4nvà tập S4n+3 cũng là tập cân đối. Thật vậy,• Từ tập S4n ta chọn ra các tập con ba phần tử sau:

{1;2n+n;2n+n+1}.{3;2n+n−1;2n+n+2}.{5;2n+n−2;2n+n+3}....{2n−1;2n+1;4n}.

Rõ ràng các tập con này đều thỏa mãn có một phần tử bằng tổng hai phần tử cònlại. Còn lại các số sau trong tập S4n là 2;4;6; . . . ;2n. Tuy nhiên vì tập Sn cân đốinên tập {2;4;6; . . . ;2n} cũng cân đối. Vậy tập S4n là tập cân đối.• Tương tự từ tập S4n+3 ta chọn ra các tập con ba phần tử sau:

{1;2n+n+2;2n+n+3}.{3;2n+n+1;2n+n+4}.{5;2n+n;2n+n+5}....{2n+1;2n+2;4n+3}.

Và còn lại các số 2;4;6; . . . ;2n. Suy ra tập S4n+3 là tập cân đối.

Trở lại bài toán. Ta có:831 = 4.207+3.207 = 4.51+3.52 = 4.12+3.12 = 4.3.

Mà tập {1;2;3} là tập cân đối nên theo nhận xét: ta xây dựng được các tập hợp cân đốitheo quy trình sau:

{1;2;3}→ {1;2; ...;12}→ {1;2; ...;52}→ {1;2; ...;207}→ {1;2; ...;831}.

Do đó A = {1;2; . . . ;831} là tập hợp cân đối.